Download as pdf or txt
Download as pdf or txt
You are on page 1of 161

MTAP 100 ASSESSMENT #2

1st Semester AY 202-2023

1. All are true about Lipids, except:


1. Composed of carbon-nitrogen bonds
2. Provide stability to cell membrane
3. Insoluble to chloroform
4. Insoluble to water

A. 1 and 4
B. 1 and 3
C. 1 and 2
D. 1 only

LIPIDS - referred to as fats, composed mostly of carbon-hydrogen bonds


- Primary sources of fuel, provide stability to cell membrane and allow for transmembrane
transport.
- SOLUBLE: organic solvents (chloroform and ether)
- INSOLUBLE: blood and water

2. Statement 1: Lipids require special transport mechanisms for circulation in the blood.
Statement 2: Lipoprotein is the special transport mechanism used.
A. 1st statement is true, 2nd statement is false
B. 1st statement is false, 2nd statement is true
C. Both statements are true
D. Both statements are false

LIPIDS - referred to as fats, composed mostly of carbon-hydrogen bonds


- Primary sources of fuel, provide stability to cell membrane and allow for transmembrane
transport.
- They require special transport mechanism (LIPOPROTEINS) for circulation in the blood

3. All of the following are the major Lipids, except:


1. Phospholipids
2. Fat soluble vitamins
3. Lipoprotein
4. Fatty acids

A. 2 and 3 only
B. 1 only
C. 2 and 4 only
D. 3 only

Major Lipid: Phospholipids


- Fat soluble vitamins(ADEK)
- Phospholipids
- Fatty acids
- Triglycerides
- Cholesterol
Lipoprotein -its main purpose is to transport TAG and cholesterol to sites of energy and
utilization

1
4. All of the following are true about Phospholipid, except:
1. Originates in the kidney and intestine
2. Most abundant lipids derived from phosphatidic acid
3. It is an amphiphatic lipid
4. It is similar structure to triglycerides, except that they contain two fatty acids

A. 1
B. 2
C. 3
D. 4

Phospholipid - the most abundant lipids derived from phosphatidic acid


- Originates in the liver and intestine
- It is produced from the conjugation of two fatty acids and a phosphorylated glycerol
- It is an amphiphatic lipid - contains polar hydrophilic heads groups and nonpolar
hydrophobic fatty acid side chains.
- It is similar structure to triglycerides, except that they contain two fatty acids

5. It is the largest and least dense of the lipoprotein particles.


A. Chylomicron
B. High Density Lipoprotein
C. Low Density Lipoprotein
D. Very low density Lipoprotein

Chylomicron - the largest and the least dense of the lipoprotein particles.
- Density: < 0.95 kg/L
HDL- smallest lipoproteins but the most dense
- Density: <1.063-1.21 kg/L
LDL - DENSITY: 1.019-1.063 KG/l
VLDL - DENSITY: 0.95 - 1.006 Kg/L

6. It transports excess cholesterol from the tissues and returns to the liver.
A. Chylomicron
B. High Density Lipoprotein
C. Low Density Lipoprotein
D. Very low density Lipoprotein

CHYLOMICRON - it transports exogenous/dietary TAG to liver, muscles and fat depot


HDL-It transports excess cholesterol from the tissues and returns to the liver.
LDL- transports cholesterol to the peripheral tissues
VLDL - transports endogenous TAG form the liver to muscle, fat depots, and peripheral tissues.

7. It is a rare autosomal recessive disorder characterized by complete absence of HDL due to a


mutation in the ABCA1 gene on chromosome 9.
A. Niemann-pick disease
B. Tangier’s disease
C. Tay-sachs disease
D. Sitosterolemia

Tangier's disease - the mutation leads to an inability to effectively transfer cholesterol and
phospholipids from within the cell onto nascent apoA1 proteins in plasma.

2
Niemann-pick disease (lipid storage disease) - it is an inherited disorder of lipid metabolism, in
which there are accumulations of sphingomyelin in the bone marrow, spleen and lymph nodes.
- Involves deficiency of enzyme responsible for removing phosphorycholine form
sphingomyelin
Tay-sachs disease - is an inherited neurodegenerative disorder of lipid metabolism
characterized by a deficiency of the enzyme hexosaminidase A, which results in the
accumulation of sphingolipids in the brain.
Sitosterolemia - mutation in ABCG8 or ABCG5 gene, both located at chromosome 2p21.

8. It catalyzes the esterification of cholesterol from HDL; enables HDL to accumulate cholesterol
as cholesterol ester.
A. Lecithin Cholesterol Acyl Transferase
B. Lipoprotein Lipase
C. Endothelial lipase
D. Hepatic lipase

LCAT - It catalyzes the esterification of cholesterol from HDL; enables HDL to accumulate
cholesterol as cholesterol ester.
Lipoprotein Lipase - hydrolyzes TAG and cholesterol esters in lipoproteins
Endothelial lipase- hydrolyzes phospholipids and TAG in HDL
Hepatic lipase - Hydrolyzes TAG and phospholipids from HDL

9. Chemical method/ s used to determine triglycerides.


1. Van Handel and Zilversmith
2. Glycerol Kinase Method
3. Hantsch condensation
4. Libermann Burchardt reaction

A. 1 and 3
B. 2 and 3
C. 3 and 4
D. 1,2 and 3

Van Handel and Zilversmith - chemical method - colorimetric method for TAG
Glycerol Kinase Method - enzymatic method for TAG
Hantsch condensation - chemical method - Fluorometric method for TAG
Libermann Burchardt reaction - Chemical method for cholesterol

10. It is the reference method for quantification of lipoproteins.


A. Ultracentrifugation
B. Electrophoresis
C. Immunochemical methods
D. Chemical precipitation

Ultracentrifugation- It is the reference method for quantification of lipoproteins


Electrophoresis - electrophoretic pattern : HDL,VLDL,LDL, chylomicron (most anodal to fastest)
Immunochemical methods - it uses antibodies specific to epitopes on the lipoproteins
Chemical precipitation -it uses polyanions and divalent cations such as magnesium, calcium
and manganese.

11. All of the following causes decreased cholesterol, except:


1. Malnutrition
2. Alcoholism
3
3. Malabsorption syndrome
4. Severe burns

A. 1
B. 2
C. 3
D. 4

Decreased cholesterol
- Severe hepatocellular disease
- Malnutrition
- Severe burns
- Hyperthyroidism
- Malabsorption syndrome
Increased cholesterol
- Hyperlipoproteinemia type II,III, V
- Biliary cirrhosis
- Nephrotic syndrome
- Poorly controlled diabetes mellitus
- Alcoholism
- Primary hypothyroidism

12. VLDL remnant


A. Intermediate Density Lipoprotein
B. Lipoprotein A
C. Lipoprotein X
D. B-VLDL

Intermediate Density Lipoprotein - It is a product of VLDL catabolism - VLDL remnant


Lipoprotein A - sinking pre-B lipoprotein - due to electrophoretic mobility same as VLDL but
density like LDL
Lipoprotein X- It is an abnormal liprotein found in obstructive jaundice and LCAT deficiency
B-VLDL- abnormally migrating B-VLDL

13. It is also known as sinking pre-B lipoprotein


A. Intermediate Density Lipoprotein
B. Lipoprotein A
C. Lipoprotein X
D. B-VLDL

Intermediate Density Lipoprotein - It is a product of VLDL catabolism - VLDL remnant


Lipoprotein A - sinking pre-B lipoprotein - due to electrophoretic mobility same as VLDL but
density like LDL
Lipoprotein X- It is an abnormal liprotein found in obstructive jaundice and LCAT deficiency
B-VLDL- abnormally migrating B-VLDL

14. Statement 1: Fatty acids are very important source of energy


Statement 2: Fatty acids provide the substance for conversion to glucose.
A. 1st statement is true, 2nd statement is false
B. 1st statement is false, 2nd statement is true
C. Both statements are true
D. Both statements are false

4
Fatty acids are a very important source of energy and Fatty acids provide the substance for
conversion to glucose(gluconeogenesis)

15. The biuret method in measuring proteins is based on:


A. bonding of at least two peptide bonds with Cu2+
B. measurement of nitrogen content of proteins
C. protein error of indicator that produce colors when dyes bind proteins
D. reaction of phosphotungstomolybdic acid with protein

RATIO: The biuret procedure is recommended by the IFCC (International Federationof Cliical
Chemistry). The reaction is based on the principle that alkaline medium and the presence of
at least two peptide bonds, cupric ions will complex with groups involved int he peptide
bond to form a violet-colored chelate. Both proteins and peptides are measured in the biuret
method

Reagents:
1. Cupric ions - complexes with the peptide bond
2. Sodium potassium tartrate - forms complex with cupric ions to prevent their
precipitation in the alkaline solution; prevents turbidity
3. Potassium iodide - antioxidant; prevents autoreduction of Cu2+

*Lipemia should be avoided; considered an interference.

Biuret is not sensitive to protein levels below 0.1 g/dL; not sensitive enough to assay total
proteins in CSF, urine or transudates.

CHOICE B: Kjeldahl method: measurement of the nitrogen content of proteins as ammonium


ion by back titration following oxidation of proteins by sulfuric acid and heat.It assumes that
proteins average 16% nitrogen by weight. Protein in grams per deciliter is calculated by
multiplying protein nitrogen by 6.25. Kjeldahl method is reference method for total protein that is
used to assign a protein assay value to calibrators.

CHOICE C: Proteins have the ability to change the color of some acid-base indicators without
altering the pH

CHOICE D: Folin-Ciocalteu reaction

16. Which of the following is associated with hyperproteneimia?


A. Starvation
B. Liver failure
C. Nephrotic syndrome
D. Malignancy

RATIO: Malignant diseaseas are associated with increased production of acute phase proteins.
Starvation, liver failure and nephrotic syndrome = low protein levels
Starvation = since food is the primary source of proteins
Liver failure = since proteins are synthesized in the liver
Nephrotic syndrome = damage in the kidney cause proteins to be lost via the urine

17. A patient was diagnosed with emphysema. Which of the following electrophoretic patterns
is associated with the condition?

5
RATIO:
A: Reference/Normal Pattern
B. Monoclonal gammopathy
C. alpha-1-anti-trypsin deficiency (in cases of emphysema, juvenile hepatic cirrhosis)
D. Nephrotic syndrome

18. A patient is diagnosed with liver cirrhosis. Which of the following proteins is least affected?
A. IgG
B. Albumin
C. Hemoglobin
D. Ceruloplasmin

RATIO: proteins are produced in the liver. Any diseases that affect the synthetic ability of the
liver causes a significant decrease in the blood protein levels. Antibodies, however, are least
affected because they are primarily synthesized by plasma cells.

6
19. During microangiopathic hemolytic, which of the following proteins is expected to decrease?
A. Hemoglobin
B. Haptoglobin
C. Ceruloplasmin
D. C-reactive protein

RATIO: Haptoglobin is the transport protein of free hemoglobin. During intravascular hemolytic
such as in MAHA, free hemoglobin is released. To salvage hemoglobin, haptoglobin will bind
with it and bring hemoglobin in the liver for processing. Consequently, haptoglobin becomes
saturated and will decrease when hemolysis exceeds the salvage mechanism.

7
Ceruloplasmin - transports copper
CRP - first marker to increase during inflammation

20. A dye-binding technique is used for albumin measurement. Which of the following dye is
most preferred because of it is most specific?
A. Bromcresol purple
B. Bromcresol green
C. Methyl orange
D. Hydroxyazobenzene benzoic acid

RATIO: BCP is the most sPecific dye for albumin. BCG is the most sensitive dye.
Methyl orange and HABA are dyes that can be used.

Methyl orange is not specific for albumin and β-lipoproteins and some α1- and α2-globulins will
bind to this dye, which could result in falsely elevated values. HABA is more specific for
albumin, but severalcompounds, such as salicylates (aspirin), penicillin, conjugated bilirubin,
and sulfonamides, interfere with the binding of albumin to HABA, which can result in falsely low
values. BCG is not affected by interfering substances such as bilirubin and salicylates; however,
hemoglobin binds to BCG and can lead to falsely high albumin values. For example, for every
100 mg/dL of hemoglobin, albumin is increased by 0.1 g/dL. BCG has been reported to
overestimate low albumin values in patients when accompanied by an elevated α-globulin
fraction, such as occurs in nephrotic syndrome or end-stage renal disease. BCP binds
specifically to albumin and is not subject to most interferences, but does appear to be affected
by the presence of bilirubin. BCP is precise and exhibits excellent correlation with
immunodiffusion reference methods; however, BCP is not without its disadvantages. In patients
with renal insufficiency, the BCP method has been found to underestimate the serum albumin.

Additional notes:
Bromophenol blue, Ponceau S, amido black 10B, lissamine green, and Coomassie brilliant blue
are commonly used to stain protein bands after electrophoresis. Although dyebinding methods
are simple and fast, the unequal dye-binding response of individual proteins requires caution
when using this type of method with the complex mixture of proteins found in human serum .

8
21. This is the result of protein electrophoresis from a 45-year old patient. Which of the following
is an associated condition?

A. Multiple myeloma
B. Malignancy
C. Hepatic cirrhosis
D. Rheumatoid arthritis

RATIO: beta-gamma bridging. Hepatic cirrhosis produces a polyclonal gammopathy associated


with a high IgA levels. This obliterates the valley between beta and gamma zone.
Malignancy and RA = polyclonal gammopathy classified as chronic inflammatory. There is
increased in almost all APRs.
Multiple myeloma = produces a zone of restricted mobility usually in the gamma but sometimes
in the beta or alpha-2 region

22. A patient is suspected of having acute myocardial infarction. Which of the following markers
first increases during AMI?
A. Lactate dehydrogenase
B. Creatine Kinase-MB
C. Aspartate Transaminase
D. Myoglobin

RATIO: M-C-T-A-L (order of increase of cardiac markers). Myoglobin, CK-MB, Troponin I, AST,
LDH.

*Cardiac troponins are considered the gold standard for diagnosis. If the CK-MB and myoglobin
are normal but the troponin is elevated, it is possible that damage to the heart was minor or that
the injury took place more than 24hours in the past.

Cardiac troponins, particularly troponin I and troponin T, are quickly replacing total CK and CK-
MB measurements because they are more specific to cardiac tissue and remain elevated in the
circulation for a longer period of time.

Elevates within 3-12 hrs of onset, peak within 12-24 hours; remains elevated for 1-3 weeks.

9
23. What condition is most associated with the following results?
CK-MB = elevated
Troponin I = within reference interval

A. Duchene type muscular dystrophy


B. Acute myocardial infarction
C. Hemolytic anemia
D. Hepatitis

RATIO: As mentioned in the previous item, cardiac troponins are more specific to cardiac tissue
and remain elevated in the circulation following acute myocardial infarction.

If CK, CK-MB and myoglobin concentrations are normal but troponin is elevated, it is possible
that damage tot he heart was minor or that the injury took place more than 24 hours in the past.

When CK is elevated, but troponin is within the reference interval, it is likely that symptoms
present due to a cause other than acute myocardial infarction, such as skeletal muscle injury, as
in the case of Duchene musculary dystrophy.

24. Nephrotic syndrome is a protein-losing condition. Which of the following acute phase
reactants remain the circulation during nephrotic syndrome?
A. Alpha-2-macroglobulin
B. Albumin
C. Haptoglobin
D. Cerulopasmin

RATIO: In nephrotic syndrome, the problem is within the podocytes of the kidneys, which allows
the excretion of proteins via the urine. Hence in nephrotic syndrome, it is protein losing via the
kidneys and manifest as proteinuria. Most of the proteins are excreted, except alpha-2-
macroglobulin because primarily of its size. That is why during protein electrophoresis in
nephrotic syndrome, most bands are decreased, with a relative increase in alpa-2 band
because alpha-2-macroglobulin remains in the circulation.

25. Which of the following is true about the condition associated with the electrophoresis pattern
below:

1. It is an acquired disorder associated with two distinct bands in the albumin region
2. It is also called pre-albuminemia
3. It is termed bisalbuminemia

10
A. 1,2,3
B. 1,3 only
C. 2,3 only
D. 3 only

RATIO: Bisalbuminemia is a genetic condition resulting from a mutation. The albumin produced
because of the mutation has unusual molecular characteristics. Bisalbuminemia is
demonstrated by the presence of two albumin bands during electrophoresis, instead of the
single band usually observed. Pre-albumin is another type of protein that migrates in the region
farther from the albumin region.

26. Which of the following describes the synthetic function of the liver organ?

A. example is gluconeogenesis
B. conjugation of bilirubin to be excreted
C. release of bile acids.
D. proteins, albumins, cholinesterase, coagulation proteins.

Ratio: The liver is capable of performing a lot of function, METABOLISM, REMOVAL OF


WASTE, and SYNTHETIC function. Synthetic function means the capacity of the liver to
synthesize. And these proteins, albumins, cholinesterase and coagulation proteins are
synthesized by the liver.

27. Which of the following precautions should be followed when collecting blood samples for
bilirubin testing?
A. Specimen should be whole blood.
B. Must be kept away from direct exposure to light.
C. Specimen should be transported with ICE.
D. A and B.

Ratio: Bilirubin is easily oxidized by light. Hence, specimen should be kept away from direct
exposure to light.

28. What is the end product of Van den bergh Reaction in bilirubin detection?
A. Potassium dichromate
B. Azobilirubin
C. Hippuric Acid
D. Sodium benzoate

Ratio:

11
29. What is the end color of Evelyn and Malloy Assay?
A. Blue Color
B. Reddish Purple
C. Blue-green
D. colorless

Ratio:

30. What is the reagent used to dissociate B1 from albumin in Evelyn-Malloy Assay?
A. Methanol
B. Caffeine
C. Sodium Benzoate
D. B and C

Ratio:

31. Which of the following conditions causes pre-hepatic hyperbilirubinemia?


A. Hemolytic anemia
B. Dubin-Johnson Syndrome
C. Rotor Syndrome
D. Cholelithiasis

Ratio:

12
32. Which of the following conditions causes post-hepatic hyperbilirubinemia?
A. Hemolytic anemia
B. Dubin-Johnson Syndrome
C. Rotor Syndrome
D. Cholelithiasis

Ratio:

33. Which of the following is an end-product of amino acid and nucleic acid metabolism?
A. Bilirubin
B. Fatty acid
C. Ammonia
D. None of the above

Ratio:

34. Which of the following enzymes are used to assess the liver function test?
A. ALP, ALT, CHS and LD
B. ALP only
C. ALP and ALT only
D. CHS and ALT only
13
Ratio: ALP - Alkaline phosphatase, ALT - Alanine transferase, CHS - cholinesterase, and LD is
Lactate Dehydrogenase. All are enzymes of liver.

35. Elevated alkaline phosphatase is indicative of


A. obstructive processes
B. chronic alcoholism
C. liver cancer
D. none of the above

Ratio:

36. Elevated gamma-glutamyl transferase is indicative of


A. obstructive processes
B. chronic alcoholism
C. liver cancer
D. none of the above

14
Ratio:

37. Which of the following IS NOT an oxidoreductase?


A. Lactate dehydrogenase
B. Glucose-6-phosphate dehydrogenase
C. Acid Phosphatase
D. Malate dehydrogenase

Ratio: LD, G6PD and MD is oxidoreductase, whereas ACP is hydrolase

38. Which of the following is NOT a transferase?


A. AST
B. CK
C. ALP
D. GGT

Ratio: AST, CK and GGT are transferases, ALP is hydrolase

39. What is the IUB and EC number of the enzyme ALP?


A. Hydrolase, EC 3.1.3.1
B. Hydrolase, EC 3.4.15.1
C. Hydrolase, EC 3.1.3.2
D. Hydrolase, EC 3.1.1.7

40. What is the IUB and EC number of the enzyme AST?


A. transferase, EC 2.6.1.1
B. transferase, EC 2.7.3.2
C. transferase, EC 2.3.2.1
D. transferase, EC 2.6.1.2

Choices for 41-46.


A. Both statements are true
B. Both statements are false
C. Only one statement is correct

C41. (1) If ionic strength is too high, enzyme activity drops


(2) Heparin inhibits AMS and ALT.

Ratio:
Ionic strength is the capacity of the atoms to bond to each other, hence, even with enzyme to
catalyze the breakdown of molecules, still it does not supersede the ionic strength.

15
A42. (1) EDTA inhibits many enzyme activity
(2) Fluoride does not affect enzyme activity.

Ratio: EDTA is a chelator, which when it chelates, it affects enzyme activity (especially thouse
with Mg and Zn as inclusion). Fluoride is known to be an enzyme disruptor.

C43 (1) End-point analysis the simplest and most widely used technique in Enzyme
Analysis
(2) ACP is a hydrolytic enzyme secreted by a number of cells.

Ratio: Enzyme analysis is kinetic in nature as we are measuring the enzyme activity.
ACP is secreted primarily from prostate gland and prostatic secretions, but smaller amounts are
found in spleen, bone marrow, erythrocytes, liver, kidneys and platelets.

A44. (1) Hudson method required p-Nitrophenylphospate as substrate.


(2) RIA and EIA is the best method for prostatic ACP.

Ratio:

A45. (1) Prostatic carcinoma will yield prostatic ACP


(2) Hairy Cell Leukemia will also yield elevated ACP

C46. (1) LD is a zinc containing enzyme


(2) LD is found exclusively in heart muscle.

Ratio:

47. Which of the following method detects isoenzymes of LD


A. Electrophoresis
B. Chemical method
C. RIA
16
D. GC-MS

Ratio: Electrophoresis is the most commonly used to determine isoenzymes because of its band
separation capacity.

48. Which of the following is the correct order of isoenzymes of LD in descending order:

A. LD2, LD1, LD4, LD3, LD5


B. LD5, LD4, LD3, LD2, LD1
C. LD2, LD1, LD3, LD4, LD5
D. None of the above

49. What is the absorbance of Oliver-Rosalki technique for Ck?

A. 540 nm
B. 340 nm
C. 260 nm
D. none of the above

Ratio:

17
50. Which of the following does not cause elevated CK?
A. MI
B. Alcoholic myopathy
C. Rhabdomyolysis
D. Leukemia

51. Acute pancreatitis may cause _____ elevation of AMS.


A. 10X
B. 4-6X
C. 2X
D. none of the above

Ratio:

52. According to their site of action, which hormone acts locally by diffusing from its origin to
target cells in the neighborhood and affects its function.
A. Autocrine
B. Intracrine
C. Paracrine
D. Endocrine
Ratio: Autocrine- acts within the cell of origin resulting in self-regulation of its function;
Endocrine- targets a distant cell through the bloodstream; Intracrine- acts inside a cell,
regulating intracellular events.

53. Intracellular receptors are:


A. composed of peptides and proteins and are hydrophilic

18
B. composed of cholesterol components and are hydrophilic
C. composed of cholesterol components and are hydrophobic
D. composed of proteins and peptides and are hydrophobia
Ratio: Intracellular receptors are receptor proteins found on the inside of the cell, typically in the
cytoplasm or nucleus. In most cases, the ligands of intracellular receptors are small,
hydrophobic (water-hating) molecules, since they must be able to cross the plasma membrane
with carrier proteins in order to reach their receptors. Intracellular receptors are the primary
receptors for steroid hormones.

54. Based on the structure/composition, which type makes up the majority of hormones?
A. Peptide hormones
B. Steroid hormones
C. Amino acid derivatives
D. both a and c
Ratio: Most of the hormones in the body are polypeptides and proteins (Guyton and Hall 14 th
Edition). Peptide hormones represent a major class of hormones that are made from amino
acids by specialized endocrine glands.

55. Which type of hormone facilitates diffusion before attachment to a specific receptor within
the cell?
A. Peptide hormones
B. Steroid hormones
C. Amino acid derivatives
D. both a and c
(Same ratio with item no. 2)
Ratio: Intracellular receptors are receptor proteins found on the inside of the cell, typically in the
cytoplasm or nucleus. In most cases, the ligands of intracellular receptors are small,
hydrophobic (water-hating) molecules, since they must be able to cross the plasma membrane
with carrier proteins in order to reach their receptors. Intracellular receptors are the primary
receptors for steroid hormones.

56. The following are examples of amino-acid derived hormones, except:


A. thyroxine
B. melatonin
C. serotonin
D. oxytocin
Other examples of amino-acid derived hormones: catecholamines, serotonin, melatonin, and
the thyroid hormones thyroxine and triiodothyronine. Oxytocin is a cyclic nonapeptide which
plays a critical role in lactation and major role in labor and parturition. It is also associated with
empathy, trust, sexual activity, and relationship-building. Thus, it is also referred to as the “love
hormone”.

57. Which of the following tissues does not secrete steroid hormones?
A. Ovaries
B. Pituitary gland
C. Testes
D. Adrenal cortex
Ratio: The hormones of the pituitary gland are protein or polypeptide in nature and vary in
complexity.

19
58. Posterior pituitary gland stores and releases which of the following tropic hormone?
A. TSH
B. Prolactin
C. Growth hormone
D. ADH
Ratio: Posterior pituitary gland stores and releases two hormones: oxytocin and antidiuretic
hormone (ADH or vasopressin). Hypothalamus makes oxytocin and ADH and then stimulates
the posterior pituitary when to store and release them by sending nerve signals through the
pituitary stalk.

59. As a counter hormone of PTH, calcitonin is released by the thyroid in response to:
A. Hypercalcemia
B. Hypocalcemia
C. Hypophosphatemia
D. Hypocalcemia and hypophosphatemia
Ratio: Calcitonin is secreted by the parafollicular cells of the thyroid gland. This hormone
opposes the action of the parathyroid glands by reducing the calcium level in the blood. If blood
calcium becomes too high, calcitonin is secreted until calcium ion levels decrease to normal.
- Parathyroid hormone is the most important regulator of blood calcium levels. It has specialized
calcium sensing receptors (CSRs) that respond to rising or falling calcium levels by increasing
or decreasing PTH secretion, respectively. PTH mobilizes calcium from bone by increasing
bone resorption. In response to low blood calcium levels, its effect is to increase those levels.

60. Secretion of hormones by the anterior pituitary may be controlled by the circulating levels of
hormones from the respective target gland, as well as hormones secreted by what organ?
A. Posterior lobe of the pituitary gland
B. Intermediate lobe of the pituitary
C. Hypothalamus
D. Adrenal medulla
Ratio: The hypothalamus secretes hormones that stimulate or suppress the release of
hormones in the pituitary gland, in addition to controlling water balance, sleep, temperature,
appetite, and blood pressure.

61. The target cells stimulated by angiotensin are found in the:


A. ovary
B. testes
C. adrenal cortex
D. parathyroid
E. adenohypophysis
Ratio: Aldosterone secretion in adrenal cortex is mainly regulated by the renin-angiotensin
system which functions to regulate blood pressure and sodium balance. Kidneys release renin
which cleaves angiotensin 1 from angiotensinogen. Angiotensin- converting enzyme (ACE)
converts angiotensin I to angiotensin II, which acts as a powerful vasoconstrictor and stimulates
aldosterone release.

62. GnRH stimulates the secretion of


A. Prolactin and TSH
B. Growth hormone
C. Luteinizing hormone
D. ACTH

20
Ratio: Gonadotropin-Releasing Hormone (GnRH) stimulates both LH and FSH production;
Thyrotropin-Releasing Hormone (TRH) stimulates the secretion of both TSH and Prolactin;
Growth Hormone Releasing Hormone (GHRH) stimulates the production of Growth hormone;
Corticotropin-releasing hormone, a main stimulus for ACTH secretion.

63. Which of the following produces a hormone that serves as a first responder to stress by
acting within seconds to promote the fight-or-flight response?
A. Pituitary Gland
B. Adrenal Cortex
C. Adrenal Medulla
D. Thyroid Gland
Ratio: In response to sympathetic stimulation, the medulla secretes catecholamines directly into
the circulation which are responsible to promote the body’s fight or flight response.

64. The presence of a very high titer for antithyroglobulin antibodies and the detection of
antithyroid peroxidase antibodies is highly suggestive of what disorder?
A. Graves’ disease
B. Hashimoto’s thyroiditis
C. T3 thyrotoxicosis
D. Thyroid adenoma
Ratio:
- Thyroid peroxidase is an enzyme localized in the thyroid cell membrane which plays an
important role in thyroid hormone biosynthesis. TPO assists the chemical reaction that adds
iodine to a protein called thyroglobulin, a critical step in generating thyroid hormones.
- Thyroglobulin acts as a substrate for the synthesis of the thyroid hormones thyroxine (T4) and
triiodothyronine (T3), as well as for iodide storage and the storage of the inactive forms of
thyroid hormone within the follicular lumen of a thyroid follicle.
- Hashimoto’s thyroiditis aka Chronic Lymphocytic thyroiditis is the most common cause of
hypothyroidism. In this condition, antibodies of TPO and Thyroglobulin lead to decreased thyroid
hormone production by the thyroid gland.

65. How is primary hypocortisolism differentiated from secondary hypocortisolism?


A. ACTH is elevated in primary and decreased in secondary
B. ACTH is decreased in primary and elevated in secondary
C. Cortisol is elevated in primary and decreased in secondary
D. Cortisol is decreased in primary and elevated in secondary
Ratio: Addison disease, also known as primary adrenal Insufficiency, hypoadrenalism or
hypocortisolism, is a chronic endocrine disorder in which the adrenal glands fail to produce
adequate levels of steroid hormones, mostly cortisol and to a certain degree aldosterone. In
primary hypocortisolism, the cortisol level is low and ACTH level is high.
- Secondary adrenal insufficiency or hypocortisolism occurs when the pituitary gland doesn't
make enough of the hormone ACTH. The adrenal glands then don't make enough cortisol.
(ACTH and cortisol level is low)

21
66. Considering that hyperglycemia normally suppresses GH secretion, glucose administration
may be used for the diagnosis of:
A. Hypoadrenalism
B. Acromegaly
C. Cushing’s disease
D. Conn’s syndrome
Ratio: Acromegaly results from pathologic or autonomous growth hormone excess and, in the
vast majority of patients, is a result of a pituitary tumor; Hypoadrenalism aka Addison’s disease-
Cortisol insufficiency; Cushing’s disease- a type of Cushing syndrome that is caused by a
benign tumor in the pituitary gland that secretes too much ACTH resulting to hypercortisolism;
Conn’s syndrome is a rare condition caused by overproduction of aldosterone.

67. Which is the most widely used screening test for Cushing’s syndrome?
A. Dexamethasone suppression test
B. Cosyntropin stimulation test
C. Captopril suppression test
D. Metyrapone stimulation test
Ratio: Dexamethasone suppression test is the most widely used screening test for Cushing’s
syndrome. Dexamethasone acts as an exogenous cortisol substitute suppressing ACTH if the
pituitary gland is normal and cortisol secretion if the adrenal gland is normal.

68. Which of the following conditions is characterized by primary hyperaldosteronism caused by


adrenal adenoma, carcinoma, or hyperplasia?
A. Cushing’s syndrome
B. Addison’s disease
C. Conn’s syndrome
D. Cushing’s disease
Ratio: Conn’s syndrome is a rare condition caused by overproduction of aldosterone. It can be
caused by either hyperactivity in one adrenal gland (unilateral disease) or both adrenal gland
(bilateral disease). Unilateral disease is usually caused by an aldosterone producing adenoma
(benign tumor).

69. Which of the following constitutes the second line of control of thyroid hormone synthesis
and secretion?
A. Hypothalamus
B. Adenohypophysis
C. Neurohypophysis
D. Thyroid follicles
Ratio: Anterior pituitary gland (Adenohypophysis); Posterior pituitary gland Neurohypophysis
- Thyroid gland is the first line of control of thyroid hormone synthesis. The second line of control
is the Adenohypophysis which secretes TSH that stimulates the thyroid gland to synthesize and
secrete thyroid hormone.

22
70. The concentration of serum TSH is characteristically low in:

A. Primary and secondary hypothyroidism


B. Primary and secondary hyperthyroidism
C. Primary hyperthyroidism and secondary hypothyroidism
D. Primary hypothyroidism and secondary hyperthyroidism
Ratio: Hyperthyroidism is overactivity of thyroid gland which is characterized by high levels of
serum thyroxine and triiodothyronine, and low levels of thyroid-stimulating hormone. Secondary,
or central hypothyroidism is labeled when the thyroid gland itself is normal, and the pathology is
related to the pituitary gland (responsible for the secretion of TSH) or hypothalamus. Therefore
in secondary hypothyroidism, TSH and thyroid hormones are low.

71. Which of the following thyroid hormones is considered the most biologically active?
A. Free T3
B. Free T4
C. T3 bound to TBG
D. T4 bound to TBG
Ratio: T3 is three to eight times more metabolically active than T4 and often considered to be
the active form of thyroid hormone. They combined mainly with thyroxine-binding globulin (TBG)
and much less so with thyroxine-binding prealbumin and albumin.

72. A 65-year-old woman presents with fatigue, hypothermia, pericardial effusions, and hair
loss. Her thyroid function tests show a significantly elevated TSH and a low free T4. All of the
following laboratory test abnormalities may be associated with her underlying condition except:
A. Pernicious Anemia
B. Hyponatremia
C. Hypocholesterolemia
D. Elevated CPK levels
Ratio: The result indicates hypothyroidism (High TSH, Low FT4). Hypothyroidism can lead to a
variety of other abnormalities because of the diffuse distribution of thyroid hormone receptors
and many metabolic effects of thyroid hormone: Hyponatremia, Anemia, Elevated CPK levels
and hypercholesterolemia.

73. Which placental hormone has some structural homology with the anterior pituitary tropic
hormones?
A. HCG
B. HPL
C. Estriol
D. Estrone
Ratio: Human chorionic gonadotropin (hCG) is a hormone secreted by syncytiotrophoblast
tissue typically found in early embryos and which will eventually be part of the placenta during
pregnancy. It is a dimer consisting of a 145 amino acid beta-subunit that is unique to hCG and a
92 amino acid alpha-subunit. The alpha-subunit is identical to that for luteinizing hormone (LH),
follicle-stimulating hormone (FSH), and thyroid-stimulating hormone (TSH) secreted in anterior
pituitary gland.

23
TDM and Toxicology

74. In which of the cases below is qualitative analysis of the drug usually adequate?
A. To determine whether the dose of a drug with a low therapeutic index is likely to be
toxic.
B. To determine whether a patient is complying with the physician’s instructions.
C. To adjust dose if individual differences or disease alter expected response.
D. To determine whether the patient has been taking amphetamines.

Ratio: The purpose of therapeutic drug monitoring is to achieve a therapeutic blood drug level
rapidly and minimize the risk of drug toxicity caused by overdose. Therapeutic drug monitoring
is a quantitative procedure performed for drugs with a narrow therapeutic index (ratio of the
concentration producing the desired effect to the concentration producing toxicity). Drug groups
that require monitoring because of high risk of toxicity include aminoglycoside antibiotics,
anticonvulsants, antiarrhythmics, antiasthmatics, immunosuppressive agents used for transplant
rejection, and psychoactive drugs. When testing for abuse substances, the goal is usually to
determine whether the drug is present or absent. The most common approach is to compare the
result to a cutoff determined by measuring a standard containing the lowest level of drug that is
considered significant.

75. The term pharmacokinetics refers to the:


A. Relationship between drug dose and the drug blood level
B. Concentration of drug at its sites of action
C. Relationship between blood concentration and therapeutic response.
D. The relationship between blood and tissue drug levels.

Ratio: Pharmacokinetics is the mathematical expression of the relationship between drug dose
and drug blood level. When the appropriate formula is applied to quantitative measures of drug
dose, absorption, distribution, and elimination, the blood concentration can be accurately
determined.

76. The term pharmacodynamics is an expression of relationship between:


A. Dose and physiological effect
B. Drug concentration at target sites and physiological effect
C. Time and serum drug concentration
D. Blood and tissue drug levels

Ratio: Pharmacodynamics is the relationship between the drug concentration at the receptor
site (tissue concentration) and the response of the tissue to that drug. For example, the
relationship between lidocaine concentration in the heart muscle and the duration of the action
potential of Purkinje fibers.

77. Which specimen is the sample of choice for lead screening?


A. Whole blood
B. Hair
C. Serum
D. Urine

Ratio: Lead accumulates in RBCs, bones, and neural tissues, and whole blood, hair, and urine
are suitable for demonstrating lead toxicity. Greatest sensitivity is obtained by using whole
blood, which can detect exposure over time. Because lead is rapidly eliminated from plasma,
serum or plasma should not be used to test for lead exposure. Lead binds to sulfhydryl groups
of proteins such as delta-aminolevulinic acid (Δ-ALA) dehydratase and ferrochelatase and
24
interferes with heme synthesis. This results in increased free erythrocyte protoporphyrin,
erythrocyte zinc protoporphyrin, urinary coproporphyrin III, and δ aminolevulinic acid, which are
also useful markers for lead poisoning. When screening for lead poisoning in children, the
method of choice is graphite furnace atomic absorption spectrophotometry or inductively
coupled plasma mass spectroscopy because they offer the best analytical sensitivity.

78. Quantitative of a drug by gas chromatography-mass spectroscopy (GS-MS) is usually


performed in which mode?
A. Total ion chromatography
B. Selective ion monitoring
C. Ion subtraction
D. Selective reaction monitoring

Ratio: Most GC-MS instruments use an electron beam to split the drug emerging from the
column into its component ions. These are drawn into the mass analyzer, usually a vacuum
chamber containing two pairs of charged rods (a positive pair and a negative pair) called a
quadrupole analyzer. By changing the potential and radio frequency applied to the rods, the
travel of ions will vary depending upon their mass to charge (m/z) ratio. As ions emerge from the
mass filter, they are detected by an electron multiplier tube. CG-MS instruments can be
operated in two modes, total ion chromatography and selective ion monitoring. A total ion
chromatograph displays the retention time of all ions detected and their abundance. It is
primarily used for identification of unknown compounds. SIM mode measures the abundance of
one or more principal ions that provides sufficient specificity to eliminate potential interfering
substances and greater quantitative sensitivity

79. Which could account for drug toxicity following a normally prescribed dose?
1 - Decreased renal clearance caused by kidney disease
2 - Discontinuance of another drug
3 - Altered serum protein binding caused by disease

A. 1 and 2
B. 1 only
C. 1 and 3
D. 1, 2, and 3

Ratio: Therapeutic drug monitoring is necessary for drugs that have a narrow therapeutic index.
Individual differences alter pharmacokinetics, causing lack of correlation between dose and drug
blood level. These include age, diet, ingestion with or without food, genetic factors, exercise,
smoking, pregnancy, metabolism of other drugs, protein binding, and disease states.

80. The study of pharmacogenomics involves which type of testing?


A. family studies to determine the influence of drug resistance
B. Testing drugs with cell cultures to determine the minimum toxic dosage
C. Testing for single nucleotide polymorphisms known to affect drug metabolism
D. Comparison of dose response curves among family members

Ratio: Pharmacogenomics refers to the study of genes that affect the performance of a drug in
an individual. One method is to test for single nucleotide polymorphisms (SNPs) using DNA
microarrays in genes such as those that code for the cytochrome P450 enzymes involved in the
metabolism of many drugs. Genetic variations of one such enzyme may account for individual
pharmacokinetic differences and can be used to predict the efficacy of the drug.

25
81. Select the five pharmacological parameters that determine serum drug concentration.
A. Absorption, anabolism, perfusion, bioactivation, excretion
B. Liberation, equilibration, biotransformation, reabsorption, elimination
C. Liberation, absorption, distribution, metabolism, exertion
D. Ingestion, conjugation, integration, metabolism, elimination

Ratio: Liberation is the release of the drug and absorption is the transport of drug from the site
of administration to the blood. The percent of drug absorption and the rate of absorption
determine the bioavailable fraction, f. This is the fraction of the dose that reaches the blood.
Distribution refers to the delivery of the drug to the tissues. It involves dilution and equilibration
of the drug in various fluid compartments including the blood, and is influenced by binding to
proteins and blood cells. Metabolism is the process of chemical modification of the drug by cells.
This results in production of metabolites with altered activity and solubility. Excretion is the
process by which the drug and its metabolites are removed from the body

82. Which route of administration is associated with 100% bioavailability?


A. Sublingual
B. intramuscular
C. Oral
D. Intravenous

Ratio: When a drug is administered intravenously, all the drug enters the bloodstream, and
therefore, the bioavailable fraction is 1.0. All other routes of administration require absorption
through cells, and this process reduces the bioavailable fraction. The bioavailable fraction for a
drug given orally can be calculated by dividing the peak blood concentration after oral
administration by the peak drug concentration after IV administration. A value of 0.7 or higher is
desired for drugs given orally

83. The phrase first-pass hepatic metabolism means that:


A. 100% of a drug is excreted by the liver
B. All drug is inactivated by hepatic enzymes after one pass through the liver
C. Some drug is metabolized from the portal circulation, reducing bioavailability
D. The drug must be metabolized in the liver to n active form

Ratio: Drugs given orally enter the blood via the portal circulation and are transported directly to
the liver. Some drugs are excreted by the liver, and a fraction will be lost by hepatic excretion
before the drug reaches the general circulation. An example is propranolol, a β-blocker that
reduces heart rate and hypertension. The bioavailable fraction is 0.2–0.4 when given orally
because much of the drug is removed by first-pass hepatic metabolism.

84. For drugs with first-order elimination, which statement about drug clearance is true?
A. Clearance = elimination rate / serum level
B. It is most often performed by the liver
C. It is directly related to half-life
D. Clearance rate is independent of dose

Ratio: First-order elimination represents a linear relationship between the amount of drug
eliminated per hour and the blood level of drug. For drugs following linear kinetics, clearance
equals the elimination rate divided by the drug concentration in blood. When clearance (in
milligrams per hour) and f are known, the dose per hour needed to give a desired average drug
level at steady state can be calculated. Clearance is inversely related to the drug’s half-life and
is accomplished mainly by the kidneys.
26
85. The pH of the blood is maintained at 7.4. It is necessary to have:
A. 10:1 ratio of bicarbonate to carbonic acid
B. 20:1 ratio of bicarbonate to carbonic acid
C. 1:20 ratio of bicarbonate to carbonic acid
D. 20:1 ratio of carbonic acid to bicarbonate

RATIO: Blood pH is maintained at slightly alkaline levels (7.35-7.45, ~7.4) because of the
interaction of the primary buffer systems. In plasma, the bicarbonate-carbonic acid systme is
one of the principal buffers:

It consists of a weak acid (carbonic acid) and a slat of its conjugate base (bicarbonate).
Carbonic acid is a weak acid because it does not completely dissociate into H+ and HCO3–,
whereas a strong acid, such as HCl, completely dissociates into H+ and Cl– in solution. When
hydrogen ions are added to the bicarbonate–carbonic acid system, the HCO3 – will combine
with the H+ to form H2CO3 (Eq. 17-2). Therefore, bicarbonate–carbonic acid reaction is driven
to the right, increasing the amount of carbonic acid and consuming bicarbonate ion. Conversely
when a strong base is added, H2CO3 will combine with the OH– ions to form H2O and the
weaker conjugate base HCO3-. Although the bicarbonate–carbonic acid system has low
buffering capacity, it is still the most important buffer system in extracellular fluids for three
reasons: (1) H2CO3 dissociates into CO2 and H2O, allowing CO2 to be eliminated by the lungs
and H+ as water; (2) changes in CO2 modify the ventilation (respiratory) rate; and (3) HCO3 –
concentration can be changed by the kidneys. When the kidneys and lungs are functioning
properly, the ratio of HCO3– to H2CO3 is 20:1, corresponding to a pH of 7.40.

86. What is the blood pH, given the following data:


pCO2 = 44mmHG
TCO2 = 29 mmol/L

A. 6.28
B. 7.42
C. 6.76
D. 7.44

RATIO: pH = 6.1 + log (HCO3/(pCO2 x 0.03))

pH = 6.1 + log 27.68/1.32


pH = 6.1 + log 21
pH = 7.42

87. A sealed specimen is left at room temperature for 3 hours. What changes in the blood gas
parameters are expected?
A. pO2 decreases, pCO2 increases, pH decreases
B. pO2 increases, PCO2 increases, pH increases
C. pO2 decreases, PCO2 decreases, pH decreases
D. pO2 increases, PCO2 increases, pH decreases

RATIO: Quality Assurance during Blood gas measurement (Bishop 7ed, p. 976)
27
Notes highlight:
In most instances, the ideal collection device for arterial blood sampling is a 1- to 3-mL
self-filling, plastic, disposable syringe, containing the appropriate type and amount of
anticoagulant. Evacuated collection tubes are not appropriate for blood gases. While both dry
(lyophilized) and liquid lithium heparin are acceptable anticoagulants, the liquid form is not
recommended because liquid heparin can dilute the sample and possibly alter the sample due
to equilibration with room air.
Transport time and analysis should be minimal to reduce cellular metabolism.
Anaerobic condition results in oxygen and glucose consumption (pO2 decreases) and
carbon dioxide and lactate production (pCO2 increases thereby making the pH acidic =
decrease pH). Placing the filled syringe in an ice water slurry promptly after the draw minimizes
cell metabolism. There is potential for pO2 to increase due to oxygen diffusing from the water
through the pores of the plastic syringe. In addition, lower temperatures cause increased
oxygen solubility in blood and a left shift in the oxyhemoglobin dissociation curve resulting in
more oxygen combining with hemoglobin. As a consequence, when the sample is heated by the
blood gas analyzer, the measured pO2 is falsely elevated.

Factors affecting blood gases and pH measurement:


1. Temperature (most important) - maintained at 37 +/- 1 C
2. Hyperproteinemia - build up of proteins on the surface of the sensing membrane causes on
the variation in the reading of pO2.
3. Consumption of Oxygen because of bacterial contamination → false low pO2
4. Improper transport of specimen - transport should be on ice. Improper transport causes
variation in the pO2, pCO2 and pH.

88. In order to maintain electroneutrality in the red blood cells, bicarbonate leaves the RBC in
exchange of what ion?
A. Sodium
B. Potassium
C. Chloride
D. Phosphate

RATIO: The red blood cell membrane is permeable to both bicarbonate and chloride ions.
Chloride ions participate in buffering the blood by diffusing out of or into the red blood cells to
compensate for the ionic change that occurs when bicarbonate enters or leaves the red blood
cell. This is called the chloride shift.

The dissociation of H2CO3 causes development of a concentration gradient due to the increase
in HCO3 – concentration; accordingly, biocarbonate ions diffuse from the red cells and diffuse
into the plasma. To maintain electroneutrality (the same number of positively and negatively
charged ions on each side of the red cell membrane), chloride diffuses into the cell. This is
known as the chloride shift (also known as the Hamburger shift). The H+ generated is buffered
by binding with the recently deoxygenated hemoglobin molecules. This is why the abbreviation
HHb is used to designate deoxygenated hemoglobin, because it has hydrogen bound instead of
oxygen. (Bishop 7ed, p. 947)

28
89. A 30-year old unknown male came to the emergency department unconscious. He had
shallow breaths, and has clammy cold skin. His blood gas analysis results are:
pH = 7.29
pCO2 = 50 mmHG
HCO3 = 25 mmol/L

What do these results indicate?

A. Partially compensated metabolic alkalosis


B. Uncompensated respiratory acidosis
C. A dual problem of acidosis
D. An error in one of the blood gas measurements

RATIO:

pH is 7.29 = acidosis
pCO2 = increased (indicating the problem is respiratory)
HCO3 = within reference range; indicating the kidneys have not compensated for the acidosis.

Respiratory acidosis compensation: kidneys retain HCO3 (so increase in HCO3 should be
observed when kidneys will compensate)

90. A 30-year old unknown male came to the emergency department unconscious. He had
shallow breaths, and has clammy cold skin. His blood gas analysis results are:
pH = 7.29
pCO2 = 50 mmHG
HCO3 = 25 mmol/L

In this case, what is the compensatory mechanism?

A. Hypoventiliation
B. Decreased reabsorption of bicarbonate by the kidneys
C. Increased Na+/H+ exchange by the kidneys, with the excretion of H+
D. Decreased ammonia formation by the kidneys

RATIO: Primary respiratory acidosis results from a decrease in alveolar ventilation


(hypoventilation), causing a decreased elimination of CO2 by the lungs. In respiratory acidosis,
the initial defect is associated with the lungs, and the organ responsible for compensation are
the kidneys. They act by: increasing the production of ammonia, in exchange of Na for H+ with
the excretion of H+, and the reabsorption of H+.

In primary respiratory acidosis, the compensation occurs through metabolic processes.


However, as previously mentioned, the metabolic processes, unlike the respiratory, require
hours to days to affect the pH. Therefore, it takes days to weeks for full compensation to occur.
29
The kidneys respond to respiratory acidosis by increasing the excretion of H+ and reclamation
of HCO3 –. As a result of this, HCO3 – in the blood increases causing the pH to return to
normal.

91. The following blood gas results are obtained from a 20-year old female brought to the
hospital. She was hyperventilating.
pH = 7.58
pCO2 = 55 mmHg
HCO3 = 18 mmolL

What do the results indicate?

A. Partially compensated metabolic alkalosis


B. Uncompensated respiratory acidosis
C. A dual problem of acidosis
D. An error in one of the blood gas measurements

RATIO: Here the pH and case information indicate alkalosis, but both the metabolic
(decreased HCO3) and respiratory (increased PCO2) components indicate acidosis. Most likely
there is a problem/error in one or more of the measurements.

92. Which data sheet in the laboratory contains basic information about the specific chemical or
product like its expiration dates, health hazard and protection information?
A. Material safety data sheet
B. Personal data sheet
C. Exposure control plan
D. Chemical hygiene plan

Ratio: . A summary of the MSDS information requirements includes the following: ■ Product
name and identification ■ Hazardous ingredients ■ Permissible exposure limit (PEL) ■ Physical
and chemical data ■ Health hazard data and carcinogenic potential ■ Primary routes of entry ■
Fire and explosion hazards ■ Reactivity data ■ Spill and disposal procedures ■ PPE
recommendations ■ Handling ■ Emergency and first aid procedures ■ Storage and
transportation precautions ■ Chemical manufacturer’s name, address, and telephone number ■
Special information section

93. Which of the following regulations could relate to laboratory safety?


1 - Clean Water Act
2 – Resource Conservation and Recovery Act
3 – Toxic Substances Control Act

A. 1 only
B. 1 and 2
C. 1 and 3
D. 1, 2, and 3

Ratio: There are other federal regulations relating to laboratory safety, such as the Clean Water
Act, the Resource Conservation and Recovery Act, and the Toxic Substances Control Act.
(Bishop)

30
94. Which of the following protective measures would best use for germicidal lamps used in
biologic safety eye and skin protection?
A. Engineered shielding and posted pacemaker warning.
B. Containment and appropriate warning labels
C. Filters, diffusers, and nonreflective surfaces
D. Eye and skin protection; UV warning labels

Ratio:

95. Any blood, body fluid, or other potentially infectious material spill must be cleaned up, and
the area or equipment must be disinfected immediately. Cleanup includes:
1- Common aqueous detergent
2 - Disinfect the spill site using 1% bleach

A. 1 only
b. 2 only
C. Both 1 and 2
D. Neither 1 nor 2

Ratio: Any blood, body fluid, or other potentially infectious material spill must be cleaned up, and
the area or equipment must be disinfected immediately. Cleanup includes the following
recommendations: ■ Wear appropriate protective equipment. ■ Use mechanical devices to pick
up broken glass or other sharp objects. ■ Absorb the spill with paper towels, gauze pads, or
tissue. ■ Clean the spill site using a common aqueous detergent. ■ Disinfect the spill site using
approved disinfectant or 10% bleach, using appropriate contact time

96. If a chemical spill occurs in the laboratory, first step to do is:


A. Clean the spill
B. Call for help
C. Write immediately a report
D. Assist or evacuate personnel

Ratio: Strict attention to good laboratory technique can help prevent chemical spills. However,
emergency procedures should be established to handle any accidents. If a spill occurs, the first
step should be to assist/evacuate personnel, and then confinement and cleanup of the spill can
begin. There are several commercial spill kits available for neutralizing and absorbing spilled
chemical solutions. However, no single kit is suitable for all types of spills. Emergency
procedures for spills should also include a reporting system

31
97. Fires have been divided into four classes based on the nature of the combustible material
and requirements for extinguishment. Which of the following belong to Class A?
1 – elecrical equipment
2 – wood
3 – plastic

A. 1 only
B. 2 only
C. 2 and 3
D. 1 and 2

Ratio: Fires have been divided into four classes based on the nature of the combustible material
and requirements for extinguishment: Class A: ordinary combustible solid materials, such as
paper, wood, plastic, and fabric Class B: flammable liquids/gases and combustible petroleum
products Class C: energized electrical equipment Class D: combustible/reactive metals, such as
magnesium, sodium, and potassium.

98. Which of the following statements is correct? Choose the best answer.
A. Liquid wastes and flammable solvents must be collected in approved containers and
segregated into compatible classes.
B. If recycling is not feasible, disposal arrangements of flammable material can be made
by the laboratory personnel.
C. Needles should be transported, recapped, bent, or broken by hand.
D. Radioactive wastes can be treated and disposed by incineration

Ratio:

99. Which of the following statements is correct? Choose the best answer.
A. Biohazard hoods remove particles that may be harmful to the employee who is
working with infective biologic specimens.
B. Biologic specimens should remain capped during centrifugation to prevent spillage.
C. Clean any spill site using a common aqueous detergent.
D. All of the choices.

100. Material Data Safety Sheets:


1 - are written by manufacturers
2 – are written by government agencies

A. 1 is correct
B. 2 is correct
C. 1 and 2 are correct
D. Neither 1 nor 2 is correct

32
MTAP 100 ASSESSMENT #1
1st Semester AY 202-2023

1. Which of the following tests would require a blood collected in royal blue top tube?
A. Pyruvate
B. Hepatitis B antibody
C. Free hemoglobin
D. Zinc

RATIO: Zinc is one of the most important trace element. Measurement of trace elements in the blood/serum
requires royal blue-top evacuated tube. Royal blue tube is made with special glass and stopper materials that
are free of trace element contamination as possible. Apart from trace element determination, royal blue is also
used for the determination of drug level and toxicology screen. Royal blue top tubes contain EDTA, heparin, or
no additive.

2. A neonate is due for screening to detect metabolic and genetic abnormalities. In what manner is blood
collected in this case?
A. Venipuncture via the median vein
B. Venipuncture via the dorsal vein
C. Capillary puncture from the lateral plantar surface of the heel
D. Capillary puncture from the 3rd or 4th finger

RATIO: Neonatal screening for metabolic and genetic abnormalities is done through newborn screening. It is
performed by capillary puncture (lateral plantar heel-stick). And blood is collected in filter paper. Puncture on
the 3rd or 4th finger of newborns is a big no-no because of the short distance between the skin surface and
bone.

3. Which parameter is most important to consider in order to maintain the integrity of a specimen for blood gas
analysis?
A. Exposure to light
B. Exposure to air
C. Ensure that the specimen is fasting
D. Ensure that the specimen is centrifuged immediately

RATIO: Blood gas analysis include the measurement of pO2 and pCO2. It is of utmost importance to not to
expose the sample to room air when collecting, transporting and making O2 measurements. Contamination of
the sample with room air (pO2 >150 mmHg) can result in significant error because leukocytes continue to
metabolize O2.

A. exposure to light is a concern for bilirubin


C. fasting specimen is best for fasting blood glucose and lipid panel
D. blood gas analysis does not involve centrifugation of the specimen

4. A patient is due for urine hormone measurement. Which of the following urine collection is best to perform in
this case?
A. 24-hour urine
B. Random urine
C. Midstream clean-catch urine
D. Routine urine specimen

RATIO: For urine hormone analysis, a 24-hour urine is collected to consider diurnal variations of hormones.
Refrigeration is the primary means of storage of samples. Collection is done example:
Upon waking up (6AM) urination - not included in the collection because this primarily belongs from the
previous day.
Urine after 6AM is collected until the following day. Last collection will be at 6AM the following day.

5. As a phlebotomist, what order of draw are you going to follow when the tests requested are: Prothrombin
Time, Complete Blood Count, Fasting Blood Sugar, Lipid Panel
1. Citrate tube
2. EDTA tube
3. Serum separator tube
4. Fluoride tube

A. 1 → 3 → 2 only
B. 3 → 2 → 1 only
C. 1 → 2 → 3 → 4
1
D. 3 → 1 → 2 → 4

RATIO: The tubes that you will need are: Citrate for prothrombin time, EDTA for CBC and Serum separator
tube for FBS and Lipid panel.

Any time a coagulation test other than PT or PTT is the first or only tube collected, a few mL of blood should be
drawn tino a nonaddtive tube or another coagulation tube before the coagulation specimen is collected. The
extra tube is called a “clear” or “discard” tube because it is used to remove tissue fluid from the needle and is
then thrown away.

6. During venipuncture, the patient was vigorously pumping his fist. Which of the following test/s is/are
affected?
1. Potassium
2. Creatine Kinase
3. Sodium
4. Alkaline phosphatase

A. 1, 2, 3, 4
B. 1, 2 only
C. 2, 3 only
D. 2, 4 only

RATIO: Potassium and creatine kinase are abundant in our muscles. Hence, vigorous pumping of the fist
causes both to leak from intracellular environment into the circulation. Sodium and ALP are not affected by
vigorous pumping of fist.

7. Hand hygiene during routine blood collection requires antiseptic cleaning with:
1. Formaldehyde
2. 20% phenol
3. 70% isopropyl alcohol
4. Iodine

A. 3,4 only
B. 2,3 only
C. 1,4 only
D. 3 only

RATIO: For routine venipuncture, 70% isopropyl alcohol may be used as an alternative to hand washing if
there is no visible soiling or blood contamination.

Additional info: during routine venipuncture, 70% isopropyl alcohol is the antiseptic of choice in cleaning the
venipuncture site. Povidone iodine is only used when the collection is for blood culture and blood donation.
Povidone iodine is never used in routine venipuncture as it can cause false elevation of Potassium, Uric Acid,
Phosphate

Benzalkonium chloride - used as disinfectant for mearument of blood alcohol levels

2
8. Allen’s test is a blood collection procedure performed:
A. In glucose tolerance test
B. In arterial blood gas test
C. In prothrombin time test
D. In hormone

RATIO: Allen’s test is performed prior to arterial puncture. It is a test to assess whether the ulnar artery can
provide enoch blood supply to the radial artery during collection.

9. Serum iron is one of the request for Patient XYZ. Which of the following is best to do during blood collection
for serum iron?
A. Collection is done early in the morning
B. Collection is done in 24-hour period to consider diurnal variation
C. Collection is using arterial puncture
D. Collection is through capillary puncture

RATIO: Iron (and cortisol) exhibits diurnal variation. Their levels peak early in the morning and decline in the
afternoon. The best time to collect for iron (and cortisol) is during early in the morning. Collection of sample in
the afternoon will result to false low levels.

10. Which of the following is true about tying of tourniquet during routine venipuncture>
A. Above the antecubital for 3 minutes
B. Above the antecubital for no more than 1 minute
C. Below the antecubital for about 1 minute
D. Below the antecubital for 3 minutes

RATIO: Tourniquet application should be 2-4 inches (equivalent to three fingers) above the puncture site
(above the antecubital area) for no longer than 1 minute.

Prolonged tourniquet application will result to hemoconcentration (thereby cause false increase in most cells
and analytes) and possibly hemolysis.

If tourniquet is not available, a blood pressure cuff may be used. During routine venipuncture, it is inflated at
60mmHg.

11. If a patient develops a hematoma during venipuncture, which of the following is the best course of action?
A. remove the needle and apply adequate pressure
B. apply pressure to the site and try to finish the collection
C. call the nurse immediately
D. readjust/redirect the needle and continue the collection

RATIO: If the needle punctures the vein through and through, it could possibly result to leakage of blood to the
surrounding tissues, resulting to hematoma formation. To prevent this, it is best to stop the collection and apply
pressure on the puncture site.

When the patient experiences discomfort (i.e.g pain, hematoma), stop the collection and apply appropriate
patient care. You may resume venipuncture on the other site with the permission of the patient. A medical
technologist is allowed to make 2 attempts before endorsing to another medtech.

12. When atoms absorbs or emits energy, what characteristic absorption or emission spectra is expected?

A. Line spectrum
B. Band spectra
C. Continuous spectra
D. Line and Band Spectrum

Ratio: Since ATOM is in its simplest form, it yields a specific energy equating to specific wavelength, hence a
line spectrum is expected. For MOLECULE (which consists of two or more atoms - creating a complex mixture
of atoms - a BAND Spectra is yielded. Whilst Light emitted by incandescent solids (tungsten or deuterium) it is
in a continuous spectra.

Bishop, 8th Edition, Page 312.


3
13. Which of the following best describes the Beer-Lambert Law?

A. The amount of light absorbed is inversely proportional to the concentration of the unknown substance.
B. The concentration of a substance is inversely proportional to the logarithm of transmitted light.
C. The concentration of a substance is directly proportional to the logarithm of transmitted light.
D. A and C are combined principles of Beer-Lambert Law.

Ratio:
Beer-Lambert Law states that the concentration of a substance is directly
proportional to the amount of light absorbed or inversely proportional to
the logarithm of the transmitted light.

Note:
● Some of the lights are absorbed, while the rest passes through
and is converted into an electrical signal.
● Percent transmittance is the ratio of the radiant energy transmitted
divided by the radiant energy incident of the sample.
● All light absorbed or blocked results in 0% Transmittance. A level
of 100% Transmittance is obtained IF NOT LIGHT IS ABSORBED.
● The %Transmittance measured in commercial
spectrophotometers is the ratio of the sample transmitted beam divided
by the blank transmitted beam.

14. Which of the following light sources for spectrophotometers are capable of providing continuous emission
of light down to 165 nm?
A. Deuterium discharge lamp
B. Low-pressure mercury lamps
C. Medium-pressure mercury lamps
D. High-pressure mercury lamps

Ratio: For UV works, the commonly used lamps are the deuterium lamps and mercury lamps.
- Deuterium discharge lamps provide continuous emission down to 165 nm.
- Low-pressure mercury arc lamps emit a sharp line spectrum with both UV and visible lines.
- Medium and High pressure mercury lamps emit a continuum from UV to the mid-visible region.

For Visible and near-infrared regions


- Incandescent tungsten or tungsten-iodide lamp. **commonly used source of light

4
15. Which of the following is the most commonly used monochromator?
A. Colored glass filters
B. Interference filters
C. Glass Prisms
D. Diffraction Gratings

Ratio:
● Colored Glass Filters - least expensive, usually pass a relatively wide band of radiant energy and have
low transmittance of the selected wavelength. Although not precise, they are simple and inexpensive.
● Interference filters - produce monochromatic light. Transmits multiples of the desired wavelengths. It
can be constructed to pass a very narrow range of wavelengths with good efficiency.
● Glass prism - a narrow beam of light focused on a prism is refracted as it enters the denser glass. The
prism can be rotated, allowing only the desired wavelength to pass through an exit slit.
● Diffraction Gratings - Commonly used monochromators. Consists of many parallel grooves etched onto
a polished surface. Diffraction, the separation of light into component wavelengths, is based on the
principle that wavelengths bend sa they pass a sharp corner.

16. Which of the following DOES NOT describe the principle of atomic absorption spectrophotometry?

A. It measures the concentration of the substance by virtue of detecting the transmission of


electromagnetic radiation by atoms rather than by molecules.
B. It measures the concentration of the substance by detecting the transmission of electromagnetic
radiation by molecules rather than by atoms.
C. It measures the concentration of the substance by detecting the absorption of electromagnetic
radiation by atoms rather than by molecules.
D. It measures the concentration of the substance by detecting the absorption of electromagnetic
radiation by molecules rather than by atoms.

Ratio: AAS is used to measure concentration by detecting the absorption of electromagnetic radiation by
atoms rather than by molecules.

17. Which of the following methods is RECENTLY and COMMONLY used for Na+, K+ or Li+ determination?

A. Flame photometry
B. AAS
C. Ion-selective electrodes
D. A and C

Ratio:
- Flame emission photometer measures light emitted by excited atoms. (Na+, K+ and Li+ are ions that
are capable of emitting specific wavelengths **specific colors**). The intensity of the color and the
specific color indicates the concentration and the type of ion present, respectively.
- Ion Selective Electrode is an analytical technique used to determine the activity of ions in aqueous
solution by measuring the electrical potential. **relatively inexpensive and easy to operate.
- ISE are electrochemical ion sensors that convert the activity of a target ion into an electrical
potential as the measurable signal.

18. If the color of the solution is Red-orange, what will be the absorbed color?
A. Violet
B. Blue-green
C. Red
D. Orange

5
Ratio:

Wavelength Color Absorbed Color Transmitted (Color Seen)

350-430 Violet Yellow

430-475 Blue Orange

475-495 Blue-green Red-Orange

495-505 Green-blue Orange-red

505-555 Green Red

555-576 Yellow-Green Violet-red

575-600 Yellow Violet

600-650 Orange Blue

670-700 Red Green

19. Which of the following is part of the components of the THIN-LAYER CHROMATOGRAPHY?
A. Gas, injection port, column, oven, detector, recorder.
B. Solvent, pump, injection port, column, detector, recorder.
C. Sorbent-coated glass or plastic plate, closed container, solvent.
D. Gas, pump, column, recorder.

Ratio:

20. How to correct the absorbance of the unknown solution with colored reagents?

A. by using water blanks


B. by using reagent blanks
C. by using air blanks
D. by using sample blanks

RATIO: A spectrophotometer can be set to 100%T with the reagent blank instead of water, the absorbance of
reagents is automatically subtracted from each unknown reading. The reagent blank does not correct for
absorbance caused by interfering chromogens in the sample such as bilirubin, hemolysis or turbidity.

21. Which of the following statements DOES NOT MATCH?

A. Ion-selective electrodes - pH measurement


B. Electrophoresis - Sodium measurement
C. Spectrophotometry - Glucose measurement
D. Flame Photometry - Sodium measurement

Ratio:
- ISE - potential difference between 2 electrodes directly related to concentration of analyte.
- USE: pH, PCO2, PO2, Na+, K+, Ca2+, Li+, Cl-

- Electrophoresis - separation of charged particles in electrical field. Anions move to positively charged
pole (anode); cations to negatively charged pole (cathode). The greater the charge, the faster the
migration.
- USE: Serum protein, hemoglobin electrophoresis
6
22. A linearity study was performed on a visible spectrophotometer at 650 nm and the following absorbance
readings were obtained:

Concentration of Standard Absorbance

10.0 mg/dL 0.20

20.0 mg/dL 0.41

30.0 mg/dL 0.62

40.0 mg/dL 0.79

50.0 mg/dL 0.92

The study was repeated using freshly prepared standards and reagents, but results were identical to those
shown. What is the most likely cause of these results?
A. wrong wavelength used
B. Insufficient chromophore concentration
C. Matrix interference
D. Stray Light

Ratio: Stray light is the most common cause of loss of linearity at high-analyte concentrations. Light transmitted
through the cuvette is lowest when absorption is highest. Therefore, stray light is a greater percentage of the
detector response when sample concentration is high. Stray light is usually most significant when
measurements are made at the extremes of the visible spectrum because lamp output and detector response
are low.

23. In mass spectroscopy, the term base peak typically refers to:
A. The peak with the lowest mass
B. The peak with the most abundance
C. A natural isotope of the molecular ion
D. The first peak to reach the mass detector.

Ratio: THe base peak is typically the “molecular ion” or parent ion, meaning that it is the initial fragment made
by releasing an electron.

Laboratory Mathematics (Questions 24-34)

24. A new tumor marker for ovarian cancer is evaluated for sensitivity by testing serum samples from patients
who have been diagnosed by staging biopsy as having malignant or benign lesions. The following results were
obtained:

Number of malignant patients who are positive for CA-125 =21 out of 24
Number of benign patients who are negative for CA-125 = 61 out of 62

What is the sensitivity of the new CA-125 test?


A. 98.4%
B. 95.3%
C. 87.5%
D. 85.0%

Ratio: Sensitivity is defined as the percentage of persons with the disease who have a positive test result. It is
calcualted as true-positives divided by the sum of TP and FN

% Sensitivity = TP X 100
TP + FN
Sensitivity = (21 X 100) / (21 +3) = 87.5%

25. What is the molarity of a solution comprising 2 moles of sucrose (table sugar) in 4 liters?
A. 1 M
B. 0.50 M
C. 0.25 mole
D. 1 mole

Ratio:
The ratio is 2 mol per 4 L: 2 mol 4L = 0.50 mol/L = 0.50 M

7
26. Calculate % (w/w) of 9.2 g CaCl2 dissolved in 800 g of water.
A. 1.1 % (w/w)
B. 0.1 % (w/w)
C. 1.0 % (w/w)
D. 10 % (w/w)

Ratio: (9.2 g/ 809 g) x 100% = 1.1% (w/w)

27. Calculate the normality of 14.9 g KCl in 200 mL of solution.


A. 10 N
B. 1.1 N
C. 0.5 N
D. 1 N

Ratio: 14.9 g (mol/74.6 g) (1/0.200 L) = 1.0 N

28. One of the automated instruments in your laboratory requires the preparation of a special cleaning
solution, made by mixing 30 mL of concentrate with enough water to bring the final volume to 250 mL. How
much concentrate do you use to make 1 L of the cleaning solution?
A. 75 ml
B. 120 ml
C. 12 ml
D. 100 ml

Ratio: First, convert one of the units into the other: 1 L = 1000 mL.
Next, set up an equation of ratios:
30 mL concentrate: 250 mL solution = ____ : 1000 mL solution
(30 mL concentrate)(1000 mL solution) divided by 250 mL = 120 mL concentrate

29. If isopropyl alcohol is added to 50 mL of water until the volume of the solution is 150 mL, what is the %
(v/v)?
A. 33% (v/v)
B. 3.3% (v/v)
C. 0.3% (v/v)
D. 30% (v/v)

Ratio: The concentration is the volume of alcohol divided by the total solution volume:
(50 mL/150 mL) x 100% = 33% (v/v)

30. Calculate the molarity of 4 g KOH in 1 L of solution.


A. 0.071 M
B. 0.051 M
C. 0.7 M
D. 0.5 M

Ratio:( 4.0 g) (mol / 56.1 g)(1 / 1 L) = 0.071 M

31. Consider the serial dilution of a sample at 50,000 ng/dL. If the tube dilution is 1:10, what is the
concentration in the fourth tube?
A. 50 ng/dl
B. 0.5 ng/dl
C. 500 ng/dl
D. 5.0 ng/dl

Ratio: Dsample = (Dtube) exp N = (1/10) exp 4 = 1/10,000. Thus, the concentration in tube #4 is
5 ng/dL

32. If the tube dilution is 1:3, how many tubes are required in a series to achieve a sample dilution of 1:243?
A. 3
B. 5
C. 10
D. 7

8
Ratio:

33. Convert 2.0 mEq/L magnesium (atomic weight = 24.3) to milligrams per deciliter.
A. 0.8 mg/dL
B. 1.2 mg/dL
C. 2.4 mg/dL
D. 4.9 mg/Dl

Ratio: To convert from milliequivalent per liter to milligrams per deciliter, first calculate the milliequivalent
weight (equivalent weight expressed in milligrams), which is the atomic mass divided by the valence. Because
magnesium is divalent, each mole has the charge equivalent of 2 mol of hydrogen. Then, multiply the
milliequivalent per liter by the milliequivalent weight to convert to milligrams per liter. Next, divide by 10 to
convert milligrams per liter to milligrams per deciliter. Milliequivalent weight Mg = 24.3 ÷ 2 = 12.15 mg/mEq 2.0
mEq/L × 12.15 mg/mEq = 24.3 mg/L 24.3 mg/L × 1.0 L/10.0 dL = 2.4 mg/dL

34. How many milliliters of a 2,000.0 mg/dL glucose stock solution are needed to prepare 100.0 mL of a 150.0
mg/dL glucose working standard?
A. 1.5 mL
B. 7.5 mL
C. 15.0 mL
D. 25.0 mL

Ratio: To calculate the volume of stock solution needed, divide the concentration of working standard by the
concentration of stock standard, then multiply by the volume of working standard that is needed. C1 × V1 = C2
× V2, where C1 = concentration of stock standard V1 = volume of stock standard C2 = concentration of
working standard V2 = volume of working standard 2000.0 mg/dL × V1 = 150.0 mg/dL × 100.0 mL V1 = (150.0
÷ 2000.0) × 100.0 mL V1 = 7.5 mL

35. Which of the following measures of centers is NOT COMMONLY used in the descriptive statistics related to
internal quality control program?
A. Mean
B. Median
C. Mode
D. Average

Ratio: Mean, Median, Mode are three commonly used measures of center of a data set. Mean is aka as
average. Median is the middle point and is often used with skewed data. While mode is rarely used, but is
more often used to describe data set that seem to have two centers (i.e. bimodal).

36. Which of the following statements describes accuracy?


A. It is the ability of a method to measure the smallest concentration of the analyte of interest.
B. It is the ability by a method to be easily repeated.
C. It is the ability of a method to produce a near or close assayed value to the true or target value.
D. It is the ability of a method to give repeated results on the same sample that agree with one another.

37. Which of the following statements describes Sensitivity?


A. It is the ability of a method to measure the smallest concentration of the analyte of interest.
B. It is the ability by a method to be easily repeated.
C. It is the ability of a method to produce a near or close assayed value to the true or target value.
D. It is the ability of a method to give repeated results on the same sample that agree with one another.

Ratio (36 and 37):


- In the concept of quality control the following are defined as
- Sensitivity: the capacity of the method(test) to measure the smallest concentration of the analyte
of interest.
- Specificity: the capacity of the method(test) to detect only the analyte of interest.
- Accuracy: the nearness or closeness of the assayed value to the true or target value.
- Precision: the ability of an analytical method to give repeated results on the same sample that
agree with one another.

9
38. Compute for the diagnostic specificity of the HbSAg test kit given with the following data:

Total number of tests = 300


Number of non-reactive results with no hepatitis infection = 170
Number of individuals without the infection = 210
Number of reactive results with hepatitis infection = 90
Number of individuals with the infection = 87

A. 80.95%
B. 56.67%
C. 103.44%
D. 30%

Ratio:
%Sensitivity = 100 X (number of diseased individuals with positive results/total number of diseased individuals tested)
%Specificity = 100 X (number of diseased individuals with negative results/total number of individuals without disease)

39. Which of the following IS NOT being monitored by quality control?


A. Monitor and maintain the stability of the laboratory equipment
B. Monitor and assess the quality of reagents
C. Avoid and monitor technical errors.
D. Monitor the scheduling of staffs and employees

Ratio: QC is a MUST regular practice of the clinical laboratory in order to MAKE SURE that the tests done in
the clinical laboratory is ACCURATE AND PRECISE (thereby making it reliable). It detects changes in
performance between the present operations and the “stable” operations. Part of the objectives of the quality
control are:
- To check the stability of the machine
- To check the quality of reagents
- To check technical errors

40. Which of the following DOES NOT DESCRIBE random error?


A. It is an error that influences observations consistently in constant differences (one direction).
B. It refers to the difference between the target value and the assay value.
C. It is the basis for varying differences between repeated measurements.
D. It is most often due to instrument, operator and environment conditions (variations in techniques)

Ratio: Random Error vs. Systematic Error

Random Error – affect precision; unable to predict because they have no known pattern, and may aternate
between a positive or negative direction.

Systematic error – are predictable and cause a constant difference in results that are consistently positive or
negative or stay the same. Such errors may be due to incorrect calibration, deteriorated reagents, instrument
malfunction, etc.

41. What is the other name of Gaussian Curve?


A. Cumulative Sum Graph
B. Twin Plot
C. Bell-shaped Curved
D. Levey-Jennings Graph

42. In general concepts of quality control, what is the acceptable range of confidence limit of the laboratory
data?
A. +/- 1SD
B. +/- 2SD
C. +/- 3SD
D. none of the above

Ratio: 95% confidence limit is +/- 2SD according to the Gaussian Curve (and this is the acceptable range in the
QC if based of Westgard Multirule or Levey Jennings Chart.

10
43. If deterioration of reagent will yield a “trend” pattern in the chart. What will be the visible pattern if improper
calibration of the instrument is made?

A. Trend
B. Shift
C. Outliers
D. No visible pattern

Ratio: Common classification of errors observed in LJ chart are:


- Trend - formed by control values that either increase or decrease for six consecutive days, and usually
caused by systematic errors; gradual change in the mean that is reflected either a decrease or increase
of consecutive control values (generally the number of consecutive observations signifying a trend is six
or more)

- Shift - formed by control values that distribute themselves on one side or either side of the mean (shift
in the reference range is due to transient instrument differences); sudden change in the mean that is
reflected as consecutive control values above or below the mean.

44. Which of the following assays has the poorest precision?

Analyte Mean (mmol/L) Standard deviation


A Ca 2.5 0.3
B K 4.0 0.4
C Na 140 4.0
D Cl 100 2.5

Ratio: Although Calcium has the lowest SD, it represents the assay with poorest precision. Relative precision
between different analytes or different levels of the same analyte must be evaluated by the coefficient of
variation (CV) because standard deviation is dependent upon the mean. CV = s X 100/mean. This normalizes
standard deviation to a mean of 100. The CV for calcium in the example is 12.0%.

45. Which of the following plot is best for detecting all types of QC errors?
A. Youden-plot
B. Levy-Jennings
C. Linear Regression
D. Cusum

Ratio: LJ chart or plot is a graph of all QC results with concentrations plotted on the y-axis and run number on
the x-axis. The mean is at the center of the y axis, and concentrations corresponding to -2 and +2s are
highlighted. Results are evaluated for multi rule violations across both levels and runs. Corrective actions for
shifts and trends can be taken before QC rules are broken.

46. From the data shown below, determine the mode:


130 135 134 136 135
130 134 136 131 132
137 140 140 136

A. 137 B. 136 C. 134 D. 135

Ratio: Mode is defined as the most frequently occurring value in a set of values;
- 136 occurred 3 times and it is the value that occurred most in the series.

46. Given the following sodium control values (mEq/L): 138, 145, 139, 142, 140, 143
Determine the standard deviation.

A. 0.71 B. 1.88 C. 2.65 D. 3.5

Ratio: Please do the computation for SD (standard deviation)

11
47. When a certain set of control values was arranged from the highest value to the lowest, the middlemost
value would be known as:
A. Mean B.Median C.Mode, D. StandardDeviation

Ratio: Median is the middlemost value in an array.


- Mean is the sum of all the values divided by the number of values.
- Mode is the most frequently occurring value in the series of values.

48. What is the minimum requirement for performing QC for a glucose assay?

A. One level assayed every 8 hours


B. Two levels assayed within 8 hours
C. Two levels assayed within 24 hours
D. Three levels assayed within 24 hours

Ratio: The minimum requirement for frequency of quality control for a general chemistry analyte (based upon
the Clinical Laboratory Improvement Act, 1988) is two levels of control assayed every 24 hours. Some
laboratories prefer to assay two control levels every 8 hours to increase the opportunity for error detection.

Note: two controls every 8 hours are required for blood gases, automated hematology, and point-of-care
glucose testing to comply. Analytes that display different CVs at the low, normal, and high ranges require 3
levels of control in 24 hours. These include blood gases, therapeutic drugs, and hormones.

49. When the magnitude of error increases with increasing sample concentration, it is called as _____.
A. constant error
B. Proportional error
C. Random error
D. Bias

Ratio: Proportional error (slop or percent error) results in greater absolute error (deviation from the target
value) at higher sample concentration. Constant error refers to a difference between the target value and the
result, which is independent of sample concentration.

50. All of the lab’s policies, processes, procedures and resources needed to achieve quality testing is termed
as ___.

A. Quality assurance
B. Analytical Quality Assurance
C. Post Analytical Quality Assurance
D. Quality System

Ratio:
- Quality assurance is explained as the process by which lab ensures quality results by closely
monitoring preanalytical, analytical and postanalytical stages of testing.
- Preanalytical QA is defined as everything that precedes performance e.g. test ordering, patient
preparation, patient ID, specimen collection, specimen transport, specimen processing.
- Analytical QA is everything related to assay e.g. test analysis, QC, reagents, calibration, preventive
maintenance
- Post Analytical QA is everything that comes after test analysis (e.g. verification of calculation and
reference ranges, review of results, notification of critical values, results reporting, test interpretation by
physician, follow-up patient care.
- Quality system is all of the lab’s policies, processes, procedures & resources needed to achieve quality
testing.

51. It is an integral part of the analytical phases of quality assurance where process of monitoring results from
control samples to verify accuracy of patients results.
A. External Quality Control
B. Internal monitoring systems
C. Quality Assurance
D. Quality Control

12
52. Describe the following chart:

A. It represents the normal distribution of the data set for QC.


B. A control result outside established limits.
C. There are consecutive control values on the same side of the mean.
D. There are values decreasing for 6 consecutive runs.

Ratio for 52 and 53

53. Describe the following chart

A. It represents the normal distribution of the data set for QC.


B. A control result outside established limits.
C. There are consecutive control values on the same side of the mean.
D. There are values decreasing for 6 consecutive runs.

13
54. In a chart, you see 10 consecutive controls on same side of mean, which of the following statements best
describes the situation?
A. It is described as 1-3s and there is a random error.
B. It is described as 2-2s and there is systematic error.
C. It is described as 10-x and there is systematic error.
D. It is described as 4-1s and there is systematic error.

Ratio:

55. Precision is also known as

A. Accuracy
B. Reliability
C. Reproducibility
D. Reportable

56. Which of the following is a term describing “positive results in patient who has the disease”

A. True positive
B. False positive
C. True negative
D. False negative

57. Delta checks is

A. study to verify accuracy of new method


B. test results that indicate a potentially life-threatening situation.
C. comparison of patient data with previous results.
D. procedure specified by manufacturer to evaluate critical operating conditions

58. It promotes absorption of sodium in the distal convoluted tubule.


a. Aldosterone
b. Renin
c. Antidiuretic Hormone
d. All of the choices is correct

Aldosterone
- sodium retention hormone
- promotes absorption of sodium in DCT
- promotes sodium retention and potassium excretion
ADH
- Hormone produced by the hypothalamus to regulate water reabsorption in the collecting duct
RENIN
-Proteolytic enzyme produced by the kidneys that reacts angiotensinogen to produce angiotensin to
increase blood pressure

59. It is a reduction in serum sodium concentration caused by a systematic error in measurement.


a. Hypernatremia
b. Hyponatremia
c. Pseudohyponatremia
d. B and C

Pseudonatremia- False decrease of sodium


- Cause is in vitro hemolysis - a well known cause of PSEUDOHYPERKALEMIA
- Marked hemolysis may cause decreased of sodium levels due to dilutional effect
- Hgb released from the red cells cause additional reduction in serum Na+
Hypernatremia
- Caused by loss of water, gain of sodium, or both
Hyponatremia

14
- The most electrolyte disorder, it is defined as reduced plasma sodium concentration

60. All are true about chloride, except:


1. It is the major intracellular anion
2. It is excreted in the urine and sweat
3. It promotes the maintenance of water balance and osmotic pressure in conjunction with sodium
4. Maintains electrical neutrality

a. 1
b. 2 and 4
c. 3
d. 3 and 4

RATIO: Chloride is the MAJOR EXTRACELLULAR ANION- chief counter ion of sodium in ECF. In most
processes, Cl- shifts secondarily to a movement of Na+ or HCO3-. It maintains electroneutrality through
chloride shift. In this process, CO2 generated by cellular metabolism within the tissue diffuses out into the
plasma and red cells. In the red cell, CO2 forms carbonic acid (H2CO3) which splits into H= and HCO3.
Deoxyhemogobin buffers H+, whereas the HCO3 diffuses out into the plasma and Cl- diffuses into the red cell
to maintain the electric balance of the cell.

61. All are the factors affecting magnesium level in the blood except:
1. Parathyroid hormone
2. Thyroxine
3. Aldosterone
4. Vasopressin

a. 1 and 2
b. 2
c. 1, 3 and 4
d. 4

Parathyroid hormone
- it increases renal reabsorption of Mg and it increases intestinal absorption of Mg
Thyroxine
- It increases renal excretion of Mg
Aldosterone
- It increases renal excretion of Mg
Vasopressin
- Reabsorption of water

62. Statement 1: Potassium deficiency also causes loss of sodium.


Statement 2: Serum potassium levels are low, the tubules will conserve sodium and excrete the potassium
in exchange for the loss of the monovalent cation.
a. 1st statement is true, 2nd statement is false
b. 1st statement is false, 2nd statement is true
c. Both statements are true
d. Both statements are false

Potassium deficiency also causes loss of sodium because of inverse relationship of the two ions in the renal
tubules - when serum potassium levels are low, the tubules will conserve potassium and excrete the sodium in
exchange for the monovalent cation.

63. The patient was suffering from prolonged diarrhea. What is the possible result that can be expected?

a. Hypernatremia
b. Hyponatremia
c. Hypokalemia
d. hyperkalemia

Hypernatremia:
- Excess water loss: DI, Renal tubular disorder, prolonged diarrhea, profusse sweating, severe burns,
vomiting, fever, hyperventilation
- Decreased water intake
- Increased intake or retention

64. There several methodologies available for measuring Cl-, except:


1. Schales and schales
2. Spectrophotometric methods
3. Ion selective electrode
4. Atomic absorption Spectrophotometry

a. 1 and 4

15
b. 1 and 2
c. 3
d. 4

Atomic absorption Spectrophotometry - reference method used for Calcium


METHODS USED FOR CHLORIDE
- MERCURIMETRIC TITRATION (Schales and schales)
- Spectrophotometric methods
- Ion selective electrode
- Coulometric amperometric titration - cotlove chloridometer (sweat chloride test)

65. Statement 1: Heparinized plasma is preferred over serum in potassium determination.


Statement 2: platelets contain K+ that is released into serum on clot formation.
a. 1st statement is true, 2nd statement is false
b. 1st statement is false, 2nd statement is true
c. Both statements are true
d. Both statements are false

Potassium is released during clotting. When the platelet count is increased (thrombocytosis), serum K may be
further elevated. That is why, heparinized plasma is preferred, esp. When there is thrombocytosis.

Note: further read on the specimen consideration for K in Bishop.

66. All of the methods are used for Potassium determination, except:
1. Emission flame Photometry
2. Ion selective electrode
3. Colorimetry
4. Glass aluminum silicate

a. 1 and 2
b. 2
c. 3 and 4
d. 4

Ion selective Electrode (Glass Aluminum Silicate) -most commonly used method dor Na determination

METHODS FOR POTASSIUM DETERMINATION


- Emission flame Photometry
- Ion selective electrode(Valincomycin gel as selective membrane)
- Colorimetry(Lockhead and Purcell)
- Atomic absorption Spectrophotometry

67 . Intracellular cation second in abundance to potassium.


a. Magnesium
b. Sodium
c. K
d. Calcium

Magnesium is an intracellular cation and it is the fourth most abundant cation in the body. Majority is stored in
the bones (53%), in the muscles (46%) and soft tissue, (1%) in seum and RBC.

68. It is the most commonly used method to measure serum inorganic phosphate.
a. Fiske Subbarow method
b. Ammonium molybdate method
c. Ion selective electrode
d. A and B

Fiske Subbarow method (Ammonium molybdate method)


- the most commonly used method to measure serum inorganic phosphate
- end product: ammonium-molybdate complex(unstable)
- the unreduced complex at 340 nm is the most accurate measurement of inorganic phosphorus in
serum.
- the pH must be maintained in the acid range because high range(alkaline can result in reduction of
the complex)

69. All are the factors affecting Phosphate concentration, except:


1. Calcitonin
2. Growth hormone
3. Albumin
4. Aldosterone

a. 1 and 2

16
b. 3
c. 3 and 4
d. 2

FACTORS AFFECTS PHOSPHATE CONCENTRATION


- PTH - decreases phosphate by renal excretion
- Calcitonin - inhibits bone resorption
- GH - increase phosphate renal reabsorption

70. It is the most common cause of Hypophosphatemia.


a. Alcohol abuse
b. Tubular failure
c. Renal failure
d. Hypervitaminosis D

All of the following causes hypophosphatemia:


- Alcohol abuse - most common cause
- Myxedema
- Primary hyperparathyroidism
- Avitaminosis D
All of the following causes hyperphosphatemia:
- Hypoparathyroidism
- Renal failure (tubular failure)
- Lymphoblastic leukemia
- Hypervitaminosis D

71. All of the following is not true about Potassium, except:


1. K+ is mostly reabsorbed by active and passive mechanisms in the Proximal convoluted tubule
2. K+ is reabsorbed together with aminoacids and glucose in ascending loops of Henle.
3. It is the major intracellular cation.
4. K+ filtered and reabsorbed in the collecting duct
a. 1 and 3
b. 2 and 3
c. 3 and 4
d. 1 and 4

POTASSIUM: is the major intracellular cation


- It is filtered at the glomerulus and is mostly (70%-80%) reabsorbed by active and passive mechanisms
in PCT
- In the ascending loop of henle, K+ is absorbed together with Na+ and Cl- by sodiumm potassium
chloride cotransporter

72. It is a common metallic element important for the synthesis of hemoglobin.


a. Iron
b. Ferritin
c. Total iron binding capacity
d. A and B

Iron - It is a common metallic element important for the synthesis of hemoglobin.


Ferritin - stored iron
TIBC - the amount of iron that could be bound by saturating transferrin and other minor-iron binding proteins
present in the serum or plasma sample

73. Unsaturated Iron Binding Capacity (UIBC) formula:


a. TIBC - ferritin
b. UIBC - serum iron
c. TIBC - serum iron
d. Ferritin - plasma iron

UIBC - it is a measure of the reserve iron binding capacity of trasnferrin


-formula : TIBC -serum iron

74. All of the following are correct, except:


a. Calmagite method = (+) reddish-violet complex
b. Formazen Dye method = (+) colored complex
c. Dye lake method = clayton yellow
d. Clark collip precipitation = yellow

Colorimetric methods for Magnesium Determination:


- Magnesium thymol blue method = colored comples
- Calmagite method = (+) reddish-violet complex
- Formazen Dye method = (+) colored complex

17
- Dye lake method - Titan Yellow dye = clayton yellow or thiazole yellow
- AAS -reference method
CLARK COLLIP PRECIPITATION - END PRODUCT: OXALIC ACID (PURPLE)
- Method used for calcium determination.

75. In the urease enzymatic method, urea is converted to?


A. Ammonia
B. Nitrogen
C. Carbon Dioxide
D. Amino acids

Ratio: The enzyme urease hydrolyses urea in the sample, and the ammonium ion produced in the reaction is
quantified.

76. Uric Acid levels is used to diagnose and monitor:


A. Gouty Arthritis
B. Tumor Lysis Syndrome
C. End-stage renal disease
D. A and B
E. All of the above

Ratio:
- Tumor lysis syndrome (TLS) is an oncologic emergency that is caused by massive tumor cell lysis with
the release of large amounts of potassium, phosphate, and nucleic acids into the systemic circulation.
Catabolism of the nucleic acids to uric acid leads to hyperuricemia.
- Gout is caused by a condition known as hyperuricemia. When there is too much uric acid in the body,
uric acid crystals (monosodium urate) can build up in joints, fluids, and tissues within the body.
Repeated bouts of gout can lead to gouty arthritis, a worsening form of arthritis
- End-stage renal disease- Urea levels

77. Creatinine synthesis in the liver requires these following amino acids except?
A. Glycine
B. Alanine
C. Methionine
D. Arginine

Ratio:
- Creatine is synthesized primarily in the liver from arginine, glycine, and methionine. It is then
transported to other tissues, such as muscle, where it is converted to creatine phosphate (a high
energy source). Creatine phosphate loses phosphoric acid and creatine loses water to form creatinine,
which diffuses into the plasma and excreted in the urine.
- Alanine- supports the function of the liver. It is a source of energy for muscles and the central nervous
system. It strengthens the immune system and helps the body use sugars.

78. A medical technologist obtains a BUN result of 61 mg/dl and a serum creatinine result of 1.1 mg/dL on a
patient. These results indicate:
A. Lesch-Nyhan Syndrome
B. Glomerulonephritis
C. Congestive heart failure
D. Nephrolithiasis

Normal values:
BUN= 6-20 mg/dL Creatinine= 0.9-1.3mg/dL (Male); 0.6-1.1. mg/dL (Female)

18
Result shows high BUN and normal serum creatinine values which indicates Congestive Heart Failure
(Prerenal Azotemia); Glomerulonephritis- Renal Azotemia (High BUN and Crea); Nephrolithiasis- post-renal
azotemia (High BUN and Crea); Lesch-Nyhan Syndrome- confuser, causes hyperuricemia.

79. A high BUN:Creatinine ratio with a significantly elevated creatinine is usually seen in:
A. Nephrolithiasis
B. liver disease
C. Lesch-Nyhan Syndrome
D. Congestive heart failure
Ratio: Nephrolithiasis (Postrenal azotemia)- high BUN:creatinine ratio (BCR) with a significantly elevated
creatinine levels; Liver disease- causes decreased plasma urea concentration; CHF (Prerenal azotemia)- high
BUN:creatinine ratio (BCR) with normal creatinine; Lesch-Nyhan Syndrome- confuser, causes hyperuricemia.

80. What is the urea concentration if the BUN level is 6 mg/dL?


A. 18.3 mg/dL
B. 12.8 mg/dL
C. 10.3 mg/dL
D. 9.3 mg/dL

Ratio: BUN concentration can be converted to urea concentration by multiplying by 2.14.

81. The uricase method for uric acid assay depends on:
A. ultraviolet absorption at 293 nm before and after treatment with uricase
B. collection of evolved gas after uricase treatment
C. titration before and after uricase treatment
D. formation of a colored solution

Principle of Uricase Method: The enzyme uricase catalyzes the oxidation of uric acid to form allantoin. Uric
Acid has a maximum peak of absorption at 293mm. The decrease in the absorbance is proportional to the
concentration of uric acid present in the sample.

82. In the diacetyl monoxime method, what does diacetyl react with to form a yellow product?
A. creatinine
B. uric acid
C. urea
D. ammonia

RATIO: Diacetyl Monoxime Method Principle: Diacetyl monoxime breaks down into diacetyl during the reaction
in the presence of heat (provided from a boiling water bath). Diacetyl and urea then condense in the same
medium under the presence of a strong acid to form the yellow-colored diazine product and water.

19
83. When mixed with phosphotungstic acid, what compound causes the reduction of the former to a tungsten
blue complex?
A. creatinine
B. uric acid
C. urea
D. ammonia

RATIO: Caraway Method: based on the oxidation of the uric acid in a protein-free filtrate, with subsequent
reduction of phosphotungstic acid to tungsten blue. The method lacks specificity.

84. Which of the following statements regarding creatinine methods is incorrect?


A. Fuller’s earth reagent improves the specificity of the Jaffe reaction.
B. Reaction of creatinine with alkaline picrate without adsorbent is highly specific.
C. The kinetic Jaffe method prevents interference by noncreatinine chromogens.
D. Isotope dilution mass spectrometry is used as a reference method.

Ratio:
A- More accurate results are obtained when creatinine in a protein-free filtrate is adsorbed onto Fuller’s earth
(aluminum magnesium silicate) or Lloyd’s reagent (sodium aluminum silicate), then eluted and reacted with
alkaline picrate. Adsorbent improves specificity.
C- Kinetic Jaffe Method eliminates some of the nonspecific reactants, but it is subject to interference by a-keto
acids and cephalosporins. Despite these problems, it is still used routinely because it is inexpensive, rapid, and
easy to perform.
D- IDMS is the reference method for Creatinine and Urea.

85. A 12-year-old patient had 10 episodes of voluminous watery diarrhea and no urine outputs in the last 24
hours and was brought to the ER with the following initial laboratory result:
Na= 155 mmol/L BUN= 66mg/dL ABG= metabolic acidosis
Potassium= 3 mmol/L Creatinine= 1.1 mg/dL
The patient above has:
A. Severe liver disease
B. Prerenal Azotemia
C. Postrenal Azotemia
D. Renal Azotemia

Normal values:
BUN= 6-20 mg/dL Creatinine= 0.9-1.3mg/dL (Male); 0.6-1.1. mg/dL (Female)
Result shows: High BUN:Crea ratio with normal Creatinine. The patient had 10 episodes of voluminous watery
diarrhea which indicates Dehydration. Dehydration causes prerenal Azotemia; Severe liver disease- Low
BUN:Crea ratio; Postrenal Azotemia and Renal Azotemia- High BUN:Crea ratio with an elevated creatinine
levels

86. When measuring ammonia blood level, the following might cause a false increase level of this analyte
except?
A. Immediately after phlebotomy, the blood sample was stored at room temperature.
B. The patient had two cigarettes 15 minutes prior to blood extraction.
C. The patient was fasting for hours prior to blood collection.
D. A and B
E. All of the choices will falsely increase the blood ammonia levels.

Specimen Requirements for Ammonia:


Venous blood should be obtained without trauma and placed on ice immediately.
Heparin and EDTA are suitable anticoagulants.
Cigarette smoking by the patient is a significant source of ammonia contamination
Hemolysis should be avoided.
Patients should be fasting 12 to 14 hours to avoid lipemia, which interferes with the test.

20
87. Ammonia concentrations are usually measured to evaluate:
A. Glomerulonephritis
B. Acid-base status
C. Congestive heart failure
D. Hepatic encephalopathy

Ratio: Ammonia blood test is most commonly used to diagnose and monitor hepatic encephalopathy, a severe
liver disease.

88. Which of the following is/are product/s of sucrose hydrolysis?


A. 2 glucose molecules
B. galactose and glucose
C. maltose and glucose
D. fructose and glucose

RATIO: glucose + glucose = maltose; galactose + glucose = lactose; matlose and glucose = just a confuser;
fructose + glucose = sucrose

89. Which of the following blood glucose levels would you expect to result in glucose in the urine?
A. 75 mg/dL
B. 150 mg/dL
C. 180 mg/dL
D. 230 mg.dL

RATIO: The renal threshold for glucose is 160-180 mg/dL. Any plasma glucose above the renal threshold will
result to the excretion of glucose in the urine (glucosuria)

90. A known Type II DM patient was given 2 months of treatment. He went to his doctor for follow-up after the
2-month treatment. Which of the following tests may be performed to asses the average glucose level in this
case?
A. Fasting blood sugar
B. Glycated hemoglobin
C. Two-hour oral glucose tolerance test
D. Random blood sugar

RATIO: Fructosamine test and glycated hemoglobin are tests for monitoring glucose control over a period of
time. Fructosamine is a glucose monitoring for 2-3 weeks period while glycated hemoglobin is for 2-3 months
monitoring. FBS is primarily a screening test and 2-hr OGTT as a confirmatory test.

Glycated hemoglobin is the non-enzymatic, irreversible attachment of glucose to hemoglobin A1.


Specimen collection: non-fasting blood in EDTA
Reference range: 4-6%; <7% is effective treatment

Fructosamine: attachment of glucose to protein, primarily albumin


Specimen: serum or plasma
REference range: 205-285 umol/L

91. The doctor is monitoring a Type I DM patient for possible complications. All of the following tests are helpful
in this case, EXCEPT:
A. Serum creatinine
B. Serum urea nitrogen
C. Serum bilirubin
D. Urinary albumin

RATIO: The possible complications of diabetest mellitus are: nephropathy, retinopathy, neuropathy, and
cardiovascular diseases. Bilirubin is not a helpful test in this case. Serum creatinine, urea nitrogen, and urinary
albumin are kidney function tests.

92. In most laboratory techniques of measuring glucose, serum or plasma is preferred over whole blood
because
1. Glucose levels are more stable when serum or plasma is separated
2. Using serum or plasma increases the specificity of the test for glucose
3. Using whole blood would require immediate mixing before sampling, and, thus inconvenient

A. 1, 2, 3
B. 1,2 only
C. 1 only
D. 2 only

21
RATIO: Glucose determination is generally performed on serum or plasma. Serum or plasma is convenient to
use in most automated systems because serum does not require mixing before sampling. Glucose is more
stable in separated serum or plasma because this minimizes glycolysis. With using serum or plasma,
specificity for glucose is increased because red blood cells contribute to various interferences during testing.

93. Mr. X submitted himself for routine fasting blood sugar. His result was 115 mg/dL. Which among the
following category is he classified?
A. Normal for his age
B. Type I DM
C. Type II DM
D. Impaired fasting glucose

RATIO: According to the American Diabetes Association, the diagnosis of diabetes mellitus is as follows:
Fasting blood sugar of ≥126 mg/dL
Random blood sugar of ≥200 mg/dL
2-hr Oral glucose tolerance test of ≥200mg/dL

Impaired fasting glucose is defined as glucose level between 100-125 mg/dL.

94. A woman in her 20th week of gestation was subjected to GDM screening. An initial screening test was
performed with 50-g glucose load. After 1 hour, her result was 115 mg/dL. So the doctor proceeded to an oral
glucose tolerance test with a 75g glucose load. Evaluate the case.
1. GDM screening must be performed at 24-28 weeks gestation
2. OGTT must be performed only when the challenge test is ≥140 mg/dL
3. The glucose load for GDM is 100 g.

A. 1,2,3
B. 1,2 only
C. 1,3 only
D. 3 only

RATIO: If a woman is at high risk for GDM, there should be an initial screening early in the pregnancy. If she
is not found to have GDM during the initial screening, the woman should be retested at 24-28 weeks of
gestation. For women of average risk, testing should be performed at 24-28 weels pf gestatopm/

DIAGNOSTIC FOR GDM:


FBS = ≥126 mg/dL
Random blood sugar of ≥200 mg/dL

If unequivocal hyperglycemia is not apparent, retesting must be performed on a subsequent day.

TWO STEP APPROACH:


1. Initial screening (Glucose challenge test) with 50g oral glucose load (time of day or time of last meal not
relevant). Serum is measured at 1hr
**if the test value exceeds ≥140 mg/dL, an OGTT is performed.

2. OGTT for GDM - with 100g glucose load


Criteria:
FBS = >95 mg/dL
1st hr OGTT = >180 mg/dL
2nd hr OGTT = >155 mg/dL
3rd hr OGTT = >140 mg/dL

95. The patient’s plasma glucose level is 98 mg/dL. What is the expected CSF glucose concentration?
A. 60 mg/dL
B. 66 mg/dL
C. 49 mg/dL
D. 55 mg/dL

RATIO: The reference interval for glucose in CSF is 60% of the plasma glucose. Reference interval for CSF
glucose is 40-70 mg/dL. Low CSF glucose is associated with: bacterial meningitis and tuberculous meningitis,
whereas viral disease generally presents with a normal level of CSF glucose.

22
96. All of the following are reagents of the serum glucose method, EXCEPT:
1. NAD+
2. Reduced chromogen
3. Glucose oxidase
4. Peroxidase

A. 1 only
B. 1, 2 only
C. 2, 3 only
D. 2, 4 only

RATIO: Enzymatic measurement of glucose are more specific for glucose. Enzymes act on glucose but not on
other sugars, and not on othr reducing substances: GLUCOSE OXIDASE and HEXOKINASE METHOD.

In the glucose oxidase meathed, it measures beta-D-glucose only (beta-D glucose is 65%; alpha-D-glucose is
35%)

Hexokinase method: most specific glucose method; reference method. It measures both alpha and beta-D-
glucose. During measurement, the enzyme mutarotase converts alpha-glucose to beta glucose.

97. You received a CSF specimen for glucose analysis at 7:00 pm. The sample was cloudy and contains
blood. Which of the following is correct?
A. Upon receipt, freeze the sample and assay the next day..
B. Glucose measurement cannot be performed on the sample.
C. Upon receipt, centrifuge the sample and assay immediately.
D. Upon receipt, refrigerate the sample and assay the next day..

RATIO: CSF samples should be assayed as soon as possible upon receipt of the specimen. A cloudy sample
may be due to bacteria. Bacteria and red blood cells may consume glucose in vitro. Delays in testing could
significantly result to false low readings. It is best to centrifuge the specimen to remove any cellular materials.
Assay the sample immediately.

98. Which of the following is/are TRUE about Type I DM?


1. Insulin-dependent diabetes mellitus
2. Ketosis-prone diabetes
3. Receptor-deficient diabetes mellitus
4. Non-insulin dependent diabetes mellitus

A. 1,2 only
B. 3,4 only
C. 1,3 only
D. 2,4 only

23
RATIO:

TYPE I DM TYPE II DM

Pathogenesis Beta cells destruction Insulin resistance

Incidence rate 10-15% 90-95%

Onset Any; childhood/teens Any; over 40 y/o

Risk factor Genetic, autoimmune Genetic, obesity, lifestyle

C-peptide levels Decreased or undetectable detectable

Pre-diabetes Autoantibodies (+) Autoantibodies (-)

Symptomatology Symptoms develop abruptly Symptoms develope


gradually (some px are
asymptomatic)

Ketosis Commonl poorly controlled rare

medication Insulin absolute (IV or IM) Oral agents

99. Mr. Y has a uric acid level of 55.2 mg/dL. How will this affect the glucose measurement using glucose
oxidase/peroxidase method?
A. false low
B. false high
C. glucose level will be accurate
D. glucose level will exceed the linearity of the method

RATIO:

Note:
TRINDER’S REACTION is involved in the glucose oxidase method. Bilirubin, uric acid, ascorbate, tetracycline,
hemoglobin, and glutathione can interfere in the assay by competing for H2O2. This results in the loss of
H2O2 and thus leads to false readings.

100. Which of the following is/are TRUE about fluoride when used as an anticoagulant for glucose testing?
1. It acts by inhibiting the enzyme enolase and thus preserves glucose
2. It acts by chelating Mg2+
3. It can preserve glucose for upto 48 hours
4. It is added if serum is in contact with cells for longer than 30 minutes

A. 1,2,3,4
B. 1,2,3 only
C. 1,2 only
D. 1 only

RATIO: Fluoride preserves glucose by inhibiting the enzyme enolase. It acts by chelating Mg2+, which is an
activator of enolase. It can preserve glucose for up to 48 hrs.

For glucose measurement, testing should be performed within 30 minutes of collection to prevent cellular
consumption of glucose. If there is a delay of more than 30 minutes, fluoride should be added to the blood
sample.

Fluoride acts as preservative.

24
Analyte/Substance Reference range Conversion
Factor (CF)
Conventional Other (conventional x CF
= other)

Phospholipid 150-380 mg/dL

Lecithin/Sphingomyelin (L/S) ≥2
ratio

Cholesterol < 200 mg/dL

Borderline: 200-239
High = ≥240

Cholesterol Ester 70% of Cholesterol

Free cholesterol 30% of Cholesterol

Triglycerides (TAGs) < 150 mg/dL


AKA. Triacylglycerol; neutral fats
Borderline high= 150-199
High = 200-499
Very high = > 500

Fatty Acids 9-15 mg/dL

VLDL-C/triglyceride ratio 0.2


0.3 for
hyperlipoproteinemia
patients

Plasma Proteins 18-45 mg/dL 180-450 mg/L 10

Albumin 3.5-5.0 g/dL 35-50 g/L 10

Globulin 2.3-3.5 g/dL 23-35 g/L 10

Alpha1-antitrypsin (AAT) 145-270 mg/dL 1.45-2.70 g/L 0.01

Alpha1-fetoprotein (AFP) 5 ng/mL

Alpha1-Acid 55-140 mg/dL 0.55-1.40 g/L 0.01


GLycoprotein/orosomucoid
(AAG)

Alpha1-Antichymotrypsin (𝝰1-x) 30-60 mg/dL 0.30-0.60 g/L 0.01

Hemopexin 50-115 mg/dL 0.50-1.15 g/L 0.01

Group-specific component 20-55 mg/dL 0.20-0.55 g/L 0.01


(GC) globulin

Haptoglobin 26-185 mg/dL 0.26-1.85 g/L 0.01

Ceruloplasmin 18-45 mg/dL 180-450 mg/L 10

Alpha2-macroglobulin (AMG) 150-420 mg/dL 1.50-4.20 g/L 0.01

Beta2-Microglobulin 0.2-2.8 µg/dL

Transferrin (siderophilin) Male= 215-365 mg/dL 2.15-3.65 g/L 0.01

Female= 250-380 mg/dL 2.50-3.80 g/L

Fibrinogen 200-400 mg/dL 2.0-4.0 g/L 0.01

C-reactive protein (CRP) < 1 mg/dL

Troponins (Tn) <0.1 ng/mL < 0.1 µg/L


Albumin excretion rate in urine < 20 µg/min

CSF protein 15-45 mg/dL

LIVER FUNCTION TESTS

Total Protein 6.5-8/3 g/dL

Albumin 53-65% (of total


protein)
3.5-5.0 g/dL
Class E.C. number Enzyme Class Function Examples

1 Oxidoreductase Removal or addition of electrons (redox) LD, G6PD

2 Transferases Transfers chemical groups GGT, AST, ALT, CK

3 Hydrolases Splits/removes through hydrolysis LPS, AChE, Pseudocholinesterase, ALP, ACP, 5’


Nucleotidase, AMS

4 Lyases Splits/removes without hydrolysis Aldolase

5 Isomerases Catalyzes arrangement Ribose Phosphate Isomerase, Glucose Phosphate


Isomerase

6 Ligases Joining of molecules Synthase

Enzymes E.C. Nomenclature Major/Most Specific Tissue source Most Notable Diagnostic Significance Reference

LD-1&2: Heart, RBCs, kidneys Pernicious anemia (↑) Forward 100-255 U/L
Lactate Dehydrogenase (LD) 1.1.1.27 LD-3: Lungs, Pancreas Spleen Hemolytic disorders (anemia) (↑) Reverse 80-280 U/L
LD-4&5: Skeletal muscles, liver, spleen Myocardial Infarction (↑)
LD-2>LD-1 (flips during MI)

Glucose-6-Phosphate Dehydrogenase 1.1.1.49 Adrenal cortex, spleen, RBCs, lymph nodes Myocardial infarction and megaloblastic anemia (↑) 10-15 U/g hemoglobin
(G6PD) Drug-induced hemolytic anemia (↓) 1200-2000 mU/mL packed RBC

Gamma-Glutamyl Transferase (GGT) 2.3.2.2 Epithelial cells of biliary ductules (liver), kidney, Alcoholism/Acute Alcoholic Hepatitis (↑) Males 6-45 U/L
prostate, pancreas Females 5-30 U/L

Aspartate Aminotransferase (AST) 2.6.1.1 Cardiac, Liver, skeletal muscle Acute Myocardial Infarction (↑) 5-37 U/L

Acute Inflammatory Liver conditions (↑)


Alanine Aminotransferase (ALT) 2.6.1.2 Liver ● I.e. Hepatitis 6-37 U/L
Hepatitis Treatment and drug therapy

Creatine Kinase (CK) 2.7.3.2 Brain; smooth, skeletal, and cardiac muscles Duchenne’s Muscular Dystrophy (highest) (↑) Males 15-160 U/L
Acute Myocardial Infarction (CK-MB) (↑) Females 15-130 U/L
CK-MB <6% of total CK (< 5 µg/L)

Lipase (LPS) 3.1.1.3 Pancreas (most specific marker) Acute Pancreatitis (↑) 0-1.0 U/mL
Chronic Pancreatitis (↓)

True Acetylcholinesterase (AChE) 3.1.1.7

Pseudocholinesterase 3.1.1.8 Liver, myocardium, pancreas Insecticide/pesticide poisoning (↓) 0.5-1.3 pH units
Post-surgery evaluation of muscle relaxants (↓)

Alkaline phosphatase (ALP) 3.1.3.1 Non-specific Liver: Obstructive Jaundice (↑) 30-90 U/L
Placenta, intestine, liver, bone Bone: Paget’s disease (osteitis deformans) (↑)

Acid phosphatase (ACP) 3.1.3.2 Prostate Prostatic adenocarcinoma (↑) Total 2.5-11.7 U/L
Prostatic 0-3.5 ng/mL

5’ Nucleotidase 3.1.3.5 Liver Hepatobiliary disease, Infiltrative liver lesions 0-1.6 units

60-180 SU/dL
95-290 U/L
Amylase (AMY/AMS) 3.2.1.1 Acinar cells of the pancreas and salivary glands Acute pancreatitis (AP) (↑) AMY:CREA ratio
● 1-4%
● ≥ 4% (AP)

Angiotensin Converting Enzyme (ACE) 3.4.15.1 Lungs, testes, macrophages, epithelioid cells Sarcoidosis, MS, Addison’s disease, acute and chronic bronchitis,
HIV, leprosy

Aldolase 4.1.2.13 Skeletal muscles, bone marrow (?), liver Skeletal muscle disease, Leukemia, Hemolytic anemia, Hepatic
Cancer (↑)

Ribose Phosphate Isomerase 5.3.1.6

Glucose Phosphate Isomerase 5.3.1.9


1/7/23, 5:50 PM Medical Technology Board Examination Review Notes on Clinical Chemistry 2 - Students Lessons and Examinations

NEED HELP WITH YOUR PAPER? CLICK HERE! 

Park Systems
Infused with innovative robotics, intelligent learning
features, safety mechanisms

Park Systems Open

Medical Technology Board Examination Review Notes on


Clinical Chemistry 2
  JULY 04, 2018

 SHARE:  Facebook  Twitter  Google+  Pinterest

Clinical Chemistry

1.  Sodium fluoride is used in specimen collection to:


a. Prevent glycolysis
b. Prevent conversion of prothrombin to thrombin
c. Chelate calcium
d. Bind calcium

2. Serum iron should be drawn at the same time on successive days to avoid:
     a. Diurnal variation
     b. Chance of consumption of dietary iron
     c. Effect of medication
     d. Gastrointestinal absorption of iron

POPULAR

Medical Technology Board


Examination Review Notes
Recalls
 JULY 04, 2018

Medical Technology Board


Examination Review Notes on
3. The venipuncture site for a routine puncture is commonly cleaned with: Hematology
 JULY 04, 2018
     a. 90% alcohol
     b. Betadine
The Parable of Sadhu –
     c. 70% alcohol
Question of Ethics 
     d. Quarternary ammonium compounds  JUNE 28, 2018

lessonsandexaminations.blogspot.com/2018/07/medical-technology-board-examination-review-notes-on-clinical-chemistry.html 1/15
1/7/23, 5:50 PM Medical Technology Board Examination Review Notes on Clinical Chemistry 2 - Students Lessons and Examinations

4. Which of these substances cannot be preserved by freezing?


     a. BUN
     b. CK isoenzymes
     c. LDH
     d. Prostatic ACP

5. Which of the following is an appropriate sample for blood ammonia


determination?
     a. Arterial blood immediately chilled for 12 hours
     b. Arterial blood immediately chilled for 8 hours
     c. Arterial blood immediately chilled for 4 hours
     d. Freshly drawn blood

6. Which statement about wavelength calibration is true?


a. A didymium filter will have an absorbance maximum at 585 nm
b. Holmium oxide glass gives an absorbance maximum at 361 nm
c. The line emission of a hydrogen lamp at 656 nm can be used for
calibration
d. All of these

7. In spectrophotometric analysis, what is the purpose of the reagent


blank?
     a. Correct for interfering chromogens
     b. Correct for lipemia
     c. Correct for protein
     d. Correct for color contribution of the reagents

8. Nephelometry is based on the measurement of light that is:


     a. Blocked by particles in suspension
     b. Scattered by particles in suspension
     c. Produced by fluorescence
     d. Produced by excitation of ground state atoms

9. When measuring K+ with an ion-selective electrode by means of a liquid


ion-exchange membrane, what antibiotic will be incorporated into the
membrane?
     a. Vancomycin
     b. Streptomycin
     c. Valinomycin
     d. Nonactin

10. The purpose of the glass coils in a continuous flow system is to:
     a. Provide proper mixing
     b. Prevent carry-over of sample
     c. Allow visual inspection
     d. Allow close packing of tubing

11. The theoretic pH of pure water is:


     a. 6.0
     b. 7.0
     c. 7.4
     d. 8.0

12. How many mL of 95% alcohol do you need to make 1 liter of a 70%
solution?
     a. 74 mL
     b. 138 mL
     c. 737 mL
     d. 1360 mL

lessonsandexaminations.blogspot.com/2018/07/medical-technology-board-examination-review-notes-on-clinical-chemistry.html 2/15
1/7/23, 5:50 PM Medical Technology Board Examination Review Notes on Clinical Chemistry 2 - Students Lessons and Examinations
13. If 0.5 mL serum is diluted to a volume of 10 mL with normal saline, LABELS
what dilution is achieved?
     a. 1:40 PROFESSIONAL REGULATION COMMISSION
     b. 1:10 (470)
BOARD PASSERS (145)
     c. 1:5
     d. 1:20 DEFENSIVE DRIVING (132)

ROOM ASSIGNMENTS (130)


14. An explosion could result if:
ROAD SAFETY DRIVING (127)
     a. Acid is added to water
     b. Water is premixed with a weak salt TOP 10 PASSERS (105)

     c. Acid is neutralized and water added LEARN HOW TO DRIVE (97)
     d. Water is added to concentrated acid
ACADEMIC PAPERS (94)

DRIVING SCHOOL (92)

TOP PERFORMING SCHOOLS (87)

15. Given the following laboratory data: HUNTING (83) HUNTING IN LABRADOR (83)
     Serum creatinine = 4.0 mg/dL
ENTERPRISE RESOURCE PLANNING SOFTWARE
     Urine creatinine = 80 mg/dL
(77)
     24 hour volume = 360 mL CUSTOMER RELATIONSHIP MANAGEMENT (74)

REMOTE CONTROL (50) RESEARCH (47)


The creatinine clearance is:
SOUND SYSTEMS (47) SUBWOOFERS (47)
  a. Normal for an adult
  b. Low for an adult DEER HUNTING (41)

  c. High for an adult    HUMAN CAPITAL MANAGEMENT (39)


  d. Unable to calculate with data given
HUMAN RESOURCES MANAGEMENT SOFTWARE
(39)
16. All emergency (STAT) laboratory analyses should be reported to the SURROUND SYSTEM (37) COMPUTER (33)

ordering physician within: HARD DRIVE (32) NOTEBOOK (32)


     a. 30 mins to 1 hour
PROFESSIONAL TEACHERS (31)
     b. 10 to 20 minutes
LAPTOP (31) COYOTE HUNTING (29)
     c. 1 to 2 hours
     d. 3 hours FISHING (28) DUCT TAPE (25)

ELECTRICAL TAPE (25)


17. A properly labeled specimen will include all of the following EXCEPT:
     a. Name of patient FISHING IN LABRADOR (25)

     b. Unique identification number MASKING TAPE (25) PACKING TAPE (25)
     c. Date of collection
SURGICAL TAPE (25)
     d. Date of birth of patient
MEDICAL TECHNOLOGY (23)

18. Which of the following is caused by a random error? FINANCIAL MANAGEMENT (22)
1. R:4s     3. 1:2s       5. 10:x
CURLY FRIES (20) FRENCH FRIES (20)
2. 4:1s     4. 1:3s       6. 2:2s
PROFESSIONAL TEACHER (20)
a. 1, 2, 3, and 6
     b. 1, 4 and 6 MASTER PLUMBERS (18)
     c. 1 and 4 INTERNET MARKETING (18)
     d. 2, 3 and 5
PERSONAL FINANCE (18)

19. It is defined as the proportion of individuals without a condition who VIDEO MARKETING (18)
have a negative test for that condition
DEBT SOLUTION (17) INSURANCE (17)
     a. Diagnostic sensitivity
SOUND THERAPY (17) DEBTS (16)
     b. Diagnostic specificity
     c. Positive predictive value MEDICAL TECHNOLOGISTS (15)

     d. Negative predictive value OCCUPATIONAL THERAPISTS (15)

PHYSICAL THERAPISTS (15)


20. It refers to the ability of a method to measure only the analyte of
interest ACCOUNTING SOFTWARE (15)

     a. Accuracy BUSINESS INTELLIGENCE SOFTWARE (15)


     b. Precision
E-COMMERCE BLOGS (15)
     c. Specificity
     d. Sensitivity MANAGEMENT INTELLIGENCE (15)

PHYSICIANS (14) NURSES (14) 


ONLINE MARKETING (14)

lessonsandexaminations.blogspot.com/2018/07/medical-technology-board-examination-review-notes-on-clinical-chemistry.html 3/15
1/7/23, 5:50 PM Medical Technology Board Examination Review Notes on Clinical Chemistry 2 - Students Lessons and Examinations
21. What term applies to the sum of all the values in a set of numbers E-COMMERCE SOFTWARE (13)
divided by the number of values in that set?
LEASE MANAGEMENT (13)
     a. Median
     b. Mode PSYCHOLOGISTS (12)

     c. Mean PSYCHOMETRICIANS (12)


     d. Standard deviation
CERTIFIED PUBLIC ACCOUNTANTS (12)

22. What does the preparation of a Levey-Jennings QC chart for any single HOW TO BUILD YOUR BUSINESS WEBSITE (12)

constituent of serum require? MOOSE HUNTING (12)


     a. Analysis of control serum over a period of 20 consecutive days
GUIDANCE COUNSELORS (11) LOVE (11)
     b. 20 to 30 analyses of the control serum on 1 day, in one batch
     c. Analyses consistently performed by one person PHARMACISTS (11)

     d. Weekly analyses of the control serum for 1 month SANITARY ENGINEERS (11)
    
CONSOLIDATED LOAN (11)
23. A group of physicians consistently complains that they are not
receiving STAT patient results quickly enough. The supervisor is likely to OUTDOOR ADVENTURE (11)

refer to which quality assessment variable? ARTICLE MARKETING (10)


     a. Test utilization
BARACK OBAMA (10)
     b. Turnaround time
     c. Specimen separation and aliquoting COFFEE MACHINES (10)

     d. Analytical methodology COFFEE MAKERS (10)

FIFTY SHADES OF GREY (10)


24. It refers to the ability of an analytical method to maintain both
accuracy and precision over an extended period of time? GUITAR PLAYING (10) GUITARS (10)

     a. Probability LOSING WEIGHT FAST (10)


     b. Practicability
PERSONAL DEVELOPMENT (10)
     c. Reliability
BLOG ARCHIVE WHITE HOUSE (10)
ROMANCE (10)
     d. Validity
►LANDSCAPE
2019 (33) ARCHITECTS (9)
25. Which Westgard multirole applies to a situation where one control point 2018 (108) (PRACTICAL) (9)
▼OPTOMETRISTS
exceeds the mean by +2SD and a second control point exceeds the mean by ▼ July (34)
-2SD? OPTOMETRISTS (WRITTEN)
Special Article: What's(9)
At the Bottom of
     a. 1:2s the Ocean?
ADDICTION (9) DRUG REHABS (9)
Licensure Examination for Teachers
     b. 2:2s (LET)/ Professi...
MECHANICAL ENGINEERS (9)
     c. 4:1s Licensure Examination for Teachers
     d. R:4s ELECTRONICS
(LET)/ TECHNICIANS
Professi... (8)
Licensure Examination for Teachers
INTERIOR DESIGNERS (8)
(LET)/ Professi...
26. Hemolysis releases intracellular ions into the serum. Therefore, even Licensure Examination for Teachers
SOCIAL WORKERS (8)
slight hemolysis will cause erroneous results for the following: (LET)/ Professi...
1. Na++      3. Mg++        5. Cl- X-RAY TECHNOLOGISTS
Licensure Examination(8) for Teachers
(LET)/ Professi...
2. K+       4. HCO3-       6. Fe++ CRIMINOLOGISTS (8) ARCHITECTS (7)
Air Transport Economics: United Arab
a. 1, 2 and 3           c. 2 Emirates Air ...
ELECTRONICS ENGINEERS (7)
b. 2, 3, 4 and 5        d. 2, 3, 6 Case Study for Saudi Arabian Airlines
FISHERIES TECHNOLOGISTS
Management S... (7)
Geology Board Examination Review Notes
27. Symptom of hypocalcemia is typically: GEOLOGISTS (7)
1
     a. Stupor Medical Technology
METALLURGICAL ENGINEERS Board
(7) Examination
     b. Tetany Review Notes ...
RADIOLOGIC TECHNOLOGIST
Criminology (7)
Board Examination Review
     c. Tremors
Notes 3
     d. Dehydration CHEMICAL ENGINEERS (7) DRUGS (7)
Criminology Board Examination Review
WEIGHTNotes 2
LOSS PROGRAM (7)
28. The Fiske-Subbarow method is based upon the reaction of inorganic Criminology Board Examination Review
Notes 1 (6)
AGRICULTURISTS
phosphorus with:
Medical Technology Board Examination
     a. Ammonium molybdate DENTISTS (PRACTICAL) (6)
Review Notes ...
     b. Oxalic acid Medical
GEODETIC Technology
ENGINEERS (6) Board Examination
     c. Mercuric nitrate Review Notes ...
NUTRITIONISTS-DIETITIANS
Medical Technology Board (6) Examination
     d. p-dimethylaminobenzaldehyde
Review Notes ...
AUTOMOTIVE INDUSTRY (6)
Medical Technology Board Examination
Review
BREEDING Notes ...(6)
STRATEGIES DECOYS (6)
Medical Technology Board Examination
Review Notes
HOME MORTGAGE (6) ...
29. Determine the anion gap from the given serum electrolyte data: Medical Technology Board Examination
MECHANICAL DECOYS (6)
Na++ = 132 mmol/L     HCO3- = 22 mmol/L Review Notes ...

Medical
MOTION Technology
DECOYS (6) Board Examination
Cl- = 90 mmol/L
Review Notes ...

lessonsandexaminations.blogspot.com/2018/07/medical-technology-board-examination-review-notes-on-clinical-chemistry.html 4/15
1/7/23, 5:50 PM Medical Technology Board Examination Review Notes on Clinical Chemistry 2 - Students Lessons and Examinations
  a. 12 mmol/L Medical
PERSONAL Technology
HEALTH (6) Board Examination
REFINANCING (6)
Review Notes ...
  b. 64 mmol/L SUGAR GLIDERS (6)
Medical Technology Board Examination
  c. 20 mmol/L Review Notes ...
TIPS IN LOSING WEIGHT (6)
  d. Cannot be determined from the information provided Medical Technology Board Examination
TIPS INReview Notes ... (6)
SOUND THERAPY
Medical Technology Board Examination
30. Major extracellular anion WEIGHTReview
LOSS (6)
Notes WORDPRESS
... (6)
     a. Sodium Medical Technology Board
CERTIFIED PLANT MECHANICS (5)Examination
     b. Potassium Review Notes ...
     c. Chloride Medical
OCULAR Technology Board
PHARMACOLOGISTS (5) Examination
Review Notes ...
     d. Bicarbonate RADIOLOGIC TECHNOLOGISTS (5)
Medical Technology Board Examination
Review Notes ...
ADVENTURE (5) BUSINESS (5)
31. Which of the following conditions will cause an increased osmolal gap? Medical Technology Board Examination
CARIBOUReview Notes
HUNTING (5)... CHEMISTS (5)
     a. Drug overdose
Medical Technology Board Examination
     b. Diabetic ketoacidosis CIVIL ENGINEERS (5) ... E-COMMERCE (5)
Review Notes
     c. Renal failure Medical Technology Board Examination
FORESTERS (5) LEGAL (5)
     d. All of these Review Notes ...
LIFE Medical
COACHESTechnology
(5) Board Examination
Review Notes ...
32. Hyperaldosteronism will cause ____ serum sodium and ___ serum potassium AERONAUTICAL ENGINEERS (4)
Medical Technology Board Examination
levels Review Recall...
BUSINESS MANAGEMENT (4)
     a. Increased; Decreased Medical Technology Board Examination
Review
CERTIFIED PLANTNotes ...
MECHANICS (CPM) (4)
     b. Increased; Increased
Medical Technology Board Examination
     c. Decreased; Increased CUSTOMS BROKERS
Review Notes(4)...
     d. Decreased; Decreased ► June (43)
LET BOARD EXAM (4)
► May (9)
MARINE ENGINEERS
► March (13) (4)
33. Which of the following is the primary mechanism for antidiuretic
hormone (ADH) release? ► February (9)ENGINEERS (COMPUTER-BASED
METALLURGICAL
     a. Hypovolemia ►EXAM)
2017 (33)
(4)NAVAL ARCHITECTS (4)
► 2016 (138)
     b. Hyperosmolar plasma
2015 (157)
►OCULAR PHARMACOLOGY (4)
     c. Renin release
► 2014 (219)
     d. Reduced renal blood flow REAL ESTATE APPRAISERS (4)
► 2013 (863)
REGISTERED ELECTRICAL ENGINEERS (4)
34. Which of the following clearance tests offers the most accurate measure
REGISTERED MASTER ELECTRICIANS (4)
of glomerular filtration?
     a. Inulin RESPIRATORY THERAPISTS (4)
     b. Creatinine DOG TRAINER (4) MIDWIVES (4)
     c. p-aminohippurate (PAH)
NATURAL MOTION DECOYS (4)
     d. Urea
PRESTASHOP (4)
35. All but one of the following matches is correct: AGRICULTURAL ENGINEERS (3)
     a. Urea – product of protein catabolism
ELECTRONICS TECHNICIAN (3)
     b. Creatine – product of muscle catabolism
     c. Uric acid – product of pyrimidine catabolism MARINE ENGINEER OFFICERS (WRITTEN) (3)

     d. Bilirubin – product of heme catabolism METALLURGICAL ENGINEERS (WES) (3)

REAL ESTATE CONSULTANTS (3) BIKING (3)


36. The classic creatinine reaction is that of:
     a. Jaffe BIRD HUNTING (3) BUSINESS TIPS (3)

     b. Lloyd CRIMINOLOGY (3) DOMAIN NAMES (3)


     c. Kjeldahl
FORECLOSURE (3) FOREX (3)
     d. Nessler
LIVE FOR GOD (3) MOVING (3)

37. To determine if a 24 hour urine collection is complete, which of the MUSIC THERAPY (3) OVERWEIGHT (3)
following determinations may be helpful?
PERFUME (3)
     a. BUN
     b. Osmolality SENATE OF THE PHILIPPINES (3)

     c. Protein WEBSITE DESIGN (3)


     d. Creatinine
AGENCYBLOC SOFTWARE CUSTOMER
RELATIONSHIP MANAGEMENT
38. A condition in which the metabolism of uric acid, but not of other (2)ALSAMARKETING MARKETING AUTOMATION
SOFTWARE
nitrogenous urinary constituents, is impaired: (2)CERTIFIED PUBLIC ACCOUNTANTS (SPLBE) (2)
     a. Uremia
     b. Azotemia
     c. Gout 

lessonsandexaminations.blogspot.com/2018/07/medical-technology-board-examination-review-notes-on-clinical-chemistry.html 5/15
1/7/23, 5:50 PM Medical Technology Board Examination Review Notes on Clinical Chemistry 2 - Students Lessons and Examinations
     d. Nephritis

39. The cause(s) of prerenal azotemia is (are):


     a. Dehydration
     b. Congestive heart failure
     c. Shock of hemorrhage
     d. All of the above

40. The urea nitrogen method using diacetyl monoxime measures:


     a. Urea nitrogen only
     b. Urea nitrogen and ammonia
     c. Urea nitrogen and amino acids
     d. Urea nitrogen and peptide bonds

41. Urea concentration is calculated from the blood urea nitrogen (BUN) by
multiplying by the factor of:
     a. 0.5
     b. 2.14
     c. 6.45
     d. 14

42. Which of the following is classified as a mucopolysaccharide storage


disease?
     a. Pompe’s disease
     b. von Gierke’s disease
     c. Andersen’s disease
     d. Hurler’s syndrome

43. Which of the following is the reference method for measuring serum
glucose?
     a. Somogyi-Nelson
     b. Hexokinase
     c. Glucose oxidase
     d. Glucose dehydrogenase

44. Which of the following hormones does not promote an increase in blood
glucose levels?
     a. Growth hormone
     b. Cortisol
     c. Insulin
     d. Glucagon

45. Which of the following does not properly describe type I diabetes
mellitus?
     a. Insulin deficiency
     b. Associated with autoimmune destruction of pancreatic beta-cells
     c. Ketoacidosis prone
     d. Occurs more frequently in adults

46. For every 1% change in the HbA1c value, ___ mg/dL is added to plasma
glucose
     a. 15
     b. 25
     c. 35
     d. 50

47. According to WHO, what is the standard glucose load for OGTT procedure?
     a. 75g
     b. 150g

     c. 50g

lessonsandexaminations.blogspot.com/2018/07/medical-technology-board-examination-review-notes-on-clinical-chemistry.html 6/15
1/7/23, 5:50 PM Medical Technology Board Examination Review Notes on Clinical Chemistry 2 - Students Lessons and Examinations
     d. 100g

48. Type I and V hyperlipoproteinemia are characterized by large increase


in:
     a. Chylomicrons
     b. Low density lipoproteins (LDL)
     c. Very low density lipoproteins (VLDL)
     d. High density lipoproteins (HDL)

49. Serum turbidity is due to:


     a. Proteins
     b. Glucose
     c. Triglycerides
     d. Cholesterol

50. To produce reliable results, at which time should blood specimens for
lipid studies be drawn?
     a. 2-4 hour fasting
     b. 6-8 hour fasting
     c. 8-10 hour fasting
     d. 12-16 hour fasting

51. Which of the following is the Friedewald formula by which LDL


cholesterol can be estimated?
a. LDL cholesterol = Total cholesterol – (Triglycerides/5 + HDL
cholesterol)
b. LDL cholesterol = Total cholesterol – (Triglycerides +
Phospholipids)
c. LDL cholesterol = HDL – Total cholesterol
d. LDL cholesterol = Total cholesterol – ½ HDL cholesterol

52. Which of the following matches is incorrect?


     a. LpX – Obstructive jaundice and LCAT deficiency
     b. Lp(a) – Sinking pre-beta lipoprotein
     c. β-VLDL – Floating beta lipoprotein
     d. IDL – Migrates in the alpha region (electrophoresis)

53. Which of the following lipoproteins is composed of 45-50% of


cholesterol esters?
     a. HDL
     b. VLDL
     c. LDL
     d. Chylomicrons

54. This is the reference method for quantitation of lipoproteins


     a. Chemical precipitation
     b. Ultracentrifugation
     c. Electrophoresis
     d. Abell, Levy and Brodie method

55. It is an autosomal recessive disorder characterized by defective apo B


synthesis and absence of VLDL, LDL and chylomicrons in the plasma
     a. Tangier’s disease
     b. Lipoprotein lipase deficiency
     c. Lecithin Cholesterol Acyl Transferase (LCAT) deficiency
     d. Basses-Kornzweig syndrome

56. Hyperalbuminemia is caused by:


     a. Gastroenteropathy

     b. Dehydration syndromes

lessonsandexaminations.blogspot.com/2018/07/medical-technology-board-examination-review-notes-on-clinical-chemistry.html 7/15
1/7/23, 5:50 PM Medical Technology Board Examination Review Notes on Clinical Chemistry 2 - Students Lessons and Examinations
     c. Burns
     d. Liver disease

57. Which of the following conditions is associated with “beta-gamma


bridging”?
     a. Multiple myeloma
     b. Malignancy
     c. Liver cirrhosis
     d. Rheumatoid arthritis

58. During serum electrophoresis, which of the following proteins migrate


in the alpha-2 region?
1. Alpha-feto protein          3. Haptoglobin     5. Transferrin
2. Hemopexin              4. Ceruloplasmin   6. Complement
  a. 1, 2, 5 and 6             c. 3 and 4
  b. 1, 3, 4, and 6            d. 2 and 3

59. Which of the following statements is true of albumin?


a. Compared to globulin, it makes up the lesser portion of total
protein
b. Its size prevents its passage through even a damaged glomerular
barrier
c. It is produced in the liver
d. Clinical problems are usually related to high serum values

60. Which reagent is employed in the serum protein determination?


     a. Molybdenum blue
     b. Ferriferrocyanide
     c. Resorcinol-HCl
     d. Biuret

61. The most sensitive marker of acute phase inflammation is probably:


     a. C3 complement
     b. C-reactive protein
     c. ESR
     d. Fever

62. Falsely elevated ammonia levels on blood could be caused by:


1. Not chilling blood & analyzing immediately 3. Poor venipuncture
technique
2. Ammonia contamination from glassware  4. Cigarette smoking
  a. 1, 2 and 3      c. 1, 2 and 4
  b. 2, 3 and 4      d. 1, 2, 3 and 4

63. Pre-hepatic jaundice is caused by:


     a. Hemolytic anemia
     b. Cirrhosis
     c. Bile duct obstruction
     d. Hepatitis

64. Kernicterus is caused by:


     a. Deposits of ferric iron in brain tissue
     b. Elevation of bilirubin in serum
     c. Deposits of unconjugated bilirubin in brain cells
     d. Deposits of conjugated bilirubin in brain cells

65. This is characterized by an inability to transport bilirubin from the


sinusoidal membrane into the hepatocyte:
     a. Dubin Johnson syndrome
     b. Crigler-Najjar syndrome

     c. Gilbert syndrome

lessonsandexaminations.blogspot.com/2018/07/medical-technology-board-examination-review-notes-on-clinical-chemistry.html 8/15
1/7/23, 5:50 PM Medical Technology Board Examination Review Notes on Clinical Chemistry 2 - Students Lessons and Examinations
     d. Rotor syndrome
66. As the red blood cells disintegrate, hemoglobin is released and
converted to the pigment bilirubin. Which organ is primarily responsible
for this function?
     a. Spleen
     b. Kidneys
     c. Intestines
     d. Liver

67. Possibly the most sensitive enzyme indicator of liver function,


particularly in obstructive jaundice is:
     a. Alkaline phosphatase
     b. Alanine aminotransferase
     c. Acid phosphatase
     d. Lactate dehydrogenase

68. In acute viral hepatitis, which of the following would NOT be


suspected?
     a. Lactate dehydrogenase – 5x increase
        b. Alkaline phosphatase - increase is greater than aspartate
aminotransferase
     c. Gamma-glutamyl transferase – mild increase
        d. Aspartate aminotransferase and alanine aminotransferase – 10- to
200-fold increase

69. Which of the following markers will remain elevated for 48-72 hours
after the onset of myocardial infarction?
     a. CK-MB
     b. AST
     c. LDH
     d. Troponin I

70. Which of the following enzymes are expected to be on peak activity 24


hours after the onset of acute pancreatitis?
     a. Amylase
     b. Lipase
     c. Both of these
     d. None of these

71. Which of the following match(es) is (are) correct?


1. CK = Tanzer-Gilbarg and Oliver-Rosalki method
2. LD = Wacker and Wrobleuski La Due method
3. LPS = Cherry Crandal method
4. AST & ALT = Karmen & Reitman-Frankel method
  a. 1, 2 and 3
  b. 2, 3 and 4
  c. 1, 3 and 4
  d. 1, 2, 3 and 4

72. Which of the following match(es) is (are) incorrect?


1. AMS = Saccharogenic method
2. GGT = Ellman method
3. CHS = Szass method
4. 5’NT = Dixon & Purdon and Campbell, Belfield & Goldberg method
  a. 1 and 4
  b. 1 and 3
  c. 2 and 4
  d. 2 and 3

73. Hemolysis affects which of the following enzymes?



1. CK-MB         3. LDH

lessonsandexaminations.blogspot.com/2018/07/medical-technology-board-examination-review-notes-on-clinical-chemistry.html 9/15
1/7/23, 5:50 PM Medical Technology Board Examination Review Notes on Clinical Chemistry 2 - Students Lessons and Examinations
2. ACP      4. AMS
  a. 1, 2 and 3
  b. 1, 2 and 4
  c. 1, 3 and 4
  d. 1, 2, 3 and 4

74. An International Unit (IU) of enzyme activity is the quantity of enzyme


that:
     a. Converts 1 micromole of substrate to product per liter
     b. Forms 1 mg of product per deciliter
     c. Converts 1 micromole of substrate to product per minute
     d. Forms 1 millimole of product per liter

75. Which isoenzyme of ALP is most heat stable?


     a. Bone
     b. Liver
     c. Intestinal
     d. Placental

76. Which of the following conditions will increase total T4 by increasing


TBG?
     a. Pregnancy or estrogens
     b. Acute illness
     c. Nephrotic syndrome
     d. Anabolic steroid use

77. Select the most appropriate single screening test for thyroid disease
     a. Free thyroxine index
     b. TSH assay
     c. Total T3 level assay
     d. Total T4 level assay

78. What is the predominant form of thyroid hormone in the circulation?


     a. Thyroxine
     b. Triiodothyronine
     c. Diiodotyrosine
     d. Monoiodotyrosine

79. The presence of a very high titer for anti-thyroglobulin antibodies and
the detection of anti-thyroid peroxidase antibodies is highly suggestive of
what disorder?
     a. Graves’ disease
     b. Hashimoto’s thyroiditis
     c. Pernicious anemia
     d. Thyroid adenoma

80. The Kober reaction is a test for:


     a. Catecholamines
     b. 17-Ketogenic steroids
     c. Estrogens
     d. 17-Hydroxycorticosteroids

81. Human chorionic gonadotropin (hCG) is secreted by the:


     a. Thymus gland
     b. Placenta
     c. Ovary
     d. Ureter

82. Which of the following is the most potent androgen?


     a. Androstenedione
     b. Dehydroepiandrosterone
     c. Androsterone 

lessonsandexaminations.blogspot.com/2018/07/medical-technology-board-examination-review-notes-on-clinical-chemistry.html 10/15
1/7/23, 5:50 PM Medical Technology Board Examination Review Notes on Clinical Chemistry 2 - Students Lessons and Examinations
     d. Testosterone

83. The chief urinary metabolite of norepinephrine and epinephrine is:


     a. Metanephrine
     b. Vanillylmandelic acid
     c. Homovanillic acid
     d. 5-HIAA

84. A high urine catecholamine level would be expected in:


     a. Argentaffinoma
     b. Pheochromocytoma
     c. Addison’s disease
     d. Cushing’s disease

85. Acromegaly is caused by:


     a. Overproduction of growth hormone
     b. Deficiency in growth hormone
     c. Galactorrhea
     d. Stimulation by GnRH

86. For what colorimetric determination is the Trinder reaction widely


used?
     a. Acetaminophen
     b. Salicylate
     c. Barbiturate
     d. Benzodiazepines

87. Acetaminophen is particularly toxic to what organ?


     a. Heart
     b. Kidney
     c. Spleen
     d. Liver

88. THC (Δ-tetrahydrocannabinol) is the principal active component of what


drug?
     a. Benzodiazepine
     b. Cocaine
     c. Marijuana
     d. Morphine

89. What is the major urinary metabolite of cocaine?


     a. Morphine
     b. Benzoylecgonine
     c. NAPA
     d. Primidone

90. Which of the following metals has a characteristic “odor of garlic” and
“metallic taste”?
     a. Cyanide
     b. Arsenic
     c. Lead
     d. Mercury

91. A blood alcohol level of 0.35-0.50 (% w/v) is associated with:


     a. Unable to stand/walk, vomiting, and impaired consciousness
     b. Decreased inhibitions, loss of critical judgment, memory impairment
& decreased reaction time
     c. Coma and possible death
     d. Mild euphoria, decreased inhibitions, and some impairment of motor

skills

lessonsandexaminations.blogspot.com/2018/07/medical-technology-board-examination-review-notes-on-clinical-chemistry.html 11/15
1/7/23, 5:50 PM Medical Technology Board Examination Review Notes on Clinical Chemistry 2 - Students Lessons and Examinations

92. The ratio of bicarbonate: carbonic acid in normal plasma is:


     a. 1:20
     b. 1:10
     c. 20:1
     d. 10:1

93. What will happen if blood is exposed to air during collection for pH
and blood gas studies?
     a. CO2 content increases
     b. pH decreases
     c. pO2 decreases
     d. pCO2 decreases

94. Emphysema, pneumonia and other pulmonary diseases are associated with
which acid-base disorders?
     a. Respiratory alkalosis
     b. Metabolic alkalosis
     c. Respiratory acidosis
     d. Metabolic acidosis

95. For each degree of fever in a patient, pO2 values will decrease by ___
% and pCO2 values will increase by ___%
     a. 7; 3
     b. 3; 7
     c. 3; 3
     d. 7; 7

96. Which biological samples should be regarded as potential hazards?


     a. Urine specimens from AIDS patients
     b. Stool specimens
     c. All biological samples
     d. Sputum specimens from TB patients

97. Flammable liquids may be stored


     a. In an ordinary refrigerator with a flammable storage label affixed
     b. In any refrigerator within the laboratory department
     c. In an explosion-proof refrigerator
     d. Only in an explosion-proof refrigerator in a remote area

98. Food and drink may be stored in a refrigerator if:


     a. Placed in a special sealed container
     b. Marked as a consumable material
     c. Food and drink may not be placed in a laboratory refrigerator
     d. Stored for short periods of time

99. Laboratory supplies should NOT be stored


     a. In cabinets and drawers
     b. On the floor
     c. On shelving
     d. At floor level if placed upon a raised platform

100. This class of fire is usually allowed to burn out and nearby materials
protected
     a. Type A (Ordinary Combustibles)
     b. Type B (Flammable Liquids)
     c. Type D (Flammable Metals)
     d. Type E (Arsenal Fire)

lessonsandexaminations.blogspot.com/2018/07/medical-technology-board-examination-review-notes-on-clinical-chemistry.html 12/15
1/7/23, 5:50 PM Medical Technology Board Examination Review Notes on Clinical Chemistry 2 - Students Lessons and Examinations
ANSWER KEY: Clinical Chemistry

1.      A
2.      A
3.      C
4.      C
5.      D
6.      D
7.      D
8.      B
9.      C
10.  A
11.  B
12.  C
13.  D
14.  D
15.  B
16.  A
17.  D
18.  D
19.  B
20.  C
21.  C
22.  A
23.  B
24.  C
25.  D
26.  D
27.  B
28.  A
29.  C
30.  C
31.  D
32.  A
33.  B
34.  A
35.  C
36.  A
37.  D
38.  C
39.  D
40.  A
41.  B
42.  D
43.  B
44.  C
45.  D
46.  C
47.  A
48.  A
49.  C
50.  D
51.  A
52.  D
53.  C
54.  B
55.  D
56.  B
57.  C
58.  C
59.  C
60.  D
61.  B 
62.  D

lessonsandexaminations.blogspot.com/2018/07/medical-technology-board-examination-review-notes-on-clinical-chemistry.html 13/15
1/7/23, 5:50 PM Medical Technology Board Examination Review Notes on Clinical Chemistry 2 - Students Lessons and Examinations
63.  A
64.  C
65.  C
66.  A
67.  A
68.  B
69.  A
70.  C
71.  D
72.  D
73.  A
74.  C
75.  D
76.  A
77.  B
78.  A
79.  B
80.  C
81.  B
82.  D
83.  B
84.  B
85.  A
86.  B
87.  D
88.  C
89.  B
90.  B
91.  C
92.  C
93.  D
94.  C
95.  A
96.  C
97.  C
98.  C
99.  B
100. D

LABELS: MEDICAL TECHNOLOGY

 SHARE:  Facebook  Twitter  Google+  Pinterest

YOU MIGHT ALSO LIKE

Medical Technology Board Exam Medical Technology Board Medical Technology Board
Reviewer HEMATOLOGY Examination Review Notes on Examination Review Notes
 JULY 04, 2019 Parasitology Recall 3
 JULY 05, 2018  JULY 04, 2018

CATEGORIES ARCHIVE POPULAR POSTS

July (34)
Medical Technology Board Examination Review Notes
Recalls 

lessonsandexaminations.blogspot.com/2018/07/medical-technology-board-examination-review-notes-on-clinical-chemistry.html 14/15
1/7/23, 5:50 PM Medical Technology Board Examination Review Notes on Clinical Chemistry 2 - Students Lessons and Examinations

Medical Technology Board Examination Review Notes


on Hematology

The Parable of Sadhu – Question of Ethics

Crafted with  by TemplatesYard | Distributed by Gooyaabi Templates     

lessonsandexaminations.blogspot.com/2018/07/medical-technology-board-examination-review-notes-on-clinical-chemistry.html 15/15
CLINICAL CHEMISTRY MCQs
A. CARBOHYDRATES

_____1. Hypoglycemia comes about for various reasons and clinical symptoms usually
occur at blood glucose concentrations:
A. <3.8mmol/L B. < 3.0mmol/L C. <2.8mmol/L D. <5.0mmol/L
_____2. The healthy organism maintains the extracellular glucose concentration
remarkably constant within the limits of:
A. 5-12mmol/L C. 2.8- 3.8mmol/L
B. 3.3 – 10mmol/L D. 1.2- 2.4 mmol/L
_____3. Which classification of diabetes mellitus is genetically predetermined, HLA-
associated, chronic autoimmune disease with insulin deficiency and glucagons
excess as its pathophysiologic sequelae?
A. Type I C. non-obese type II
B. obese type II D. MODY-maturity onset diabetes of the young
_____4. According to the WHO criteria, patients with type I diabetes mellitus are, at the
time of diagnosis, characterized by:
1. hyperglycemia 2. ketonuria 3. ketoacidosis 4. normal weight
A. only 1 and 3 are correct C. only 1,2, and 3 are correct
B. only 2 and 4 are correct D. 1,2,3 and 4 are correct
_____5. In diabetic patient, this provides information about the average blood glucose
concentration during the preceding 6-8 weeks:
A. HbA1c C. HbB
B. HbF D. HbS
_____6. This term describes the disturbed insulin effect on cells such as adipocytes,
skeletal muscle cells, and hepatocytes. In simplified terms, glucose uptake by
these insulin-dependent cells is reduced. As compensdation, a rise in insulin
secretion with hyperinsulinemia ensues.
A. progressive hyperglycemia C. insulin resistance
B. impaired glucose tolerance D. glucose-mediated insulin secretion
_____7. Diabetes mellitus secondary to certain and other diseases and disorders may
involve the following organs:
1. pancreas 2. liver 3. anterior pituitary 4. adrenal gland
A. only 1 and 3 are correct C. only 1,2, and 3 are correct
B. only 2 and 4 are correct D. 1,2,3 and 4 are correct
_____8. In the case of gestational diabetes mellitus, a normal diurnal glucose profile
performed on the 3rd to 4th postpartum day rules out persistent diabetes mellitus.
In women with macrosomic children, the diurnal glucose profile does not
become valuable as an indicator until how many weeks after delivery?
A. 3 weeks B. 6 weeks C. 9 weeks D. 12 weeks
_____9. The determination of glucose tolerance following the oral administration of
glucose is recommended as a screening test for the presence of underlying
gestational diabetes mellitus. How much gram of glucose is recommended in
this tolerance test?
A. 50 grams B. 75 grams C. 100 grams D. 120 grams
____10. Which of the following statements is true in screening tests for diabetes
mellitus?
A. The European Association for the Study of Diabetes recommends to
perform the Oral glucose tolerance test using 75g of glucose with 3 capillary
blood collections.
B. The European Association for the Study of Diabetes recommends to
perform the Oral glucose tolerance test using 100g of glucose with 4
capillary blood collections.
C. In the USA the oral glucose tolerance test is performed using 100g of
glucose with 3 venipuntures.
D. In the USA the oral glucose tolerance test is performed using 75g of glucose
with 4 venipuntures.
_____11. Which chromosome number is associated with non-HLA genes type 1 diabetes
mellitus?
A. 9 B. 11 C. 15 D. 17
_____12. Which of the following clinical laboratory tests are used for detecting the risk
of complications in newborns born to mothers with gestational diabetes or type
1 diabetes mellitus?
1. glucose 2. hematocrit 3. bilirubin 4. calcium
A. only 1 and 3 are correct C. only 1,2, and 3 are correct
B. only 2 and 4 are correct D. 1,2,3 and 4 are correct
_____13. Which of the following glucose concentrations denote plasma insulin
concentration as almost zero?
A. 20 mg/dL B. 40 mg/dL C. 60 mg/dL D. 80mg/dL
_____14. What enzyme catalyzes the oxidation of glucose into gluconic acid and H2O2?
A. glucose oxidase C. glucose hexokinase
B. glucose dehydrogenase D. mutarotase
_____15. What enzyme catalyzes the oxidation of glucose to glucolactone?
A. glucose oxidase C. glucose hexokinase
B. glucose dehydrogenase D. mutarotase

_____16. The diagnosis of diabetes mellitus is reliably established by which of the


following criteria?
1. unequivocally elevated postprandial or non-fasting glucose concentration
with or without classical symptoms of diabetes mellitus such as polydipsia,
polyuria, polyphagia, and weight loss
2. fasting and preprandial glucose values should be <90 mg/dl
3. increased fasting blood glucose values on at least two occasions.
4. glucosuria is usually present without ketonuria
A. only 1 and 3 are correct C. only 1,2, and 3 are correct
B. only 2 and 4 are correct D. 1,2,3 and 4 are correct
____17. Which of the following statements regarding glucose blood samples is/are
correct?
1. After a period of fasting, the glucose concentration in venous blood is 5-
10% lower than arterial blood.
2. deproteinization will result to glucose decrease.
3. the glucose content of erythrocytes is lower than that in plasma
4. glucose values in venous plasma are 10% lower than those in venous whole
blood.
A. only 1 and 3 are correct C. only 1,2, and 3 are correct
B. only 2 and 4 are correct D. 1,2,3 and 4 are correct
_____18. In case of capillary blood sampling, 0.01-0.05 mL of blood are placed in a
solution to hemolyze red cells if the glucose determination will be performed
using automated analysis systems. Which of the following glycolysis
inhibitors are used with hemolyzation solution?
1. maleinimide 2. monoiodine acetate 3. sodium fluoride 4. cresol
A. only 1 and 3 are correct C. only 1,2, and 3 are correct
B. only 2 and 4 are correct D. 1,2,3 and 4 are correct
_____19. Which of the following statements are correct in relation to the stability of
glucose in blood samples?
1. In venous blood at 4C there is only a slight decrease during the first 2h and
approximately 20% after 24h.
2. In collection tubes containing EDTA there is no decrease within 24h in the
presence of maleinimide or sodium fluoride
3. by adding 2.5g/L sodium fluoride or 0.5g/L sodium monoiodine acetate the
decrease in plasma glucose is only 3-5% in 24h
4. Perchloric acid deproteinized sample gives stable values in the supernatant,
obtained by centrifugation for at least 5 days.
A. only 1 and 3 are correct C. only 1,2, and 3 are correct
B. only 2 and 4 are correct D. 1,2,3 and 4 are correct
_____20. Which method of determination for glucose is the reference method?
A. oxidase B. dehydrogenase C. hexokinase D. glucometers
_____21. Among the sugars usually occurring in the blood, which of the following are
detected by glucose dehydrogenase method?
1. glucose 2. sucrose 3. xylose 4. galactose
A. only 1 and 3 are correct C. only 1,2, and 3 are correct
B. only 2 and 4 are correct D. 1,2,3 and 4 are correct
_____22. In the hexokinase method, which of the following may be a source of
methodologic errors?
A. the addition of maleinimide in a hemolysate
B. deproteinization using perchloric acid
C. drug interference
D. maleinimide inhibits erythrocyte enzymes
_____23. Which of the following are diagnostic significance of glucose determinations
in body fluids?
1. In CSF, bacteria are associated with decreases in glucose concentration
2. In nonbacterial peritonitis the ascite/blood glucose ratio is > 1.0
3. In bacterial peritonitis the ascite/blood glucose ratio is < 1.0
4. Decreased values in conjunction with pleural fluid/blood glucose ratios <1.0
may occur in effusions due to underlying bacterial, tuberculous, malignant,
and rheumatic disease.
A. only 1 and 3 are correct C. only 1,2, and 3 are correct
B. only 2 and 4 are correct D. 1,2,3 and 4 are correct
_____24. The oral glucose tolerance test will provide valuable results if the following
requirements have been met by the patient prior to the test:
1. maintenance of usual eating habits for at least 3 days
2. discontinuation of interfering medications for at least 3 days
3. continuation of usual physical activities
4. a time interval to menstruation of at least 3 days
A. only 1 and 3 are correct C. only 1,2, and 3 are correct
B. only 2 and 4 are correct D. 1,2,3 and 4 are correct
_____25. In OGTT, a load of 75g of glucose (WHO recommendation) is ingested within:
A. 5mins B. 10 mins C. 20 mins D. 30mins
_____26. The screening test for gestational diabetes mellitus recommended between 24th
to 28th week of gestation requires 50g of glucose dissolved in 200ml of water
and slowly drunk. A blood sample is collected after:
A. 30mins B. 60 mins C. 90 mins D. 120mins
_____27. Gestational diabetes mellitus is present if the following threshold values are
obtained:
1. fasting > 90mg/dL
2. 60mins > 190mg/dL
3. 120mins > 160mg/dL
4. 150mins > 90mg/dL
A. only 1 and 3 are correct C. only 1,2, and 3 are correct
B. only 2 and 4 are correct D. 1,2,3 and 4 are correct
_____28. Which of the following statements regarding glycohemoglobins are correct?
1. The degree of glycation depends essentially on the degree as well as the
duration of the blood glucose elevation.
2. The nonglycated hemoglobin is Ao
3. The reference method for glycated hemoglobin analysis is FPLC
4. The mean blood glucose of the preceding 6-8 weeks is correlated with
corresponding glycohemoglobin value.

A. only 1 and 3 are correct C. only 1,2, and 3 are correct


B. only 2 and 4 are correct D. 1,2,3 and 4 are correct
_____29. Which of the following are done in C-peptide suppression test?
1. 1g of tolbutamide is administered to the patient intravenously over 3 mins as a
5% aqueous solution
2. children receive 25mg/kg body weight of tolbutamide but no more than 1g
3. prior to the start of the test as well as 5,10,20,3040,60,90,120 and 180mins
later blood samples are collected
4. glucose, insulin and C-peptide are determined
A. only 1 and 3 are correct C. only 1,2, and 3 are correct
B. only 2 and 4 are correct D. 1,2,3 and 4 are correct
_____30. In glucagon test, determination of glucose insulin, C-peptide and B-
hydroxybutyrate are done in sample collected:
A. prior to the start as well as 1,5,10,15 and 30 mins later.
B. prior to the start as well as 5,10,15,20 and 30 mins later.
C. prior to the start as well as 5,10,15, 30 and 60 mins later.
D. prior to the start as well as 10,15, 30 and 60 mins later.
_____31. Which of the following is true about portable glucose meters?
A. Not recommended for all insulin-treated patients.
B. Should be performed at least 2 times a day for type 1 diabetes.
C. Should be performed at least 3 times a day for type 2 diabetes.
D. Should not be used for diagnosis of diabetes and have limited value in
screening.
_____32. The need to reevaluate treatment regimen for diabetes is necessary when the
glycated hemoglobin is greater than:
A. 2% B. 4% C. 6% D. 8%
_____33. Which of the following statements about type 1 diabetes mellitus are correct?
1. Formerly known as insulin dependent
2. Adult onset
3. Caused by autoimmune destruction of the B-cells of the pancreas
4. The pancreas synthesizes insulin but unable to secrete.
A. only 1 and 3 are correct C. only 1,2, and 3 are correct
B. only 2 and 4 are correct D. 1,2,3 and 4 are correct
_____34. Which of the following is the correct diagnostic criterion for establishing
diabetes?
A. Fasting blood glucose greater than 7.0mmol/L
B. Fasting blood glucose greater than 4.0mmol/L
C. Fasting blood glucose greater than 2.0mmol/L
D. Fasting blood glucose greater than 20.0mmol/L
_____35. Which of the following is the correct diagnostic criterion for establishing
diabetes?
A. 2hpp greater than 7.0mmol/L
B. 2hpp greater than 9.11mmol/L
C. 2hpp greater than 11.1mmol/L
D. 2hpp greater than 20.0mmol/L
______36. For overweight individuals, starting at age 10 years, diabetes testing should
be performed every:
A. Month B. six months C. year D. 2years
______37. For inhibition of glycolysis, how much mg of fluoride must be used per mL
of blood?
A. 1.0 B. 2.5 C. 3.0 D. 0.5
______38. For inhibition of glycosis, how much lithium iodoacetate must be used per
mL of blood?
A. 1.0mg B. 2.5mg C. 3.0mg D. 0.5mg
______39. Given a normal hematocrit, the glucose concentrations in plasma is higher
than that of whole blood by approximately:
A. 7% B. 11% C. 4% D. 17%
______40. Glucose concentrations in heparinized plasma are reported to be lower than in
serum by:
A. 2% B. 5% C. 10% D. 0.5%
______41. Which of the following enzymatic methods is least used for glucose
measurement?
A. Hexokinase C. Dehydrogenase
B. Oxidase D. Isomerase
______42. Multiple factors can interfere with glucose analysis with portable meters such
as:
1. Improper application, timing and removal of excess blood
2. Changes in hematocrit, altitude, environmental temperature or
humidity
3. Training and technique of user
4. Hypotension, hypoxia and high triglyceride concentration
A. only 1 and 3 are correct C. only 1,2, and 3 are correct
B. only 2 and 4 are correct D. 1,2,3 and 4 are correct
______43. SMBG should be performed at least how many times per day in patients with
type 1 diabetes?
A. 1 B. 2 C. 3 D. 4
______44. High-risk pregnant patients should undergo immediate testing when they are
defined as having which of the following?
1. marked obesity 3. Glycosuria
2. personal history of GDM 4. Strong family history of diabetes
A. only 1 and 3 are correct C. only 1,2, and 3 are correct
B. only 2 and 4 are correct D. 1,2,3 and 4 are correct
______45. This test is used as an index of mean glycemia and as measure of risk for the
development of diabetes complications.
A. Ketone B. GHb C. IFG D. IGT
______46.

E. LIPIDS and LIPOPROTEINS


_____31. Which of the following are true about cholesterol?
1. The sterol ring of the cholesterol molecule can be broken down further
2. The peripherally synthesized or intestinally absorbed cholesterol is transported
to the liver where it is partly converted to bile acids and the remainder
excreted unchanged into the intestine via the bile, which serves as an
emulsifying agent
3. 25 to 40% of the cholesterol in the plasma is present as esterified and the
remaining 60-75% as free “unesterified.”
4. Most of the cholesterol is transported in the plasma via low density lipoprotein
A. only 1 and 3 are correct C. only 1,2, and 3 are correct
B. only 2 and 4 are correct D. 1,2,3 and 4 are correct
_____32. It is generally agreed that coronary heart disease is infrequent at cholesterol
level <160mg/dL and the value that constitutes a threshold value beyond
which the risk of disease rises at first:
A. 200 mg/dL B. 250mg/dL C. 300mg/dL D. 350mg/dL
_____33. In cholesterol assay:
1. serum values are about 3% higher than plasma values
2. a three-minute venous compression can increase cholesterol values by up
to 10%
3. ascorbic acid and methyldopa or metamizol lead to lower cholesterol
values
4. plasma and serum may be stored for cholesterol assay for up to 4 days at
4C.
A. only 1 and 3 are correct C. only 1,2, and 3 are correct
B. only 2 and 4 are correct D. 1,2,3 and 4 are correct
_____34. Which of the following interfering factors result in too low triglyceride values
because the substances can react with H2O2?
1. hemoglobin 2. ascorbic acid 3. heparin 4. bilirubin
A. only 1 and 3 are correct C. only 1,2, and 3 are correct
B. only 2 and 4 are correct D. 1,2,3 and 4 are correct
_____35. Precipitation of VLDL and LDL can be achieved with:
1. phosphotungstic acid/ MgCl2 3. Polyethylene glycol 6000
2. heparin/MnCl2 4. dextran sulfate/ MgCl2
A. only 1 and 3 are correct C. only 1,2, and 3 are correct
B. only 2 and 4 are correct D. 1,2,3 and 4 are correct
_____36. Which of the following equations is the correct Friedewald’s formula?
A. LDL cholesterol = Cholesterol – Triglyceride / 5 – HDL cholesterol(mg/dL)
B. LDL cholesterol = Cholesterol – Triglyceride /10 – HDL cholesterol(mg/dL)
C. LDL cholesterol = Triglyceride -Cholesterol /2.2 – HDL cholesterol(mg/dL)
D. LDL cholesterol = Triglyceride - Cholesterol / 5 – HDL cholesterol(mg/dL)
_____37. This apolipoprotein makes up more than 95% of the protein coat of LDL:
A. apo A-I B. apo A-II C. apo B100 D. apo C
_____38. These are the principal apolipiproteins of HDL:
1. apo A-I 2. apo B100 3. apo A-II 4. apo-B48
A. only 1 and 3 are correct C. only 1,2, and 3 are correct
B. only 2 and 4 are correct D. 1,2,3 and 4 are correct
_____39. This apolipoprotein is the activator of LCAT:
A. apo A-I B. apo A-II C. apo A-III D. apo A-IV
_____40. Which apolipoprotein is associated with Tangier disease?
A. apo A-I B. apo A-II C. apo A-III D. apo A-IV
_____41. Which apolipoprotein is the cofactor of lipoprotein lipase?
A. apo C-I B. apo C-II C. apo C-III D. apo C-IV
_____42. Lp(a) is increased in:
1. nephritic ssyndrome
2. uremic patient undergoing hemodialysis
3. patients with uncontrolled DM
4. patients with hyperthyroidism
A. only 1 and 3 are correct C. only 1,2, and 3 are correct
B. only 2 and 4 are correct D. 1,2,3 and 4 are correct
_____43. Which of the following statements are correct about LDL:
5. LDL are formed b y the hydrolysis of VLDL
6. LDL are mainly responsible for carrying cholesterol to peripheral
cells
7. LDL are the only lipoprotein class possessing a single
apolipoprotein, apo B100.
8. LDL on electropheretic separation migrate in the a-globulin position
A. only 1 and 3 are correct C. only 1,2, and 3 are correct
B. only 2 and 4 are correct D. 1,2,3 and 4 are correct

_____44. Which of the following statements are correct about HDL?


1. HDL are the smallest lipoproteins
2. HDL has the highest protein content
3. HDL float in the ultracentrifuge at a density of 1.063 to 1.21
4. HDL facilitate and accelerate the clearance of cholesterol from cells and
vascular walls and its transport to the liver.
A. only 1 and 3 are correct C. only 1,2, and 3 are correct
B. only 2 and 4 are correct D. 1,2,3 and 4 are correct
_____45. According to Fredrickson classification of primary hyperlipoproteinemias,
which of the following do not match?
A. type I : fat-induced hypertriglyceridemia
B. type II : hypercholesterolemia
C. type III : combined endogenous and exogenous triglyceridemia
D. type IV : endogenous triglyceridemia
_____46. Which of the following serves as the building block for triglyceride and
phospholipids synthesis?
A. acetate B. cholesterol C. amino acids D. fatty acids
_____47. Which of the following lipids function as a surfactant in the lungs?
A. cholesterol B. fatty acids C. phospholipids D. triglycerides
_____48. The secretion of pancreatic lipase into the small intestine is necessary for the
hydrolysis of:
A. cholesterol B. fatty acids C. phospholipids D. triglycerides
_____49. Lipids are transported in the blood by:
A. albumin B. amino acids C. gamma globulin D. lipoproteins
_____50. The lipoprotein class involved in the transport of triglycerides from the small
intestine through the circulation to various tissues is:
A. chylomicron B. VLDL C. LDL D. HDL
_____51. The lipoprotein class associated with the removal of cholesterol from the body
is:
A. chylomicron B. VLDL C. LDL D. HDL
_____52. The lipid component whose measurement would be the most severely affected
by analyzing a non-fasting specimen is:
A. apolipiprotein B. cholesterol C. triglyceride D. fatty acid
_____53. In general, the quantitation of triglyceride is based on measuring:
A. chylomicron B. fatty acids C. glycerol D. triglyceride directly
_____54. In what form must cholesterol be for it to react with cholesterol oxidase in the
enzymatic methods?
A. esterified B. free C. bound to lipoproteins D. bound to albumin
_____55. Which of the following is not a finding in type IIa Fredrickson primary
hyperlipoproteinemias:
A. increased LDL cholesterol
B. turbid appearance of fasting serum
C. often decreased HDL cholesterol
D. normal triglyucerides
_____56. Which lipoprotein shows the following characteristics? A-fraction in
electrophoresis, 50% protein, cholesterol and phospholipids are principal
lipids, produced in the liver and intestine.
A. chylomicron B. VLDL C. LDL D. HDL
_____57. Which lipoprotein is produced only in the intestine?
A. chylomicron B. VLDL C. LDL D. HDL
_____58. Apo A-I is:
1. a cofactor of LCAT
2. a cofactor of LPL
3. structural protein of HDL
4. structural protein of chylomicron
A. only 1 and 3 are correct C. only 1,2, and 3 are correct
B. only 2 and 4 are correct D. 1,2,3 and 4 are correct
_____59. Which of the following methods are used for Lp(a) analysis?
1. immunoassays 3. radial immunodiffusion
2. electroimmunodiffusion 4. immunonephelometry
A. only 1 and 3 are correct C. only 1,2, and 3 are correct
B. only 2 and 4 are correct D. 1,2,3 and 4 are correct
_____60. This apoliprotein is a cofactor for lipoprotein lipase:
A. A-I B. B-100 C. C-II D. D

C. PROTEINS AND AMINO ACIDS


_____61. Which of the following reagents are components of biuret?
1. copper sulfate 3. sodium potassium tartrate
2. potassium iodide 4. sodium hydroxide
A. only 1 and 3 are correct C. only 1,2, and 3 are correct
B. only 2 and 4 are correct D. 1,2,3 and 4 are correct
_____62. Which of the following diseases and conditions may cause hypoproteinemia?
1. severe liver damage 3. glomerulonephritis
2. ulcerative colitis 4. skin diseases
A. only 1 and 3 are correct C. only 1,2, and 3 are correct
B. only 2 and 4 are correct D. 1,2,3 and 4 are correct
_____63. In total protein determination, which of the following statements is correct?
A. The blood should be collected with the patient in a supine position since
blood samples obtained in an upright position lead to measurement of up to
10% lower TP concentration.
B. After more than 3 mins. of venous occlusion during blood collection, the
protein level may rise by up to 10%
C. Due to fibrinogen, the mean total protein concentration in plasma is lower
than in serum.
D. The patient need to be in fasting state for the blood collection.

_____64. Which of the following substances may elevate protein level?


1. hemolysis 3. dextran
2. x-ray contrast media 4. bilirubin
A. only 1 and 3 are correct C. only 1,2, and 3 are correct
B. only 2 and 4 are correct D. 1,2,3 and 4 are correct
_____65. Which of the following diseases and conditions may cause hyperproteinemia?
1. macroglobulinemia 3. chronic inflammatory disease
2. liver cirrhosis 4. Schistosomiasis
A. only 1 and 3 are correct C. only 1,2, and 3 are correct
B. only 2 and 4 are correct D. 1,2,3 and 4 are correct
_____66. All proteins are pure polypeptide chains whose molecular mass is composed of
nitrogen in approximately:
A. 6% B. 16% C. 26% D. 36%
_____67. Which of the following is used as the calibrator in the methods of
determination for total protein?
A. bovine serum albumin C. plasma protein
B. serum protein D. hemoglobin
_____68. These proteins migrate in the alpha1 and alpha2 globulin band and are elevated
by 50-300% in acute inflammatory conditions but are decreased in acute
hepatitis, chronic active liver diseases, and protein-losing syndromes.
A. immunoglobulins C. transerythrin-transferrin group
B. acute phase proteins D. polyclonal gammopathies
_____69. Hypoalbuminemia may be caused by:
1. liver dysfunction 3. nephritic syndromes
2. pregnancy 4. acute phase response
A. only 1 and 3 are correct C. only 1,2, and 3 are correct
B. only 2 and 4 are correct D. 1,2,3 and 4 are correct
_____70. In comparison to upright position, plasma albumin concentration declines by
about 15% after a supine position has been assumed for at least:
A. 5 mins C. 15 mins
B. 10 mins D. 20 mins
_____71. Based on their electrophoretic mobility, which of the following alpha1
antitrypsin variants do not match?
A. F: located near the anode C. S: towards the cathode
B. M: medium mobility D. Z: located near the cathode
_____72. Alpha1 antitrypsin is used in the diagnosis of:
1. prolonged jaundice in neonates
2. hepatitis of uncertain origin during infancy and early childhood
3. pulmonary emphysema in adults
4. hepatitis or liver cirrhosis of uncertain origin in adults
A. only 1 and 3 are correct C. only 1,2, and 3 are correct
B. only 2 and 4 are correct D. 1,2,3 and 4 are correct

_____73. Which of the following anticoagulants may cause falsely reduced alpha1
antitrypsin concentrations in the case of determination by means of radial
immunodiffusion?
1. buffered citrate 3. EDTA
2. potassium oxalate 4. heparin
A. only 1 and 3 are correct C. only 1,2, and 3 are correct
B. only 2 and 4 are correct D. 1,2,3 and 4 are correct
_____74. The Kjeldahl procedure for total protein is based upon the premise that:
A. proteins are negatively charged
B. the pK of proteins is the same
C. the nitrogen content of proteins is constant
D. proteins have similar tyrosine and tryptophan content
_____75. All of the following statements about total protein assays are true, except?
A. refractive index of serum correlates with total protein, but is falsely high in
uremia
B. the Folin-Lowry assay is subject to interference from nonprotein reductants
C. Turbidimetric assays are subject to positive interference from many drugs
D. Direct UV methods are based on the absorbance of peptide bonds at
254nm.
_____76. Hyperalbuminemia is caused by:
A. dehydration syndromes
B. Liver disease
C. Burns
D. Gastroenteropathy
_____77. The term biuret reaction refers to:
A. the reaction of phenolic groups with CuSO4
B. coordinate bonds between Cu and carboxyl and amino groups of biuret
C. the protein error of indicator effect producing color when dyes bind protein
D. the reaction of phosphomolybdic acid with protein.
_____78. Which protein is an acute phase protein and a transport protein. Its function is
to transport intravascular, free hemoglobin to its degradation site in the
reticulo-endothelial system?
A. haptoglobin C. hemopexin
B. ceruloplasmin D. ferritin
_____79. Which protein is important for the estimation of extent of intravascular
hemolysis if serum haptoglobin shows to nonmeasurable levels?
A. hemoglobin C. hemopexin
B. ceruloplasmin D. ferritin
_____80. Which of the following is true about ceruloplasmin?
1. transport copper in plasma
2. deficiency is associated with Wilson’s disease
3. contains about 9% carbohydrate
4. synthesized in the liver
A. only 1 and 3 are correct C. only 1,2, and 3 are correct
B. only 2 and 4 are correct D. 1,2,3 and 4 are correct

KIDNEY FUNCTION TESTS


_____81. This is the gold standard in for the glomerular filtration:
A. urea clearance C. inulin clearance
B. creatinine clearance D. uric acid clearance
_____82. In order to accurately determine the GFR, a substance must meet the following
criteria:
1. free glomerular filtration 3. inert
2. no tubular absorption or secretion 4. non toxic
A. only 1 and 3 are correct C. only 1,2, and 3 are correct
B. only 2 and 4 are correct D. 1,2,3 and 4 are correct
_____83. Concerning glomerular filtration rate, which of the following statements is not
true?
A. In its early stage, type I diabetes mellitus results in a rise of the GRF as well
as the renal plasma flow.
B. Following donation of a kidney, the GFR in the organ donor stabilizes at 70-
80% of the level prior to removal of the kidney.
C. At the beginning of pregnancy the GRF decreases, by the end of the first
trimester it is decreased by 20-30%
D. A protein challenge, an amino acid infusion or the administration of
dopamine cause the GFR to rise.
_____84. Concerning an evaluation of renal function, the following laboratory
investigations are correct for routine clinical diagnostic purposes:
1. serum creatinine; is elevated as of a GFR <50mL/min/1.73m2
2. serum urea; is elevated as of a GFR <30mL/min/1.73m2
3. serum beta-2-microglobulin; a concentration of <2.4mg/L indicates a normal
or only slightly decreased GFR
4. serum cystatin C; the upper reference limit of healthy people is exceeded if
the GFR is <80mL/min/1.73m2
A. only 1 and 3 are correct C. only 1,2, and 3 are correct
B. only 2 and 4 are correct D. 1,2,3 and 4 are correct
_____85. The approved reference method for creatinine determination is the:
A. isocratic HLPC C. Jaffe method
B. Fuller’s earth method D. Kinetic method
_____86. The low analytical specificity in creatinine determination is due to non-
creatinine chromogens. Which of the following statements is true regarding
non-creatinine chromogens?
1. Non-creatinine chromogens form a reaction product with alkaline picrate
similar to that produced by creatinine.
2. During the early reaction phase, rapid reacting substances such as acetic
acid contribute towards the color reaction.
3. During the middle phase reaction, the color reaction is almost exclusively
due to creatinine.
4. During the late phase, glucose influence the color reaction.
A. only 1 and 3 are correct C. only 1,2, and 3 are correct
B. only 2 and 4 are correct D. 1,2,3 and 4 are correct
_____87. In the enzymatic method for creatinine using phenol-aminophenazone
peroxidase, creatinine is hydrolyzed to creatine by creatininase. In a further
reaction step, both creatine produced from creatinine as well as endogenous
creatine are degraded by creatinase into:
1. glycine 2. urea 3. H2O2 4. sarcosine
A. only 1 and 3 are correct C. only 1,2, and 3 are correct
B. only 2 and 4 are correct D. 1,2,3 and 4 are correct
_____88. In uric acid methodology, the transformation of uric acid to allantoin and
hydrogen peroxide by uricase in the presence of ambient air oxygen is used as
the first reaction. For routine diagnositic purposes, which of the following
enzymes are used to produce chromogen in the secondary reaction?
1. uricase-catalase reaction 3. peroxidase
2. aldehyde dehydrogenase 4. uricase
A. only 1 and 3 are correct C. only 1, 2, and 3 are correct
B. only 2 and 4 are correct D. 1, 2, 3 and 4 are correct
_____89. By definition hyperuricemia is present if the serum uric acid concentration is >
6.5mg/dL. This definition is based on physico-chemical factors taking into
consideration that the solutbility of sodium urate at 370C is:
A. 6.4 mg/dL B. 6.0 mg/dL C. 6.6 mg/dL D. 6.5 mg/dL
_____90. Which of the following substances may cause decreased levels due to
inhibition of uricase?
1. citrate 2. oxalate 3. formaldehyde 4. oxalic acid
A. only 1 and 3 are correct C. only 1,2, and 3 are correct
B. only 2 and 4 are correct D. 1,2,3 and 4 are correct
_____91. Which of the following statements regarding uric acid determination is
correct?
A. Blood collection can be performed even without fasting as long as it is
done in the morning.
B. Circadian rhythm does exist for the uric acid concentration, variations are
dependent on time of the day.
C. Blood collection performed after heavy muscular activity may result to a
decrease in concentration
D. After intensive sun exposure, blood collection may increases the result.
_____92. The urea content of the blood is derived from the blood urea nitrogen level by
multiplying the latter by a factor of:
A. 0.46 B. 2.14 C. 0.64 D. 2.41
_____93. These reagents are added to stabilize the color formed by the condensation of
diacetyl with urea to form the chromogen diazine in the diacetyl monoxime
method:
1. thiosemicarbazide 3. Fe
2. 2-oxoglutarate 4. ammonium heparinate
A. only 1 and 3 are correct C. only 1,2, and 3 are correct
B. only 2 and 4 are correct D. 1,2,3 and 4 are correct
_____94. For the differentiation of renal acute renal failure from renal acute renal failure
from prerenal acute renal failure and postrenal acute renal failure, the
urea/creatinine ratio can be used as a diagnostic criterion in addition to the
clinical symptoms. Which of the following statements are correct when a
normal protein intake of healthy people of approximately 1g/kg body
weight/day is true?
1. 35, if both are determined in mmol/L
2. 25, if the determination is performed in mg/dL
3. 12, if urea is determined as BUN and if both are determined in mg/dL
4. 15, if urea is determined as BUN and if both are determined in mmol/L
A. only 1 and 3 are correct C. only 1,2, and 3 are correct
B. only 2 and 4 are correct D. 1,2,3 and 4 are correct
_____95. The diacetyl monoxime method is relatively unspecific. Which of the
following substances are measured as well?
1. creatinine 2. allantoin 3. arginine 4. glucose
A. only 1 and 3 are correct C. only 1,2, and 3 are correct
B. only 2 and 4 are correct D. 1,2,3 and 4 are correct

CLINICAL ENZYMOLOGY
_____96. Which of the following enzymes has the longest half-life?
A. CK-MM B. ALP C. Lipase D. LD5
_____97. Which of the following CK isoenzymes has the longest half-life?
A. CK-MM B. CK-MB C. CK-BB D. CK4
_____98. In enzymology, one international unit is the quantity of enzyme which
catalyzes one micromole of substrate per minute. The catalytic enzyme
activity of a sample is expressed in:
A. Units per liter C. kilounits per liter
B. Milliunits per liter D. all of these
______99. Type 1 macroenzymes have been described for the following diagnostically
relevant enzymes, except:
1. ALT 2. ALP 3. ACP 4. AST

A. only 1 and 3 are correct C. only 1,2, and 3 are correct


B. only 2 and 4 are correct D. 1,2,3 and 4 are correct
_____100. These are individual-specific enzyme forms which are coded by allelic genes.
The multiple forms of an enzyme are inherited according to Mendelian laws.
A. Isoenzymes C. Alleloenzymes
B. Macroenzymes type 1 D. Macroenzymes type 2
_____101. Enzyme important in the diagnosis of organophosphate poisoning and liver
parenchymal damage:
A. LD B. Cholinesterase C. CK D. GGT
_____102. Enzyme important for the diagnosis of liver parenchymal damage,
myocardial infarction, hemolysis, ineffective erythropoiesis, lymphoma:
A. ALT B. AST C. LD D. CK
_____103. Which of the following enzyme patterns is correct in severe cardiac muscle
damage with cell necrosis:
A. CK>LDH>AST>ALT C. AST>ALT>CK>LDH
B. LDH>CK>ALT>AST D. ALT>CK>LDH>AST
_____104. Which of the following enzyme patterns is correct in severe skeletal muscle
damage with cell necrosis:
A. CK>LDH>AST>ALT C. AST>ALT>CK>LDH
B. LDH>CK>ALT>AST D. ALT>CK>LDH>AST
_____105. Which of the following enzyme ratios is used for the differentiation between
acute alcoholic hepatitis and acute viral hepatitis:
A. GGT/AST C. AST/ALT
B. LD/CK D. GGT/GLD
_____106. The increase in ALP is particularly marked in people of Lewis-positive blood
group:
A. A B. B C. AB D. O
_____107. Which enzyme decreases markedly with age in men while increases with age
are measured in women:
A. ALP B. ALT C. AST D. CK
_____108. Alcoholism leads to increased activities of:
1. GGT 2. ALT 3. AST 4. GLD
A. only 1 and 3 are correct C. only 1,2, and 3 are correct
B. only 2 and 4 are correct D. 1,2,3 and 4 are correct
_____109. Which of the following diseases can cause a decrease in total ALP?
1. Pituitary dwarfism 3. Malnutrition
2. Liver metastases 4. Rickets
A. only 1 and 3 are correct C. only 1,2, and 3 are correct
B. only 2 and 4 are correct D. 1,2,3 and 4 are correct
_____110. Which of the following migration patterns is correct in the anodal separation
of serum ALP at an alkaline pH on cellulose acetate strip?
A. placental ALP>liver ALP>bone ALP>intestinal ALP>bile duct ALP
B. bile duct ALP>liver ALP> bone ALP> intestinal ALP> placental ALP
C. liver ALP>bone ALP> bile duct ALP> intestinal ALP> placental ALP
D. intestinal ALP>liver ALP>placental ALP> bile duct ALP> bone ALP
_____111. Which form of ALP can be differentiated on the basis of the greater inhibition
of its activity by L-phenylalanine, lesser inhibition by homoarginine and its
resistance to heat when heated at 65C for 10minutes?
A. Placental B. liver C. bone D. bile duct
_____112. This form of ALP is thought to be a multienzyme plasma membrane complex
or a complex of the liver isoenzyme with lipoprotein X. This multiple form
of ALP is found in cholestatic and metastatic liver disease even when total
ALP is still normal.
A. Liver ALP C. High-molecular mass ALP
B. Germ cell ALP D. intestinal ALP
_____113. This form of ALP is not normally found in the serum of healthy people and
detectable in patients with seminoma of the testes, ovarian carcinoma,
pituitary tumors and thymic tumors.
A. Liver ALP C. High-molecular mass ALP
B. Germ cell ALP D. intestinal ALP
_____114. This form of ALP is produced by the enterocytes and released into the blood
in large quantities, particularly in B and O secretors.
A. Liver ALP C. High-molecular mass ALP
B. Germ cell ALP D. intestinal ALP
_____115. This is a multiple ALP form which is biochemically a heterodimer of
placental ALP and intestinal ALP. It is found in hepatocellular carcinoma
and renal cell carcinoma.
A. Placental ALP C. Bone ALP
B. Kasahara ALP D. germ cell ALP
_____116. Which of the following are inhibitors of ALP?
1. Citrate 2. Oxalate 3. EDTA 4. Heparin
A. only 1 and 3 are correct C. only 1,2, and 3 are correct
B. only 2 and 4 are correct D. 1,2,3 and 4 are correct
_____117. Which of the following drugs can decrease ALP activity?
A. Oral contraceptives C. Valproic acid
B. Allopurinol D. oxacillin
_____118. In amylase assays, which of the following are measured as end products?
1. 2-chloro-4-nitrophenol 3. 4-nitrophenol
2. P-nitrophenyl maltoheptaoside 4. Maltotetraose
A. only 1 and 3 are correct C. only 1,2, and 3 are correct
B. only 2 and 4 are correct D. 1,2,3 and 4 are correct
_____119. In neonates, the amylase measured consists only of:
A. Salivary isoenzyme C. both salivary and pancreatic
B. Pancreatic isoenzyme D. none of these

_____120. Which of the following methods are used for ACE measurement?
1. Liberman 2. Neels 3. Ryan 4. Silverstein
A. only 1 and 3 are correct C. only 1,2, and 3 are correct
B. only 2 and 4 are correct D. 1,2,3 and 4 are correct
_____121. Which of the following substrates are used in ACE measurement?
1. Benzoylglycine 3. Furylacrylic acid
2. Hippuryl-histidyl-lecuine 4. Benzoyl-glycyl-glycine
A. only 1 and 3 are correct C. only 1,2, and 3 are correct
B. only 2 and 4 are correct D. 1,2,3 and 4 are correct
_____122. An increased ACE activity is seen in:
A. Sarcoidosis C. Endothelial dysfunction
B. Toxic lung damage D. Hypothyroidism
_____123. Which of the following can reduce/inhibit ACE activity?
1. EDTA 2. Captopril 3. Enalapril 4. Zinc chelators
A. only 1 and 3 are correct C. only 1,2, and 3 are correct
B. only 2 and 4 are correct D. 1,2,3 and 4 are correct
_____124. What is the EC number of AST?
A. 2.6.1.1 B. 2.6.1.2 C. 3.1.1.7 D. 3.1.1.6
_____125. De Ritis ratio for differentiation between mild liver damage and severe liver
disease is:
A. GGT/AST B. AST/ALT C. AST/LD D. LD/GGT
_____126. This ratio is used as indicator of cholestasis and alcoholic liver damage.
A. GGT/AST B. AST/ALT C. AST/LD D. LD/GGT
_____127. Which of the following methods are used for cholinesterase determination?
1. Inhibition test with dibucaine 3. Ro 2-0683
2. Determination of fluoride number 4. Photometric rate assay
A. only 1 and 3 are correct C. only 1,2, and 3 are correct
B. only 2 and 4 are correct D. 1,2,3 and 4 are correct
_____128. Which of the following chemicals is/are associated with decrease in
cholinesterase activity?
A. Muscle relaxants C. Carbamate esters
B. Organophosphate esters D. all of these
_____129. Determination of acylcholine acylhydrolase for evaluation of liver function is
usually performed using what substrate?
A. Glycine C. choline
B. Phosphate D. nitroanilide
_____130. CK catalyzes the reversible transfer of the phosphate group from creatine
phosphate to Mg-ADP. The resulting Mg-ATP is determined in a combined
optical test using what as coenzyme?
A. Glucose-6-phosphate dehydrogenase C. Hexokinase
B. Oxidase D. Adenylate kinase
_____131. A CK-MB fraction of more than 6% of the total CK activity is regarded as
diagnostic for:
A. Myocardial infarction C. Skeletal muscle damage
B. Muscular dystrophy D. Secondary myopathies
_____132. Which of the following findings can indicate the presence of macro CK?
A. Elevated total CK with a CK-MB fraction >25%
B. Elevated total CK with a CK-BB fraction >6%
C. Elevated total CK with a CK-MM fraction >5%
D. Elevated total CK with a CK-MiMi fraction >10%
_____133. The variant of CK with higher molecular mass formed when CK is bound by
specific immunoglobulins:
A. Macro CK type 1 C. CK-MiMi
B. Macro CK type 2 D. CK-MT or CKMito
_____134. Which enzyme is specific for liver and bile duct and an increase of this
enzyme is considered as one of the most sensitive indicators of hepatobiliary
disease?
A. Cholinesterase C. CK
B. GGT D. AST
_____135. Differentiation between jaundice of hepatic or cholestatic origin can be based
on ALT activity and the:
A. AST/ALT ratio C. GGT/ALT ratio
B. GGT/AST ratio D. ALT/ALP ratio
_____136. Which enzyme is used with aminotransferases and mean cellular volume of
erythrocytes as indicators of excessive alcohol consumption?
A. Cholinesterase C. GLD
B. GGT D. AST
_____137. Which of the following anticoagulants may lead to falsely low GGT activity?
1. Citrate 2. Oxalate 3. Flouride 4. Heparin
A. only 1 and 3 are correct C. only 1,2, and 3 are correct
B. only 2 and 4 are correct D. 1,2,3 and 4 are correct
_____138. This is the only enzyme which cleaves significant amount of gluthathione.
A. Cholinesterase C. GLD
B. GGT D. AST
_____139. A (ALT + AST)/GLD ratio of <20 is seen in:
1. Obstructive jaundice 3. Metastatic liver disease
2. Biliary cirrhosis 4. Acute hypoxic liver damage
A. only 1 and 3 are correct C. only 1,2, and 3 are correct
B. only 2 and 4 are correct D. 1,2,3 and 4 are correct
_____140. A (ALT + AST)/GLD ratio of >50 is seen in:
1. Acute episodes of chronic liver diseases
2. Acute viral hepatitis
3. Cholestatic hepatic diseases
4. Acute alcoholic hepatitis
A. only 1 and 3 are correct C. only 1,2, and 3 are correct
B. only 2 and 4 are correct D. 1,2,3 and 4 are correct
_____141. These reagents can be added to the LD assay mixture as a selective inhibitor
of the LD isoenzymes containing M sub-units so that only LD-1, which is
composed of four H units, is measured.
1. Nitroanilide 3. Ro-0683
2. 1,6 hexanediol 4. Sodium perchlorate
A. only 1 and 3 are correct C. only 1,2, and 3 are correct
B. only 2 and 4 are correct D. 1,2,3 and 4 are correct
_____142. Which LD isoenzyme has the highest migration rate?
A. LD1 B. LD2 C. LD4 D. LD5
_____143. In agarose gel, this Ld isoenzyme migrates to the cathode:
A. LD1 B. LD2 C. LD4 D. LD5
_____144. In cellulose acetate electrophoresis the LD isoenzyme fractions are made
visible by coupling the enzymatically formed pyruvate with:
A. NAD C. tetrazolium salt
B. NADH D. Sodium perchlorate
_____145. This ratio is used to differentiate between prehepatic jaundice caused by
hemolysis or dyserythropoiesis from hepatic jaundice.
A. ALT/AST ratio C. LD/CK ratio
B. LD/AST ratio D. CK/AST ration
_____146. Anodic LD isoenzyme pattern is associated with:
1. Muscular dystrophy 3. Renal infarction
2. Germ cell tumor 4. Cardiac muscle damage
A. only 1 and 3 are correct C. only 1,2, and 3 are correct
B. only 2 and 4 are correct D. 1,2,3 and 4 are correct
_____147. Which anticoagulant should not be used in LD assay?
A. Oxalate B. Flouride C. both D. neither
_____148. Which of the following conditions can falsely increase LD measurement?
A. Blood sample collection after exercise
B. Use of capillary serum
C. Use of hemolyzed samples
D. All of these
_____149. At a mean activity of 165U/L, hemolysis of 0.8g hb/L leads to an LD increase
of:
A. 36% B. 58% C. 360% D. 12%
_____150. Which of the following Ld isoenzymes is not seen mainly in Duchenne
muscular dystrophy?
A. LD1 B. LD2 C. LD3 D. LD5
_____151. The most reliable method for lipase determination is the kinetic, automatic
titration of oleic acid released from an emulsion of triolein or pure olive oil
by enzymatic hydrolysis with sodium hydroxide at what pH?
A. 8.6 B. 6.5 C. 9.0 D. 8.4
_____152. As ACP is released from erythrocytes, a hemoglobin concentration of 3.4g/L
in serum may increase ACP by:
A. 50% B. 40% C. 30% D. 20%
_____153. ACP is unstable at pH above 7.0 and the pH of plasma or serum is adjusted to
4-5 by the addition of:
A. 10% acetic acid(20uL/mL) C. both
B. Sodium bisulfate 5mg/mL serum D. neither
_____154. Which of the following diseases are associated with elevations of ACP in
serum?
1. Prostatic carcinoma 3. Paget’s disease
2. Multiple myeloma 4. Gaucher’s disease
A. only 1 and 3 are correct C. only 1,2, and 3 are correct
B. only 2 and 4 are correct D. 1,2,3 and 4 are correct
_____155. Without acidification, ACP activity is decreased up to:
A. 20% in 3h C. 50% in 2h
B. 20% in 2h D. 5% in 2 h
_____156. This protein participates in the uptake and the intracellular transport of the
long-chain fatty acids in the cardiomyocytes.
A. Cardiac troponin (cTnT) C. myoglobin
B. Heart fatty acid binding protein(H-FABP) D. natriuretic peptide
_____157. This myofibrillar protein of the heart muscle is released from the myocardium
following injury and is used in the late diagnosis and monitoring of the
course of acute myocardial infarction.
A. Cardiac troponin (cTnT) C. myoglobin
B. Heart fatty acid binding protein(H-FABP) D. natriuretic peptide
_____158. This is a specific cardiospecific marker used to assess chronic or subacute
graft rejection after heart transplantation.
A. Cardiac troponin (cTnT)
B. cardiac troponin I (cTnI)
C. Heart fatty acid binding protein(H-FABP)
D. glycoprotein phosphorylase isoenzyme BB (GPBB)
_____159. This is highly sensitive to ischemia of cardiomyocytes so that the clinical
sensitivity of its measurement for diagnosing an acute coronary syndrome is
higher than that of other cardiac markers.
A. Cardiac troponin (cTnT)
B. cardiac troponin I (cTnI)
C. Heart fatty acid binding protein(H-FABP)
D. glycoprotein phosphorylase isoenzyme BB (GPBB)
______160. This is the most important natriuretic peptide of the ventricles and it is a
more sensitive and more specific indicator of ventricular overload.
A. Atrial natriuretic peptide C. brain natriuretic peptide
B. Cyclic guanosine monophosphate D. CNP

BILIRUBIN

_____161. In this method of bilirubin determination, unconjugated and conjugated


bilirubin interact with a specific charged polymer called mordant and the
concentrations of Bu and Bc are calculated from the measured reflection
densities and the predetermined molar reflectivities of the two bilirubin
species at two wavelengths and use of simultaneous equations.
A. Jendrassik-Grof C. Multi-layer film slide technology
B. Enzymatic method D. Direct spectrophotometry
_____162. This enzyme is used to oxidize bilirubin to biliverdin:
A. Bilirubin reductase C. Bilirubin oxidase
B. Bilirubin diglucoronidase D. Bilirubin dehydrogenase
_____163. Which of the following statements about unconjugated bilirubin is not
correct?
A. It is practically insoluble in water at physiologic pH and body
temperature
B. In plasma, it is present in a folded structure or so-called Z-Z(trans)
conformation loosely bound to albumin
C. It is transported in plasma loosely bound to albumin
D. It is covalently bound to albumin
_____164. Which measurement is a better criterion for the differential diagnosis of
jaundice?
A. Conjugated bilirubin C. Delta bilirubin
B. Unconjugated bilirubin D. Total bilirubin
_____165. In adults and older children, hyperbilirubinemia causes jaundice if bilirubin
value is >2.5mg/dL, whereas in infants, jaundice is seen if bilirubin values is
greater than:
A. 2.5mg/dL B. 3.0mg/dL C. 3.5mg/dL D. 4.0mg/dL
_____166. What classification of hyperbilirubinemia is caused by ineffective
erythropoiesis?
A. Prehepatic jaundice C. Posthepatic jaundice
B. Intrahepatic jaundice D. none of these
_____167. What classification of hyperbilirubinemia is caused by medication-induced
parenchymal and cholestatic liver damage as well as hepatic involvement in
other primary diseases?
A. Prehepatic jaundice C. Posthepatic jaundice
B. Intrahepatic jaundice D. none of these
_____168. Which of the following conditions are associated with unconjugated
hyperbilirubinemias?
1. Crigler-Najjar syndrome 3. Gilbert’s syndrome
2. Dubin-Johnson syndrome 4. Rotor syndrome
A. only 1 and 3 are correct C. only 1,2, and 3 are correct
B. only 2 and 4 are correct D. 1,2,3 and 4 are correct
_____169. Which of the following conditions are associated with conjugated
hyperbilirubinemias?
1. Crigler-Najjar syndrome 3. Gilbert’s syndrome
2. Dubin-Johnson syndrome 4. Rotor syndrome
A. only 1 and 3 are correct C. only 1,2, and 3 are correct
B. only 2 and 4 are correct D. 1,2,3 and 4 are correct
_____170. Which of the following statements about bilirubin is true?
A. A decline in unconjugated bilirubin is a sensitive indicator of recovery
B. An increase in delta bilirubin suggests a protracted disease course
C. An increase of conjugated bilirubin >10% of total bilirubin suggest a
hepatogenic cause
D. In the differentiation of prehepatic to hepatic jaundice, a LD/AST ratio
of >2 is suggestive of hemolytic jaundice.
_____171. Per gram of degraded hemoglobin, how much bilirubin is produced?
A. 3mg B. 15mg C. 34mg D. 51mg
_____172. Approximately how much bilirubin are synthesized daily due to physiological
breakdown of blood?
A. 100mg B. 150mg C. 200mg D. 250mg
_____173. The normal daily urobilinogen excretion in the urine is:
A. 1-2mg B. 2-4mg C. 5-10mg D. 1gm
_____174. What is the half-life of delta bilirubin?
A. 2 days B. 8 days C. 12 days D. 18 days
_____175. This is used to induce a rise of unconjugated bilirubin by activating
hemoxygenase and increase spleenic unconjugated bilirubin production.
A. Nicotinic acid C. sulfobromophthalein
B. Trichloroethylene D. dichlorobenzol
_____176. This is a laboratory evaluation system for severity of cirrhosis.
A. Sulfobromophthalein C. Nicotinic acid test
B. Child-Pugh D. Fasting
_____177. This chronic form of conjugated hyperbilirubinemias is characterized by
deficient leukotriene elimination into the bile and therefore leukotrienes are
excreted renally.
A. Crigler-Najjar syndrome C. Gilbert’s syndrome
B. Dubin-Johnson syndrome D. Rotor syndrome
_____178. In case of intensive sun irradiation, bilirubin values decreases by up to:
A. 10% after 1 hour C. 30% after 1 hour
B. 20% after 1 hour d. 50% after 1 hour
_____179. Bilirubin at room temperature, protected from light exposure is stable for:
A. 1 day B. 2 days C. 3 days D. 4 days
_____180. Which bilirubin is measured to assess success following invasive measures to
remove cholestasis?
A. Conjugated bilirubin C. Delta bilirubin
B. Unconjugated bilirubin D. Total bilirubin

ELECTROLYTES

_____181. Which of the following statements about serum calcium is not correct?
A. Free or ionized calcium accounts for 50% of total calcium
B. Protein bound calcium most of which is bound to globulins with only
small portion bound to albumin.
C. Complex-bound calcium is bound to phosphates, citrate and
bicarbonate.
D. The protein-bound calcium accounts for 45% of the total calcium.
_____182. Acidosis or a pH decrease, after the blood sample has been collected, causes a
rise in ionized calcium because of the metabolic activity of the blood cells, a
pH decrease by 0.1 leads to a reciprocal increase of ionized calcium by
approximately:
A. 0.05mmol/L C. 0.15mmol/L
B. 0.10mmol/L D. 0.20mmol/L

_____183. Alkalosis or a rise in pH, after the blood collection, causes a decrease in
ionized calcium due to the elimination of:
A. Oxygen C. Carbon dioxide
B. Hydrogen D. Chloride
_____184. This term refers to a state with serum total calcium concentration of typically
>14 mg/dL that is associated with symptoms such as volume depletion,
metabolic encephalopathy and gastrointestinal symptoms. As soon as the
hypercalcemia has been eliminated, the symptoms disappear.
A. Primary hyperparathyroidism
B. familial hypocalcemuric hypercalcemia
C. hypercalcemic crisis
D. Milk-alkali syndrome
_____185. Pseudohypoparathyroidism represents a group of states with end-organ
resistance to PTH. Which of the following statements about
pseudohypoparathyroidism is not correct?
A. In type 1a, the gene that encodes the G protein is defective
B. In type 1b, the PTH receptor is defective.
C. In type 2, the urinary excretion of cAMP is increased in these patients.
D. Type 1 is characterized by a defect at a locus before the synthesis of
cAMP while type 2 is associated with a defect at a locus beyond the
synthesis of cAMP.
_____186. Which of the following diseases may cause hypercalcemia?
A. Liver cirrhosis C. chornic renal failure
B. Addison’s disease D. acute pancreatitis
_____187. Which of the following diseases may cause hypocalcemia?
A. Pseudohypoparathyroidism C. Sarcoidosis
B. Multiple myeloma D. Primary hyperparathyroidism
_____188. Which formula is correct when adjusting total calcium if the albumin
concentration is 4g/dL?
A. Corrected calcium(mg/dL) = measured Ca (mg/dL)-albumin(g/dL) + 4.0
B. Corrected calcium(mg/dL) = albumin(g/dL) - measured Ca (mg/dL) + 4.0
C. Corrected calcium(mg/dL) = measured Ca (mg/dL)+albumin(g/dL) / 4.0
D. Corrected calcium(mg/dL) = measured Ca (mg/dL)-albumin(g/dL) / 4.0
_____189. How much increase in total calcium is anticipated in the case of a change
from supine to a standing position?
A. 4.6% B. 1.7% C. 12% D. 5.4%
_____190. How much increase in ionized calcium is anticipated in the case of a change
from supine to a standing position?
A. 4.6% B. 1.7% C. 12% D. 5.4%
_____191. Which of the following enzymes are used for phosphate measurement?
1. Purine-nucleoside phosphorylase 3. Sucrose phosphorylase
2. Xanthine oxidase 4. Phosphoglucomutase

A. only 1 and 3 are correct C. only 1,2, and 3 are correct


B. only 2 and 4 are correct D. 1,2,3 and 4 are correct

_____192. What is the molar ratio of HPO4/H2PO4?


A. 1:1 B. 1:2 C. 1:3 D. 1:4
_____193. Which of the following conditions are associated with hypophosphatemia?
1. Bodybuilding 3. Oncogenic osteomalacia
2. Alcoholism 4. Diabetic ketoacidosis

A. only 1 and 3 are correct C. only 1,2, and 3 are correct


B. only 2 and 4 are correct D. 1,2,3 and 4 are correct
_____194. In the phosphate clearance test, the interval time for collection of two urine
and blood samples is:
A. 30 mins B. 60 mins C. 90 mins D. 120 mins
_____195. The percentage tubular reabsorption of phosphate, TRF%, is a test identical to
the determination of the phosphate clearance but in addition this substance is
measured in urine and serum:
A. Inulin B. Urea C. Creatinine D. Macroglobulin
_____196. This is also called the renal phosphorous threshold which describes the
maximal phosphate concentration in the glomerular filtrate below which all
of the filtered phosphate is reabsorbed in the tubules.
A. TRF% C. TmP
B. Ccr D. Cp
_____197. Transcellular fluid constitutes how much percentage of the total body water?
A. 7.5% B. 15% C. 20% D. 45%
_____198. Interstitial fluid constitutes how much percentage of the total body water?
A. 7.5% B. 15% C. 20% D. 45%
_____199. The extracellular fluid volume is directly dependent on total body:
A. Sodium B. Phosphate C. Calcium D. Potassium
_____200. The regulatory systems of the water balance between the ICF and ECF also
referred to as tonicity are:
1. ADH concentration in plasma 3. Renal capacity
2. Thirst and water intake 4. Urea concentration in plasma

A. only 1 and 3 are correct C. only 1,2, and 3 are correct


B. only 2 and 4 are correct D. 1,2,3 and 4 are correct
_____201. An ECFV decline by >5% causes a reversible decrease in renal blood flow.
This results in:
1. The reduction of GFR
2. Increase in the secretion of ADH
3. The activation of the rennin-angiotensin-aldosterone system
4. A rise in the filtration fraction.
A. only 1 and 3 are correct C. only 1,2, and 3 are correct
B. only 2 and 4 are correct D. 1,2,3 and 4 are correct
_____202. Prerenal acute renal failure is associated with:
1. Oliguria and a urine osmolality of >600mosmol/kg water
2. A disproportional increase of urea as compared to creatinine in plasma
3. A urinary sodium concentration of <10mmol/L
4. A reduction of the fractional sodium excretion to <1%
A. only 1 and 3 are correct C. only 1,2, and 3 are correct
B. only 2 and 4 are correct D. 1,2,3 and 4 are correct
_____203. The reference method for sodium and potassium ions:
A. Flame photometry C. Ion Selective Electrode
B. Potentiometry D. Enzymatic-spectrophotometric
_____204. Which of the following conditions is not associated with hyponatremia?
A. Renal tubular acidosis C. Salt-losing nephritis
B. Congestive heart failure D. Diabetes insipidus
_____205. Which of the following conditions is associated with hyponatremia with an
ECFV excess and high total body sodium?
A. Liver cirrhosis C. Hypothyroidism
B. Pancreatitis D. Salt-losing nephritis
_____206. The end color in mercurimetric titration method for chloride using
diphenylcarbazone as indicator.
A. Green C. Purple
B. Blue D. Red
_____207. The chloride ions react with mercuric thiocyanate which together with iron
ions forms what color of complex products?
A. Green C. Purple
B. Blue D. Red
_____208. The anion gap in the plasma or serum refers to the difference between the
most prevalent cations and anions. It is calculated according to this equation:
A. Anion gap = sodium – chloride – bicarbonate
B. Anion gap = potassium + sodium – chloride
C. Anion gap = sodium – potassium – chloride
D. Anion gap = sodium – chloride
_____209. The sodium concentration in erythrocytes is one tenth of that in plasma.
Given, a preexisting sodium concentration of 140 mmol/L, a hemolysis of
5g/L cause a decrease in concentration of:
A. 0.2% C. 0.5%
B. 0.4% D. 1.2%
_____210. What is the end color in the determination of chloride using mercury
thiocyanate?
A. Purple C. red
B. Yellow D. blue
_____211. Bartter’s symdrome is a congenital disorder wherein there is a defect in
chloride absorption in the cortical part of the ascending limb of the loop of
Henle and is characterized by the presence of:
A. Metabolic acidosis C. hyporeninemic
B. Hypokalemia D. hypoaldosteronism
_____212. Which of the following renal tubular acidosis type and characteristic do not
match?
A. Type 1 RTA: distal renal tubular acidosis
B. Type 2 RTA: proximal renal tubular acidosis
C. Type 3 RTA: impaired reabsorption of bicarbonate in the proximal tubule
D. Type 4 RTA: found in patients with progressive renal failure
______213. In adults, polyuria is associated with a urine volume >2.5 to 3L/24 hour or
an excretion rate of greater than:
A. 5 mL per min C. 10 mL per min
B. 2 mL per min D. 0.5 mL per min
______214. In conditions with hyponatremia, the renal sodium excretion as well as the
chloride excretion are a measure for the ECFV status. Accordingly:
A. Patients with hyponatremia and a decreased ECFV have renal sodium
excretion of <20 mmol/L
B. Patients with hyponatremia and a normal ECFV have a renal sodium
excretion of >20 mmol/L
C. Renally induced sodium losses are associated with excretion of >20
mmol/L.
D. All of these are correct
______215. This is a test for the determination of the excreted fraction of glomerularly
filtered sodium and is a measure for the tubular absorption of sodium and an
effective test for the differential diagnosis of acute, oliguric renal failure.
A. Anion gap C. urine pH
B. Fractional excretion of sodium D. Sodium-selective ISE
______216. In hyperkalemia, a potassium excretion:
A. > 40 mmol/L is indicative of a renal cause
B. < 40 mmol/L is indicative of extrarenal cause
C. > 40 mmol/L is indicative of extrarenal cause
D. < 40 mmol/L is indicative of renal and extrarenal cause
______217. Which of the following is associated with Type IV RTA?
A. Plasma potassium: > 5.5, anion gap in urine positive, urine pH: <5.5
B. Plasma potassium: > normal, anion gap in urine positive, urine pH:
>5.5
C. Plasma potassium: normal, anion gap in urine negative, urine pH: >5.5
D. Plasma potassium: normal, anion gap in urine positive, urine pH: <5.5
______218. Which of the following is associated with type 1 RTA?
A. Plasma potassium: > 5.5, anion gap in urine positive, urine pH: <5.5
B. Plasma potassium: > normal, anion gap in urine positive, urine pH:
>5.5
C. Plasma potassium: normal, anion gap in urine negative, urine pH: >5.5
D. Plasma potassium: normal, anion gap in urine positive, urine pH: <5.5
______219. Which of the following is associated with gastrointestinal bicarbonate loss
and high urinary ammonia excretion?
A. Plasma potassium: > 5.5, anion gap in urine positive, urine pH: <5.5
B. Plasma potassium: > normal, anion gap in urine positive, urine pH:
>5.5
C. Plasma potassium: normal, anion gap in urine negative, urine pH: >5.5
D. Plasma potassium: normal, anion gap in urine positive, urine pH: <5.5
______220. Which of the following is associated with interstitial renal disease?
A. Plasma potassium: > 5.5, anion gap in urine positive, urine pH: <5.5
B. Plasma potassium: > normal, anion gap in urine positive, urine pH:
>5.5
C. Plasma potassium: normal, anion gap in urine negative, urine pH: >5.5
D. Plasma potassium: normal, anion gap in urine positive, urine pH: <5.5

BLOOD GASES

_____221. This parameter is defined as the amount of strong acid or strong base needed
to titrate extracellular fluid to pH 7.4 at a pCO2 of 40mmHg and at 37C.
A. Standard bicarbonate C. Base excess
B. cHCO3 D. anion gap
_____222.
1. Incorrect statement about naturally occurring c. Doa, Dob
antibodies d. Bga, Bgb, Bgc
a. React best in saline
b. React best at room temperature or at 8. Which of the following does not belong to the
40C high titer low avidity antibodies?
c. They are usually IgG a. Anti-Ch
d. Do not usually cross the placenta b. Anti-Yka
c. Anti-Co
2. Which antibody is most likely to be involved d. Anti-JMH
in hemolytic disease of the Newborn?
a. anti-E 9. Incorrect statement about Kell blood group
b. AntiLea a. 90% of the population is kell negative
c. Anti-M b. Anti-K reacts best in saline at room
d. Anti-Jsa temperature
c. Anti-K can cause hemolytic disease of
3. What blood type is not possible for an the newborn
offspring of AO and BO persons? d. Next to Rh (D), the Kell (K) antigen is
a. AB the most immunogenic
b. A or B
c. O
d. All are possible 10. Adults who have the Le, Se, and H genes will
exhibit which Lewis antigen on their red
4. Nature of ABO antibodies cells?
a. IgM a. Lea
b. IgG and IgM b. Leb
c. IgA c. Lea and Leb
d. IgG, IgM, and IgA d. None

5. Waldenstrom’s macroglobulinemia, multiple 11. Given the following reactions, identify the
myeloma, and increased levels of fibrinogen cause of discrepancy:
is associated with what ABO discrepancy? Cell typing: Serum typing:
a. Group I Anti-A = 4+ A1 cells = 0
b. Group II Anti-B = 2+ B cells = 4+
c. Group III a. Hypogammaglobulinemia
d. Group IV b. Hodkin’s lymphoma
c. Cold agglutinins
6. In which of the following blood group systems d. Acquired B phenomenon
may the RBC typing change during
pregnancy? 12. Which of the following is a contraindication
a. Kidd for accepting a donor for plateletpheresis?
b. Lewis a. 72 hours since the last apheresis
c. MNS b. Plasmapheresis of 800mL 1 week ago
d. Kell c. Platelet count of 100 x 109/L
d. Aspirin ingestion 7 days ago
7. Which of the following are the RBC
equivalents of human leukocyte antigens 13. Which of the following donors could be
(HLA)? accepted for blood donation?
a. Kpa, Kpb, Kpc a. 18 y.o. woman with a hematocrit
b. Jsa, Jsb value of 38%
b. 45 y.o. man with a BP of 180/100 19. Cryoprecipitated antihemophilic factor is not
c. 60 y.o. Man with history of Babesiosis recommended for the treatment of:
d. 30 y.o. man who had been vaccinated a. Hemophilia A
for rabies 6 months ago b. VWD
c. Hemophilia B
14. All of the following donors can be accepted d. Hypofibrinogenemia
for donating blood EXCEPT:
a. 30 y.o. Female who had a rubella 20. What is the purpose of adding a rejuvenating
vaccination 6 weeks ago solution to RBC products?
b. 20 y.o. Male with a body temperature of a. To restore ATP and 2,3-DPG
99.4 F b. To stabilize RBC membrane
c. 50 y.o. Man with night sweats and c. To prevent lysis of RBCs during storage
skin lesions at cold temperature
d. 45 y.o. Woman who was a recipient of d. TO increase the harvest of WBCs
recombinant growth hormone
21. Prothrombin complex concentrates are used
15. A man turns pale and complains of dizziness to treat which of the following?
while donating blood. What is the best course a. Hemophilia A
of action? b. Hemophilia B
a. Continue the donation c. Hemophilia C
b. Withdraw the needle, raise his feet, d. VWD
and administer ammonia spirits 22. What is the purpose of adding citric acid to
c. Discontinue the donation and provide a blood bag preservatives?
paper bag a. To prevent coagulation
d. Tell him to sit upright and not have a b. TO restore ATP levels
reaction until the donation is finished c. TO improve survival of red cells
d. To prevent caramelization
16. What would be the cause of jet-like pulsating
bleeding with bright red blood during 23. Fresh frozen plasma stored at -65 C would
donation? have a shelf life of
a. Increased vacuum from the blood bag a. 1 year
b. The donor is hypertensive b. 5 years
c. Brachial artery was punctured c. 7 years
d. The donor is a chronic smoker d. 10 years

17. Red Blood Cells stored in CPDA-2 blood bag 24. Each unit of cryoprecipitate prepared from
would have a shelf-life of: whole blood should contain approximately
a. 21 days how many units of factor VIII
b. 35 days a. 40
c. 2 days b. 80
d. 42 days c. 150
d. 200
18. What is the proper storage protocol for
granulocyte concentrates? 25. Quality control for RBCs requires a maximum
a. 1-6 C hematocrit level of:
b. -18 C a. 75%
c. Room temperature with agitation b. 80%
d. Room temperature without agitation c. 85%
d. 90%
d. 1,2,3, and 4
26. Cryoprecipitate that has been thawed must
be transfused within 32. Hepatitis B virus remains infectious on
a. 6 hours environmental surfaces for:
b. 8 hours a. 7 days
c. 4 hours b. 1 month
d. 24 hours c. 1 year
d. 3 days
27. Platelets prepared from a whole blood
donation require which of the following? 33. HBV is transmitted most frequently
a. A light spin followed by a heavy spin a. By unknown methods
b. Two light spins b. Through blood transfusion
c. A hard spin followed by a light spin c. By needle sharing among IV drug
d. Two heavy spins abusers
d. By sexual activity
28. A single unit of granulocyte concentrate
should contain at least ____ WBCs 34. Which of the following blood-borne
a. 1.0 x 109 pathogens is destroyed under prolonged cold
b. 1.0 x 1010 temperature storage
c. 3.0 x 1011 a. Babesia microti
d. 3.5 x 1010 b. Trypanosoma cruzi
c. Treponema pallidum
29. Which of the following antibodies is most d. Plasmodium falciparum
responsible for immediate hemolytic
transfusion reactions? 35. What procedure would help to distinguish
a. ANti-N between an anti-C and anti-Fyb in an
b. Anti-D antibody mixture?
c. Anti-B a. Use a thiol reagent
d. Anti-Leb b. Run an enzyme panel
c. Lower the pH of test serum
30. When a suspected hemolytic transfusion d. Run a regular panel
reaction occurs, the first thing to do is
a. Slow the transfusion rate and call the 36. What samples are required to perform
physician compatibility testing?
b. Administer medication to stop the a. Patient serum and donor serum
erection b. Patient serum and donor cells
c. First inform the laboratory to begin an c. Donor serum and patient cells
investigation d. Donor cells and patients cells
d. Stop the transfusion but keep the
intravenous line open with saline 37. What is the minimum testing required for a
massively transfused patient?
31. Transfusion reactions can be caused by: a. Immediate spin crossmatch or
(1) Overheated donor blood forward typing ABO
(2) Bacterial contamination b. ABO, Rh, Corssmatch
(3) Hypokalemia c. ABO, forward and reverse typing
(4) Citrate toxicity d. Antibody screen and ABO typing
a. 1,2 and 4
b. 2,3, and 4 38. What is the major advantage of gel
c. 1,3, and 4 technology
a. No cell washing steps b. Wash cells with 0.6N HCl
b. Specialized equipments c. Obtain another cord sample
c. Standardization d. Test cells using saline typing reagents
d. Use of IgG_coated control cells
45. Cold AIHA is sometimes associated with
39. The purpose of immediate spin crossmatch is infection by:
a. To ensure survival of transfused RBCs a. S. aureus
b. To determine ABO compatibility b. M. pneumoniae
between donor and recipient c. E. coli
c. Detect cold-reacting unexpected d. S. pyogenes
antibodies
d. Meet computer crossmatch 46. CEphalosporin given in massive doses has
requirements been associated with RBC hemolysis. Which
of the classic mechanisms is involved in the
40. Blood donor and recipient samples used in hemolytic process?
cross matching must be stored for a minimum a. Membrane modification
of how many days following transfusion? b. Drug adsorption
a. 2 c. Immune complex formation
b. 5 d. Autoantibody Formation
c. 7
d. 10 47. Backup copies of the information system:
a. Can be used to restore the
41. After the addition of IgG-coated RBC to a information system data and software
negative AHG reaction during an antibody b. Are used to maintain hardware
screen, a negative result is observed. Which components
of the following is a correct interpretation? c. Are performed once a month
a. The antibody screen is negative d. Are created any time changed are made
b. Reactive AHG reagent was added to the system
c. The saline washings were adequate 48. User passwords should be:
d. AHG reagent was neutralized a. Shared with others
b. Never changed
42. RBCs must be washed in saline at least three c. Kept confidential
times before the addition of AHG reagent to: d. Posted at each terminal
a. Wash away any hemolyzed cells
b. Neutralize any excess AHG reagent 49. All of the following items should be checked
c. Increase the antibody binding to antigen quarterly EXCEPT
d. Remove traces of free serum a. Cell washers
globulins b. CEntrifuge timers
c. Blood warmers
43. Blood for intrauterine transfusion should be d. Platelet incubators
all of the following EXCEPT:
a. Compatible with maternal serum 50. What is the principle involved in the gel
b. Screened for CMV technology
c. Gamma-irradiated a. Hemadsorption
d. More than 7 days old b. Hemagglutination-inhibition
c. Hemagglutination
44. What should be done if Wharton’s jelly d. Precipitation
cannot be removed from cord cells?
a. Collect a heel stick sample
51. Which of these match(es) is/are correct?
(1) Karl Landsteiner - ABO group 58. What is the major component of natural
(2) Elie Metchnikoff - Phagocytosis humoral immunity?
(3) Rosalyn Yallow - ELISA a. Immunoglobulins
(4) Jules Bordet - COmplement fixation b. Complement
a. 1,2,3 c. B cells
b. 1,2,4 d. Cytokines
c. 2,3,4
d. 1,2,3,4 59. Interferon can be produced by which of the
following?
52. When was the T cell receptor gene (1) Macrophages
discovered? (2) T lymphocytes
a. 1979 (3) Virus-infected cells
b. 1987 a. 1,2
c. 1984 b. 1,3
d. 1981 c. 2,3
d. 1,2,3
53. Cytokine produced mainly by macrophages: 60. Which of the following IgG subclasses does
a. IL-1 not cross the placenta?
b. IL-2 a. IgG1
c. IL-6 b. IgG2
d. IL-23 c. IgG3
d. IgG4
54. It is a non-specific indicator of inflammation
believed to be an antibody to the 61. What is the major amino acid component of
C-polysaccharide of pneumococci: the hinge region of an IG molecule?
a. Alpha-antitrypsin a. Lysine
b. C-reactive protein b. GLutamine
c. Serum amyloid A c. Proline
d. Ceruoplasmin d. Tryptophan

55. Acquired immunodeficiency syndrome (AIDS) 62. What region determines the immunoglobulin
shows a change in the ratio of: class?
a. T/B cells a. Fc
b. B1/B2 cells b. CH
c. Albumin/Globulin c. VL
d. CD4+/CD8+ d. CL

56. Hybridomas are formed from: 63. Marker for NK cells


a. Antibodies a. CD19
b. B lymphocytes b. CD45
c. T lymphocytes c. CD34
d. Phagocytes d. CD56

57. T cells are involved in: 64. How are cytotoxic T cells and NK cells
a. Humoral immunity similar?
b. Wheal and flare reaction a. Effective against virally infected cells
c. Cell-mediated immunity b. Recognize antigen in association with
d. Antibody production HLA class II markers
c. Do not bind to infected cells a. Haptens
d. Require antibody to be present b. Mitogens
c. Adjuvants
65. Which immunology cross-links mast cells to d. Immunogens
release histamine?
a. IgG 72. The alternative pathway of the complement
b. IgM cascade is initiated by:
c. IgE (1) IgG and IGM
d. IgA (2) Aggregates of IgA
(3) Yeast cell wall
66. Humoral immunity is due to: (4) Mannose group
a. Macrophage activity (5) Lipopolysaccharides
b. Suppressor T lymphocytes (6) CObra venom factor
c. B lymphocytes that transform into
plasma cells a. 1,2,3
d. Lymphokines b. 2,3,5,6
c. 3,4,5,6
67. The predominant antibody in an anamnestic d. 1,3,4,5
response is:
a. IgG 73. Deficiency of C3 complement component is
b. IgM associated with:
c. IgA a. Paroxysmal nocturnal hemoglobinuria
d. IgD b. Hereditary angioneurotic edema
c. Gonococcal or meningococcal infection
68. The binding strength of an antibody for an d. Severe recurrent bacterial infection
antigen is referred to as its:
a. SPecificity 74. Fetus in a mother’s womb is considered a/an:
b. Affinity a. Allograft
c. Avidity b. Autograft
d. Titer c. Heterograft
d. Syngeneic graft
69. THe HLA complex is located primarily on
chromosome 75. HLA-B27 is most commonly associated with:
a. 3 a. Systemic lupus erythematosus
b. 6 b. Rheumatoid arthritis
c. 9 c. Ankylosing spondylitis
d. 17 d. Graves’ disease

70. For an antibody-coated antigen to be 76. If the interval between heating the serum for
phagocytized, what part of the antibody the VDRL test and testing exceeds 4 hours,
molecule fits into a receptor on the the serum should be reheated at:
phagocytic cell? a. 37 C for 30 mins
a. Fab region b. 37 C for 10 mins
b. Hinge region c. 56 C for 30 mins
c. Variable region d. 56 C for 10 mins
d. Fc region
77. Patients with syphilis develop an antibody
71. These are substances added to a vaccine to response to a substance known as:
enhance the immune response: a. Reagin
b. Complement 84. Among the following congenital infections,
c. Cardiolipin which one is the most common?
d. Hemolysin a. Congenital syphilis
b. Rubella infection
78. The fluorescein-anti human gamma globulin c. Cytomegalovirus
used in the FTA-ABS test: d. Herpes virus infection
a. Makes the antigen-antibody reaction
visible 85. In the exoantigen test for Histoplasma
b. Is added to the serum before the capsulatum, which bands are present in a
antigen is added positive result?
c. Is added to the antigen before the a. A
serum is added b. H and/or M
d. Makes the Ab visible c. 1,2,3
d. HS, HL, F
79. The reactions on DNA-methyl green
substrates when testing for the presence of 86. If only anti-Hbs is positive, which of the
anti-DNase are: following can be ruled out?
a. (+) blue, (-) yellow a. HBV vaccination
b. (+) red, (-) orange b. Distant past infection with HBV
c. (+) Green, (-) colorless c. HBIG injection
d. (+) purple, (-) yellow d. Chronic HBV infection

80. The widal and weil-felix reactions are 87. This test will detect parasitic lactate
examples of techniques to detect: dehydrogenase enzyme present in malarial
a. Heterophile antibodies organisms:
b. Febrile agglutinins a. Optimal
c. Reagin b. Malaquick
d. Forssman antigen c. Monospot
d. Streptozyme
81. Forssman antibodies are adsorbed by:
a. Beef erythrocytes 88. Which is most likely a positive Western blot
b. Guinea pig kidney cells result for infection with HIV
c. Both a. BAnd at p24
d. Neither b. Bands at gp160
c. Bands at p24 and p31
82. What is the first antibody to be produced in a d. Band at p24
patient with hepatitis B virus infection?
a. Anti-Hbs 89. Which disease might be indicated by
b. Total anti-Hbs antibodies to smooth muscle?
c. IgM anti-HBc a. Myasthenia gravis
d. Anti-HBe b. Primary biliary cirrhosis
c. Chronic active hepatitis
83. What is the first antibody to be produced in a d. Hashimoto’s thyroiditis
patient with HIV infection?
a. Antibody to p24 90. Which immunofluorescence pattern indicates
b. Antibody to gp41 the need for further testing by Ouchterlony
c. Antibody to gp120 double diffusion?
d. Antibody to p15 a. Solid
b. Mottled
c. Rim d. weekly
d. Nucleolar

91. What disease is indicated by a high titer of 97. When preparing monoclonal antibodies
anti-centromere antibodies? a. Inject an animal with an antigen
a. Scleroderma b. Fuse B lymphocytes with a malignant
b. Sclerodactyly cell
c. MCTD c. Remove the B lymphocytes
d. SLE d. All of the above

92. Which of the following is a sialylated Lewis 98. The air temperature throughout the serology
blood group antigen associated with laboratory is 20 C. How will this affect RPR
colorectal carcinoma? test results?
a. Ca 19-9 a. No effect - the acceptable test range is
b. CEA 20-24 C
c. Ca 549 b. Weaken reactions so that false
d. Ca 15-3 negatives occur
c. Strengthen reaction reactions so that
93. Associated with increased serum positive titers appear elevated
alpha1-fetoprotein level d. Increase the number of false positives
(1) testicular cancer from spontaneous clumping
(2) Hepatocellular carcinoma
(3) Pancreatic carcinoma 99. A device used to measure radioactivity in
(4) Prostatic carcinoma radioimmunoassay
a. 1,2 a. Spectrophotometer
b. 1,2,3 b. Scintillation counter
c. 3,4 c. Fluorometer
d. 1,2,4 d. None of these

94. Not true regarding hCG 100. Most commonly used label in ELISA
a. Beta subunit confers immunogenic a. HRP
specificity b. FITC
b. Used to confirm pregnancy c. Tritiated hydrogen
c. Used as tumor marker d. G6PD
d. Found in hepatoma

95. What immune elements are involved in a


reaction to poison ivy?
a. IgE antibodies
b. NK cells and IgG antibody
c. B cells and IgM antibody
d. T cells and macrophages

96. Needles used to deliver VDRL antigen should


be checked
a. Monthly
b. Bimonthly
c. Daily or every time tests are
performed
1.What triggers the 2. What is 3. Which of the Ig has the 4. It is the 5. It is an Ig 6. What Ig is
immune the major Ig ability to cross the predominant found on the generally
Response? in normal placenta? Ig in surface of B responsible for an
serum? secretions lymphocytes in individual’s
IgG association immunity to
IgG. IgA with IgM invading parasites
Immunogens
IgD IgE
7. What Ig is 8. What are 9. Which of the principal 10. Which of the 11. Which of the 12. Enumerate the
effective in the three categories of antigenic principal principal categories of five sets of allotypic
agglutination principal determinants is the dominant categories of antigenic determinants markers found in
and cytolytic categories of type found on the Ig of all antigenic are individually specific humans
reax antigenic animals of a species determinants to each Ig molecule
determinants are genetically
IgM Isotype determined Gm, Km, Mm, Am,
Isotype, variations Idiotype and Hv
Allotype, and representing the
Idiotype presence of
allele genes at a
single locus
within a species

Allotype
13. What is 14. What are 15. What is the difference 16. Are proteins 17. Are oils heat 17. What is the
otherwise known as the 4 phases between the primary and heat stable or stable or labile? ability of a
the subsequent the IgM secondary antibody response labile? particular
exposure to the antibody antibody to
same antigen proceeds to Primary Secondary combine with a
producing a following a Time Shorter Labile Stable particular
memory response foreign lag phase, antigen
antigen longer
Anamnestic challenge plateau, Specificity
response and more
Lag- no antibody gradual
detectable decline
Log- antibody titer Type of IgM IgG
rises logarithmically antibody
Plateau- antibody Antibody Higher
titer stabilizes titer titer
Decline phase –
antibody is
catabolized
18. What is the 19. What is 20. What are the non specific 21. What does it 22. What is 23. What kind of
initial force of the attractive forces generated by the mean when a px is electrophoresis used to lymphocyte
attraction that functional interaction between electron cachectic identify? stimulates B cell
exists between a combining clouds and hydrophobic bonds growth and
single Fab site strength of Presence or absence of differentiation
on an antibody an antibody Van der Waals forces aberrant proteins and (humoral immunity)
molecule and a with its Poor physical state determine when
single epitope or antigen different groups of Helper T (Th) cells
determinant site proteins increase or
on the decrease in serum or
corresponding Avidity urine.
antigen

Affinity
24. What kind of 25. What kind of 26. What are the 4 types of B cell 27. In the immune 28. What Ig is the 29. What Ig is the
lymphocyte functions lymphocyte surface markers? response to foreign principal class of protective antibody
for antibody functions for the lysis proteins, the first antibody in interstitial of the mucosal
production? of virus-infected cells Ig receptor antibody to appear fluids? surfaces?
Fc receptor is?
B cells NK (Natural killer) Receptors that bind fragments of IgG IgA
cells the cleaved complement IgM
component C3
B cell surface antigens coded by
the MHC Class II
30. Name the four 31. Also known as 32. Most commonly encountered 33. Technical term 34. What transfusion 35. What should be
Treponemal species and Pediatric syphilis type of transfusion reaction for iron overload reaction is often found the ambient
the disease/s related to in patients who are temperature for
each Congenital syphilis Febrile nonhemolytic transfusion Transfusion chronically dependent component
reaction Hemosiderosis on transfunsion separation?
→ T. pallidum
subspecies pallidum Transfusion 20oC to 25oC
▪ Syphilis (STD) Hemosiderosis/Iron
→ T pallidum subspecies overload
pertenue
▪ Yaws (ulcerative skin
lesions)
→ T. pallidum
subspecies endemicum
▪ Endemic syphilis
(bejel) oral mucosa
lesions
→ T. pallidum
subspecies carateum
▪ Pinta (flat red or blue
lesions)
36. What is the 37. When can the 38. What is the shelf-life of fresh 39. What cancer 40. What are the tests 41. What is the
maximum time limit for platelet concentrate frozen plasma? antigen is associated for pre-transfusion reason why you
component separation bag be stored in with pancreatic or compatibility testing? should never let the
from the time of platelet agitators? gastric carcinoma; antigen and
collection? associated with H. ● ABO typing antibody meet?
After leaving it 1 year at -30C pylori; and Lewis ● Rh typing
8 hours undisturbed for one blood group system ● Antibody Screen It will cause
hour ● Crossmatching Intravascular
CA 19.9 Hemolysis (IVH)
42. What is the 43. What blood 44. At which chromosome number 45. What are the 46. What are the
frequency of ABO blood group system is are the ABO genes found? immunodominant enzymes coded by the
groups? associated with the sugars of the A, B, of the A, B, and O
most severe Chromosome 9 and O genes, genes, respectively?
O > A > B > AB hemolytic respectively?
transfusion reaction?
N-acetyl N-acetyl D-
ABO Blood Group galactosamine galactosaminyl
transferase

D-galactose galactosyl transferase

L-fructose None; is an amorph,


meaning it doesn’t
produce any enzyme
1/7/23, 5:53 PM Medical Technology Board Examination Review Notes on ISBB - Students Lessons and Examinations

NEED HELP WITH YOUR PAPER? CLICK HERE! 

Published with Hindawi


Math, Eng., & Comp. Sci. Articles Are Peer Reviewed, Open
Access And Available Online

Hindawi Open

Medical Technology Board Examination Review Notes on


ISBB
  JULY 03, 2018

 SHARE:  Facebook  Twitter  Google+  Pinterest

Immunology, Serology and Blood Banking

1. Incorrect statement about naturally occurring antibodies:


     a. React best in saline
     b. React best at room temperature or at 4OC
     c. They are usually IgG
     d. Do not usually cross the placenta

2. Which antibody is most likely to be involved in hemolytic disease of the


newborn?
     a. Anti-E
     b. Anti-Lea
     c. Anti-M
     d. Anti-Jsa

POPULAR

Medical Technology Board


Examination Review Notes
Recalls
 JULY 04, 2018

Medical Technology Board


Examination Review Notes on
Hematology
 JULY 04, 2018

The Parable of Sadhu –


Question of Ethics
 JUNE 28, 2018

lessonsandexaminations.blogspot.com/2018/07/medical-technology-board-examination-review-notes-on-isbb.html 1/14
1/7/23, 5:53 PM Medical Technology Board Examination Review Notes on ISBB - Students Lessons and Examinations

3. What blood type is not possible for an offspring of AO and BO persons? 


     a. AB
     b. A or B
     c. O
     d. All are possible

4. Nature of ABO antibodies:


     a. IgM
     b. IgG and IgM
     c. IgA
     d. IgG, IgM and IgA

5. Waldenström’s macroglobulinemia, multiple myeloma and increased levels


of fibrinogen is associated with what ABO discrepancy?
     a. Group I
     b. Group II
     c. Group III
     d. Group IV

6. In which of the following blood group systems may the RBC typing change
during pregnancy?
     a. Kidd
     b. Lewis
     c. MNS
     d. Kell

7. Which of the following are the RBC equivalents of human leukocyte


antigens (HLA)?
     a. Kpa, Kpb, Kpc
     b. Jsa, Jsb
     c. Doa, Dob
     d. Bga, Bgb, Bgc

8. Which of the following does not belong to the high titer low avidity
antibodies?
     a. Anti-Ch
     b. Anti-Yka
     c. Anti-Co
     d. Anti-JMH

9. Incorrect statement about Kell blood group:


     a. 90% of the population is Kell negative
     b. Anti-K reacts best in saline at room temperature
     c. Anti-K can cause hemolytic disease of the newborn
     d. Next to Rh (D), the Kell (K) antigen is the most immunogenic

10. Adults who have the Le, Se, and H genes will exhibit which Lewis
antigen on their red cells?
     a. Lea
     b. Leb
     c. Lea and Leb
     d. None

11. Given the following reactions, identify the cause of discrepancy:


Cell Typing:            Serum Typing:
Anti-A = 4+             A1 cells = 0
Anti-B = 2+             B cells = 4+

lessonsandexaminations.blogspot.com/2018/07/medical-technology-board-examination-review-notes-on-isbb.html 2/14
1/7/23, 5:53 PM Medical Technology Board Examination Review Notes on ISBB - Students Lessons and Examinations
     a. Hypogammaglobulinemia LABELS
     b. Hodgkin’s lymphoma
     c. Cold agglutinins PROFESSIONAL REGULATION COMMISSION
     d. Acquired B phenomenon (470) 
BOARD PASSERS (145)

12. Which of the following is a contraindication for accepting a donor for DEFENSIVE DRIVING (132)

plateletpheresis? ROOM ASSIGNMENTS (130)


     a. 72 hours since the last apheresis
ROAD SAFETY DRIVING (127)
     b. Plasmapheresis of 800mL 1 week ago
     c. Platelet count of 100 x 109/L TOP 10 PASSERS (105)

     d. Aspirin ingestion 7 days ago LEARN HOW TO DRIVE (97)

ACADEMIC PAPERS (94)


13. Which of the following donors could be accepted for blood donation?
     a. 18-year old woman with a hematocrit value of 38% DRIVING SCHOOL (92)

     b. 45-year old man with a blood pressure of 180/100 TOP PERFORMING SCHOOLS (87)
     c. 60-year old man with history of Babesiosis
HUNTING (83) HUNTING IN LABRADOR (83)
     d. 30-year old man who had been vaccinated for rabies 6 months ago
ENTERPRISE RESOURCE PLANNING SOFTWARE
(77)
14. All of the following donors can be accepted for donating blood EXCEPT: CUSTOMER RELATIONSHIP MANAGEMENT (74)
a. 30-year old female who had a rubella vaccination 6 weeks ago
REMOTE CONTROL (50) RESEARCH (47)
b. 20-year old male with a body temperature of 99.4OF
SOUND SYSTEMS (47) SUBWOOFERS (47)
c. 50-year old man with night sweats and skin lesions
d. 45-year old woman who was a recipient of recombinant growth hormone DEER HUNTING (41)

HUMAN CAPITAL MANAGEMENT (39)


15. A man turns pale and complains of dizziness while donating blood. What
is the best course of action? HUMAN RESOURCES MANAGEMENT SOFTWARE
a. Continue the donation (39)
SURROUND SYSTEM (37) COMPUTER (33)
b. Withdraw the needle, raise his feet and administer ammonia spirits
c. Discontinue the donation and provide a paper bag HARD DRIVE (32) NOTEBOOK (32)

d. Tell him to sit upright and not have a reaction until the donation PROFESSIONAL TEACHERS (31)
is finished
LAPTOP (31) COYOTE HUNTING (29)

16. What would be the cause of jet-like pulsating bleeding with bright red FISHING (28) DUCT TAPE (25)

blood during blood donation? ELECTRICAL TAPE (25)


     a. Increased vacuum from the blood bag
FISHING IN LABRADOR (25)
     b. The donor is hypertensive
     c. Brachial artery was punctured MASKING TAPE (25) PACKING TAPE (25)

     d. The donor is a chronic smoker SURGICAL TAPE (25)

MEDICAL TECHNOLOGY (23)


17. Red blood cells stored in CPDA-2 blood bag would have a shelf-life of:
     a. 21 days FINANCIAL MANAGEMENT (22)

     b. 35 days CURLY FRIES (20) FRENCH FRIES (20)


     c. 2 days
PROFESSIONAL TEACHER (20)
     d. 42 days
MASTER PLUMBERS (18)

18. What is the proper storage protocol for granulocyte concentrates? INTERNET MARKETING (18)
     a. 1-6OC
PERSONAL FINANCE (18)
     b. -18OC
VIDEO MARKETING (18)
     c. Room temperature with agitation
     d. Room temperature without agitation DEBT SOLUTION (17) INSURANCE (17)

SOUND THERAPY (17) DEBTS (16)


19. Cryoprecipitated antihemophilic factor is not recommended for the
MEDICAL TECHNOLOGISTS (15)
treatment:
     a. Hemophilia A OCCUPATIONAL THERAPISTS (15)
     b. von Willebrand’s disease
PHYSICAL THERAPISTS (15)
     c. Hemophilia B
ACCOUNTING SOFTWARE (15)
     d. Hypofibrinogenemia
BUSINESS INTELLIGENCE SOFTWARE (15)
20. What is the purpose of adding rejuvenating solutions to RBC products?
E-COMMERCE BLOGS (15)
     a. To restore ATP and 2,3-DPG
MANAGEMENT INTELLIGENCE (15)
     b. To stabilize RBC membrane
     c. To prevent lysis of RBCs during storage at cold temperature PHYSICIANS (14) NURSES (14)
     d. To increase the harvest of WBCs
ONLINE MARKETING (14)

lessonsandexaminations.blogspot.com/2018/07/medical-technology-board-examination-review-notes-on-isbb.html 3/14
1/7/23, 5:53 PM Medical Technology Board Examination Review Notes on ISBB - Students Lessons and Examinations

E-COMMERCE SOFTWARE (13)


21. Prothrombin complex concentrates are used to treat which of the
LEASE MANAGEMENT (13)
following?
     a. Hemophilia A PSYCHOLOGISTS (12) 
     b. Hemophilia B PSYCHOMETRICIANS (12)
     c. Hemophilia C
CERTIFIED PUBLIC ACCOUNTANTS (12)
     d. von Willebrand’s disease
HOW TO BUILD YOUR BUSINESS WEBSITE (12)
22. What is the purpose of adding citric acid to blood bag preservatives?
MOOSE HUNTING (12)
     a. To prevent coagulation
GUIDANCE COUNSELORS (11) LOVE (11)
     b. To restore ATP levels
     c. To improve survival of red cells PHARMACISTS (11)

     d. To prevent caramelization SANITARY ENGINEERS (11)

CONSOLIDATED LOAN (11)


23. Fresh frozen plasma stored at -65OC would have a shelf-life of:
     a. 1 year OUTDOOR ADVENTURE (11)

     b. 5 years ARTICLE MARKETING (10)


     c. 7 years
BARACK OBAMA (10)
     d. 10 years
COFFEE MACHINES (10)

24. Each unit of cryoprecipitate prepared from whole blood should contain COFFEE MAKERS (10)
approximately how many units of factor VIII?
FIFTY SHADES OF GREY (10)
     a. 40
     b. 80 GUITAR PLAYING (10) GUITARS (10)

     c. 150 LOSING WEIGHT FAST (10)


     d. 200
PERSONAL DEVELOPMENT (10)

BLOG ARCHIVE WHITE HOUSE (10)


ROMANCE (10)
25. Quality control for RBCs requires a maximum hematocrit level of:
     a. 75%
►LANDSCAPE
2019 (33) ARCHITECTS (9)
     b. 80% 2018 (108) (PRACTICAL) (9)
▼OPTOMETRISTS
     c. 85% ▼ July (34)
     d. 90% OPTOMETRISTS (WRITTEN)
Special Article: What's(9)
At the Bottom of
the Ocean?
ADDICTION (9) DRUG REHABS (9)
Licensure Examination for Teachers
26. Cryoprecipitate that has been thawed must be transfused within ___ (LET)/ Professi...
MECHANICAL ENGINEERS (9)
hours Licensure Examination for Teachers
     a. 6 ELECTRONICS
(LET)/ TECHNICIANS
Professi... (8)

     b. 8 Licensure Examination for Teachers


INTERIOR DESIGNERS (8)
(LET)/ Professi...
     c. 4
Licensure
SOCIAL WORKERSExamination
(8) for Teachers
     d. 24 (LET)/ Professi...
X-RAY TECHNOLOGISTS
Licensure Examination(8) for Teachers

27. Platelets prepared from a whole blood donation require which of the (LET)/ Professi...
CRIMINOLOGISTS (8) ARCHITECTS (7)
Air Transport Economics: United Arab
following? Emirates Air ...
ELECTRONICS ENGINEERS (7)
     a. A light spin followed by a heavy spin Case Study for Saudi Arabian Airlines
     b. Two light spins FISHERIES TECHNOLOGISTS
Management S... (7)
     c. A hard spin followed by a light spin Geology Board Examination Review Notes
GEOLOGISTS (7)
1
     d. Two heavy spins Medical Technology Board
METALLURGICAL ENGINEERS (7) Examination
Review Notes ...
28. A single unit of granulocyte concentrate should contain at least ____ RADIOLOGIC TECHNOLOGIST
Criminology (7)
Board Examination Review
Notes 3
WBCs CHEMICAL ENGINEERS (7) DRUGS (7)
Criminology Board Examination Review
     a. 1.0 x 109
WEIGHTNotes 2
LOSS PROGRAM (7)
     b. 1.0 x 1010 Criminology Board Examination Review
     c. 3.0 x 1011 Notes 1 (6)
AGRICULTURISTS
Medical Technology Board Examination
     d. 5.5 x 1010 DENTISTS (PRACTICAL) (6)
Review Notes ...
Medical
GEODETIC Technology
ENGINEERS (6) Board Examination
29. Which of the following antibodies is most responsible for immediate Review Notes ...
NUTRITIONISTS-DIETITIANS
Medical Technology Board (6) Examination
hemolytic transfusion reactions?
Review Notes ...
     a. Anti-N AUTOMOTIVE INDUSTRY (6)
Medical Technology Board Examination
     b. Anti-D Review
BREEDING Notes ...(6)
STRATEGIES DECOYS (6)
     c. Anti-B Medical Technology Board Examination
Review Notes
HOME MORTGAGE (6) ...
     d. Anti-Leb
Medical Technology Board Examination
30. When a suspected hemolytic transfusion reaction occurs, the first thing MECHANICAL DECOYS (6)
Review Notes ...
to do is: Medical
MOTION Technology
DECOYS (6) Board Examination
Review Notes ...

lessonsandexaminations.blogspot.com/2018/07/medical-technology-board-examination-review-notes-on-isbb.html 4/14
1/7/23, 5:53 PM Medical Technology Board Examination Review Notes on ISBB - Students Lessons and Examinations
a. Slow the transfusion rate and call the physician Medical
PERSONAL Technology
HEALTH (6) Board Examination
REFINANCING (6)
Review Notes ...
b. Administer medication to stop the reaction SUGAR GLIDERS (6)
Medical Technology Board Examination
c. First inform the laboratory to begin an investigation Review Notes ...
TIPS IN LOSING WEIGHT (6)
d. Stop the transfusion but keep the intravenous line open with saline Medical Technology Board Examination 
TIPS INReview Notes ... (6)
SOUND THERAPY
Medical Technology Board Examination
31. Transfusion reactions can be caused by: WEIGHTReview
LOSS (6)
Notes WORDPRESS
... (6)
1. Overheated donor blood      3. Hypokalemia Medical Technology Board
CERTIFIED PLANT MECHANICS (5)Examination
2. Bacterial contamination     4. Citrate toxicity Review Notes ...
     a. 1, 2 and 4 Medical
OCULAR Technology Board
PHARMACOLOGISTS (5) Examination
Review Notes ...
     b. 2, 3 and 4 RADIOLOGIC TECHNOLOGISTS (5)
Medical Technology Board Examination
     c. 1, 3 and 4 Review Notes ...
ADVENTURE (5) BUSINESS (5)
     d. 1, 2, 3 and 4 Medical Technology Board Examination
CARIBOUReview Notes
HUNTING (5)... CHEMISTS (5)
Medical Technology Board Examination
32. Hepatitis B virus remains infectious on environmental surfaces for: CIVIL ENGINEERS (5) ... E-COMMERCE (5)
Review Notes
     a. 7 days Medical Technology Board Examination
FORESTERS (5) LEGAL (5)
     b. 1 month Review Notes ...
LIFE Medical
COACHESTechnology
(5) Board Examination
     c. 1 year
Review Notes ...
     d. 3 days AERONAUTICAL ENGINEERS (4)
Medical Technology Board Examination
Review Recall...
BUSINESS MANAGEMENT (4)
33. HBV is transmitted most frequently: Medical Technology Board Examination
Review
CERTIFIED PLANTNotes ...
MECHANICS (CPM) (4)
     a. By unknown methods
Medical Technology Board Examination
     b. Through blood transfusion CUSTOMS BROKERS
Review Notes(4)...
     c. By needle sharing among IV drug abusers ► June (43)
LET BOARD EXAM (4)
     d. By sexual activity ► May (9)
MARINE ENGINEERS
► March (13) (4)

► February (9)ENGINEERS (COMPUTER-BASED


METALLURGICAL
34. Which of the following blood-borne pathogens is destroyed under ►EXAM)
2017 (33)
(4)NAVAL ARCHITECTS (4)
► 2016 (138)
prolonged cold temperature storage?
2015 (157)
►OCULAR PHARMACOLOGY (4)
     a. Babesia microti
► 2014 (219)
     b. Trypanosoma cruzi REAL ESTATE APPRAISERS (4)
► 2013 (863)
     c. Treponema pallidum REGISTERED ELECTRICAL ENGINEERS (4)
     d. Plasmodium falciparum
REGISTERED MASTER ELECTRICIANS (4)

35. What procedure would help to distinguish between an anti-C and anti-Fyb RESPIRATORY THERAPISTS (4)
in an antibody mixture? DOG TRAINER (4) MIDWIVES (4)
     a. Use a thiol reagent
NATURAL MOTION DECOYS (4)
     b. Run an enzyme panel
     c. Lower the pH of test serum PRESTASHOP (4)

     d. Run a regular panel AGRICULTURAL ENGINEERS (3)

ELECTRONICS TECHNICIAN (3)


36. What samples are required to perform compatibility testing?
     a. Patient serum and donor serum MARINE ENGINEER OFFICERS (WRITTEN) (3)

     b. Patient serum and donor cells METALLURGICAL ENGINEERS (WES) (3)
     c. Donor serum and patient cells
REAL ESTATE CONSULTANTS (3) BIKING (3)
     d. Donor cells and patient cells
37. What is the minimum testing required for a massively transfused BIRD HUNTING (3) BUSINESS TIPS (3)

patient? CRIMINOLOGY (3) DOMAIN NAMES (3)


     a. Immediate spin crossmatch or forward typing ABO
FORECLOSURE (3) FOREX (3)
     b. ABO, Rh, crossmatch
     c. ABO, forward and reverse typing LIVE FOR GOD (3) MOVING (3)

     d. Antibody screen and ABO typing MUSIC THERAPY (3) OVERWEIGHT (3)

PERFUME (3)
38. What is the major advantage of gel technology?
     a. No cell washing steps SENATE OF THE PHILIPPINES (3)

     b. Specialized equipments WEBSITE DESIGN (3)


     c. Standardization
AGENCYBLOC SOFTWARE CUSTOMER
     d. Use of IgG-coated control cells RELATIONSHIP MANAGEMENT
(2)ALSAMARKETING MARKETING AUTOMATION
SOFTWARE
39. The purpose of immediate spin crossmatch is: (2)CERTIFIED PUBLIC ACCOUNTANTS (SPLBE) (2)
     a. To ensure survival of transfused RBCs
     b. To determine ABO compatibility between donor and recipient
     c. Detect cold-reacting unexpected antibodies

lessonsandexaminations.blogspot.com/2018/07/medical-technology-board-examination-review-notes-on-isbb.html 5/14
1/7/23, 5:53 PM Medical Technology Board Examination Review Notes on ISBB - Students Lessons and Examinations
     d. Meet computer crossmatch requirements

40. Blood donor and recipient samples used in crossmatching must be stored
for a minimum of how many days following transfusion? 
     a. 2
     b. 5
     c. 7
     d. 10

41. After the addition of IgG-coated RBCs to a negative AHG reaction during
an antibody screen, a negative result is observed. Which of the following
is a correct interpretation?
     a. The antibody screen is negative
     b. Reactive AHG reagent was added
     c. The saline washings were adequate
     d. AHG reagent was neutralized

42. RBCs must be washed in saline at least three times before the addition
of AHG reagent to:
     a. Wash away any hemolyzed cells
     b. Neutralize any excess AHG reagent
     c. Increase the antibody binding to antigen
     d. Remove traces of free serum globulins

43. Blood for intrauterine transfusion should be all of the following


EXCEPT:
     a. Compatible with maternal serum
     b. Screened for CMV
     c. Gamma-irradiated
     d. More than 7 days old
44. What should be done if Wharton’s jelly cannot be removed from cord
cells?
     a. Collect a heelstick sample
     b. Wash cells with 0.6N HCl
     c. Obtain another cord sample
     d. Test cells using saline typing reagents

45. Cold AIHA is sometimes associated with infection by:


     a. Staphylococcus aureus
     b. Mycoplasma pneumoniae
     c. Escherichia coli
     d. Streptococcus pyogenes

46. Cephalosporin given in massive doses has been associated with RBC
hemolysis. Which of the classic mechanisms is involved in the hemolytic
process?
     a. Membrane modification
     b. Drug adsorption
     c. Immune complex formation
     d. Autoantibody formation

47. Backup copies of the information system:


     a. Can be used to restore the information system data and software
     b. Are used to maintain hardware components
     c. Are performed once a month
     d. Are created any time changes are made to the system

48. User passwords should be:


     a. Shared with others
     b. Never changed
lessonsandexaminations.blogspot.com/2018/07/medical-technology-board-examination-review-notes-on-isbb.html 6/14
1/7/23, 5:53 PM Medical Technology Board Examination Review Notes on ISBB - Students Lessons and Examinations
     c. Kept confidential
     d. Posted at each terminal

49. All of the following items should be checked quarterly EXCEPT: 


     a. Cell washers
     b. Centrifuge timers
     c. Blood warmers
     d. Platelet incubators

50. What is the principle involved in the gel technology?


     a. Hemadsorption
     b. Hemagglutination-inhibition
     c. Hemagglutination
     d. Precipitation
51. Which of these match(es) is(are) correct?
1. Karl Landsteiner – ABO blood group         3. Rosalyn Yallow - ELISA
2. Elie Metchnikoff – Phagocytosis            4. Jules Bordet – Complement
fixation
     a. 1, 2 and 3
     b. 1, 2 and 4
     c. 2, 3 and 4
     d. 1, 2, 3 and 4

52. When was T cell receptor gene discovered?


     a. 1979
     b. 1987
     c. 1984
     d. 1981

53. Cytokine produced mainly by macrophages:


     a. IL-1
     b. IL-2
     c. IL-6
     d. IL-12

54. It is a non-specific indicator of inflammation believed to be an


antibody to the C-polysaccharide of pneumococci:
     a. Alpha1-antitrypsin
     b. C-reactive protein
     c. Serum amyloid A
     d. Ceruloplasmin

55. Acquired immunodeficiency syndrome (AIDS) shows a change in the ratio


of:
     a. T/B cells
     b. B1/B2 cells
     c. Albumin/Globulin
     d. CD4+/CD8+ cells

56. Hybridomas are formed from:


     a. Antibodies
     b. B lymphocytes
     c. T lymphocytes
     d. Phagocytes

57. T cells are involved in:


     a. Humoral immunity
     b. Wheal and flare reaction
     c. Cell-mediated immunity
     d. Antibody production

58. What is the major component of natural humoral immunity?

lessonsandexaminations.blogspot.com/2018/07/medical-technology-board-examination-review-notes-on-isbb.html 7/14
1/7/23, 5:53 PM Medical Technology Board Examination Review Notes on ISBB - Students Lessons and Examinations
     a. Immunoglobulins
     b. Complement
     c. B cells
     d. Cytokines 
59. Interferon can be produced by which of the following?
1. Macrophages            3. Virus-infected cells
2. T lymphocytes          4. All of these
     a. 1 and 2         c. 2 and 3
     b. 1 and 3         d. 4

60. Which of the following IgG subclasses does not cross the placenta?
     a. IgG1
     b. IgG2
     c. IgG3
     d. IgG4

61. What is the major amino acid component of the hinge region of an
immunoglobulin molecule?
     a. Lysine
     b. Glutamine
     c. Proline
     d. Tryptophan

62. What region determines the immunoglobulin class?


     a. Fc
     b. CH
     c. VL
     d. CL

63. Marker for NK cells:


     a. CD19
     b. CD45
     c. CD34
     d. CD56

64. How are cytotoxic T cells and NK cells similar?


     a. Effective against virally infected cells
     b. Recognize antigen in association with HLA Class II markers
     c. Do not bind to infected cells
     d. Require antibody to be present

65. Which immunology cross-links mast cells to release histamine?


     a. IgG
     b. IgM
     c. IgE
     d. IgA

66. Humoral immunity is due to:


     a. Macrophage activity
     b. Suppressor T lymphocytes
     c. B lymphocytes that transform into plasma cells
     d. Lymphokines

67. The predominant antibody in an anamnestic response is:


     a. IgG
     b. IgM
     c. IgA
     d. IgD

68. The binding strength of an antibody for an antigen is referred to as


its:
     a. Specificity

lessonsandexaminations.blogspot.com/2018/07/medical-technology-board-examination-review-notes-on-isbb.html 8/14
1/7/23, 5:53 PM Medical Technology Board Examination Review Notes on ISBB - Students Lessons and Examinations
     b. Affinity
     c. Avidity
     d. Titer

69. The HLA complex is located primarily on:
     a. Chromosome 3
     b. Chromosome 6
     c. Chromosome 9
     d. Chromosome  17

70. For an antibody-coated antigen to be phagocytized, what part of the


antibody molecule fits into a receptor on the phagocytic cell?
     a. Fab region
     b. Hinge region
     c. Variable region
     d. Fc region

71. These are substances added to a vaccine to enhance the immune response:
     a. Haptens
     b. Mitogens
     c. Adjuvants
     d. Immunogens

72. The alternative pathway of the complement cascade is initiated by:


1. IgG and IgM        3. Yeast cell wall      5. Lipopolysaccharides
2. Aggregates of IgA  4. Mannose group        6. Cobra venom factor
     a. 1, 2 and 3
     b. 2, 3, 5 and 6
     c. 3, 4, 5 and 6
     d. 1, 2, 3 and 5

73. Deficiency of C3 complement component is associated with:


     a. Paroxysmal nocturnal hemoglobinuria
     b. Hereditary angioneurotic edema
     c. Gonococcal or meningococcal infection
     d. Severe recurrent bacterial infection

74. Fetus in a mother’s womb is considered a(n):


     a. Allograft
     b. Autograft
     c. Heterograft
     d. Syngeneic graft

75. HLA-B27 is most commonly associated with:


     a. Systemic lupus erythematosus
     b. Rheumatoid arthritis
     c. Ankylosing spondylitis
     d. Graves’ disease

76. If the interval between heating the serum for the VDRL test and testing
exceeds 4 hours, the serum should be reheated at:
     a. 37OC for 30 minutes
     b. 37OC for 10 minutes
     c. 56OC for 30 minutes
     d. 56OC for 10 minutes

77. Patients with syphilis develop an antibody response to a substance


known as:
     a. Reagin

lessonsandexaminations.blogspot.com/2018/07/medical-technology-board-examination-review-notes-on-isbb.html 9/14
1/7/23, 5:53 PM Medical Technology Board Examination Review Notes on ISBB - Students Lessons and Examinations
     b. Complement
     c. Cardiolipin
     d. Hemolysin

78. The fluorescein-antihuman gamma globulin used in the FTA-ABS test:
     a. Makes the antigen-antibody reaction visible
     b. Is added to the serum before the antigen is added
     c. Is added to the antigen before the serum is added
     d. Makes the antibody visible

79. The reactions on DNA-methyl green substrates when testing for the
presence of anti-DNase are:
     a. (+) Blue, (-) Yellow
     b. (+) Red, (-) Orange
     c. (+) Green, (-) Colorless
     d. (+) Purple, (-) Yellow

80. The Widal and Weil-Felix reactions are examples of techniques to


detect:
     a. Heterophil antibodies
     b. Febrile agglutinins
     c. Reagin
     d. Forssman antigen

81. Forssman antibodies are adsorbed by:


     a. Beef erythrocytes
     b. Guinea pig kidney cells
     c. Both
     d. Neither

82. What is the first antibody to be produced in a patient with hepatitis B


virus infection?
     a. Anti-HBs
     b. Total anti-HBc
     c. IgM anti-HBc
     d. Anti-HBe

83. What is the first antibody to be produced in a patient with HIV


infection?
     a. Antibody to p24
     b. Antibody to gp41
     c. Antibody to gp120
     d. Antibody to p15

84. Among the following congenital infections, which one is the most
common?
     a. Congenital syphilis
     b. Rubella infection
     c. Cytomegalovirus infection
     d. Herpes virus infection

85. In the exoantigen test for Histoplasma capsulatum, which bands are
present in a positive result?
     a. A
     b. H and/or M
     c. 1, 2, 3
     d. HS, HL, F

86. If only anti-HBs is positive, which of the following can be ruled out?
     a. HBV vaccination
     b. Distant past infection with HBV
lessonsandexaminations.blogspot.com/2018/07/medical-technology-board-examination-review-notes-on-isbb.html 10/14
1/7/23, 5:53 PM Medical Technology Board Examination Review Notes on ISBB - Students Lessons and Examinations
     c. HBIG injection
     d. Chronic HBV infection

87. This test will detect parasitic lactate dehydrogenase enzyme present in 
malarial organisms:
     a. Optimal
     b. Malaquick
     c. Monospot
     d. Streptozyme

88. Which is most likely a positive Western blot result for infection with
HIV?
     a. Band at p24
     b. Bands at gp160
     c. Bands at p24 and p31
     d. Band at p24 and gp120

89. Which disease might be indicated by antibodies to smooth muscle?


     a. Myasthenia gravis
     b. Primary biliary cirrhosis
     c. Chronic active hepatitis
     d. Hashimoto’s thyroiditis

90. Which immunofluorescent pattern indicates the need for further testing
by Ouchterlony double diffusion?
     a. Solid
     b. Mottled
     c. Rim
     d. Nucleolar

91. What disease is indicated by a high titer of anti-centromere


antibodies?
     a. Scleroderma
     b. Sclerodactyly
     c. MCTD
     d. SLE

92. Which of the following is a sialylated Lewis blood group antigen


associated with colorectal carcinoma?
     a. CA 19-9
     b. CEA
     c. CA 549
     d. CA 15-3

93. Associated with increased serum alpha1-fetoprotein level:


1. Testicular cancer           3. Pancreatic carcinoma
2. Hepatocellular carcinoma         4. Prostatic carcinoma
     a. 1 and 2
     b. 1, 2 and 3
     c. 3 and 4
     d. 1, 2 and 4

94. Not true regarding hCG:


     a. Beta subunit confers immunogenic specificity
     b. Used to confirm pregnancy
     c. Used as a tumor marker
     d. Found in hepatoma

95. What immune elements are involved in a reaction to poison ivy?


     a. IgE antibodies
     b. NK cells and IgG antibody

lessonsandexaminations.blogspot.com/2018/07/medical-technology-board-examination-review-notes-on-isbb.html 11/14
1/7/23, 5:53 PM Medical Technology Board Examination Review Notes on ISBB - Students Lessons and Examinations
     c. B cells and IgM antibody
     d. T cells and macrophages

96. Needles used to deliver VDRL antigen should be checked: 


     a. Monthly
     b. Bimonthly
     c. Daily or every time tests are performed
     d. Weekly

97. When preparing monoclonal antibodies:


     a. Inject an animal with an antigen
     b. Fuse B lymphocytes with a malignant cell
     c. Remove the B lymphocytes
     d. All of the above

98. The air temperature throughout the serology laboratory is 20OC. How
will this affect RPR test result?
     a. No effect – the acceptable test range is 20-24OC
     b. Weaken reactions so that false negatives occur
     c. Strengthen reactions so that positive titers appear elevated
     d. Increase the number of false positives from spontaneous clumping

99. A device used to measure radioactivity in radioimmunoassay:


     a. Spectrophotometer
     b. Scintillation counter
     c. Fluorometer
     d. None of these

100. Most commonly used label in ELISA:


     a. HRP
     b. FITC
     c. Tritiated hydrogen
     d. G6PD

ANSWER KEY: ISBB

1.      C
2.      A
3.      D
4.      D
5.      C
6.      B
7.      D
8.      C
9.      B
10.  B
11.  D
12.  C
13.  A
14.  C
15.  B
16.  C
17.  D
18.  D
19.  C
20.  A
21.  B
22.  D
23.  C

lessonsandexaminations.blogspot.com/2018/07/medical-technology-board-examination-review-notes-on-isbb.html 12/14
1/7/23, 5:53 PM Medical Technology Board Examination Review Notes on ISBB - Students Lessons and Examinations
24.  B
25.  B
26.  A
27.  A 
28.  B
29.  C
30.  D
31.  A
32.  A
33.  D
34.  C
35.  B
36.  B
37.  A
38.  C
39.  B
40.  C
41.  D
42.  D
43.  D
44.  A
45.  B
46.  A
47.  A
48.  C
49.  D
50.  C
51.  B
52.  C
53.  A
54.  B
55.  D
56.  B
57.  C
58.  B
59.  D
60.  B
61.  C
62.  B
63.  D
64.  A
65.  C
66.  C
67.  A
68.  C
69.  B
70.  D
71.  C
72.  B
73.  D
74.  A
75.  C
76.  D
77.  C
78.  A
79.  C
80.  B
81.  B
82.  C
83.  A
84.  C
85.  B
86.  D
87.  A
88.  D

lessonsandexaminations.blogspot.com/2018/07/medical-technology-board-examination-review-notes-on-isbb.html 13/14
1/7/23, 5:53 PM Medical Technology Board Examination Review Notes on ISBB - Students Lessons and Examinations
89.  C
90.  B
91.  B
92.  A 
93.  B
94.  D
95.  D
96.  C
97.  B
98.  B
99.  B
100. A

ANSWER KEY: ISBB

LABELS: MEDICAL TECHNOLOGY

 SHARE:  Facebook  Twitter  Google+  Pinterest

YOU MIGHT ALSO LIKE

Medical Technology Board Exam Medical Technology Board Medical Technology Board
Reviewer HEMATOLOGY Examination Review Notes on Examination Review Notes
 JULY 04, 2019 Parasitology Recall 3
 JULY 05, 2018  JULY 04, 2018

CATEGORIES ARCHIVE POPULAR POSTS

Medical Technology Board Examination Review Notes


July (34)
Recalls

Medical Technology Board Examination Review Notes


on Hematology

The Parable of Sadhu – Question of Ethics

Crafted with  by TemplatesYard | Distributed by Gooyaabi Templates     

lessonsandexaminations.blogspot.com/2018/07/medical-technology-board-examination-review-notes-on-isbb.html 14/14
PRE-BOARD EXAMINATION IN CLINICAL MICROSCOPY (PART 1)

1. Urine from a patient with polyuria has a high specific gravity. The patient should be evaluated for:
A. Urinary tract infection C. Diabetes insipidus
B. Diabetes mellitus D. Uremia
2. An unpreserved specimen collected at 8 AM and remaining at room temperature until the aftern shift arrives can be
expected to have:
1. Decreased glucose and ketones 3. Decreased pH and turbidity
2. Increased bacteria and nitrite 4. Increased cellular elements
A. 1, 2 and 3 B. 1, 2 and 4 C. 1 and 2 only D. 4 only
3. Red cells will disintegrate more rapidly in a urine that is:
A. Concentrated and acidic C. Dilute and acidic
B. Concentrated and alkaline D. Dilute and alkaline
4. A negative urine pregnancy performed on a random specimen may need to be repeated using a:
A. Clean-catch specimen C. Fasting specimen
B. First morning specimen D. 24-hour specimen
5. Persons taking diuretics can be expected to produce:
A. Proteinuria B. Polyuria C. Pyuria D. Oliguria
6. The renal threshold for glucose is:
A. 50 100 mg/dL B. 160 180 mg/dL C. 220 240 mg/dL D. Over 240 mg/dL
7. Increased production of vasopressin:
A. Produces a low urine volume C. Increased ammonia excretion
B. Produces a high urine volume D. Affects proximal convoluted tubule transport
8. Osmometers utilizing the freezing point colligative property of solutions are based on the principle that:
A. 1 osmole of nonionizing substance dissolve in 1 kilogram of water raises the freezing point 1.86 oC
B. 1 osmole of nonionizing substance dissolve in 1 kilogram of water lowers the freezing point 1.86oC
C. Increased solute concentration will raise the freezing point of water in direct proportion to NaCl standard
D. Decreased solute concentration will decrease the freezing point of water in direct proportion to NaCl standard
9. The PAH test is dependent on:
1. Renal blood flow 3. Tubular secretion
2. Tubular reabsorption 4. Glomerular filtration
A. 1 and 3 B. 2 and 4 C. 1 only D. 4 only
10. Which biochemical component would be present in an increased amount in dark yellow-amber-colored urine?
A. Biliverdin B. Drugs C. Bilirubin D. Urobilin
11. Specimens from patients receiving treatment for UTI, frequently appear:
A. Clear and red C. Dilute and pale yellow
B. Viscous and orange D. Cloudy and red
12. When should a 2-hour postprandial urine be collected?
A. 2 hours after fluid ingestion C. 2 hours after eating
B. 2 hours after voiding a fasting specimen D. 2 hours after fluid ingestion
13. Refractive index compares:
A. Light velocity in solutions with light velocity in solids
B. Light velocity in air with light velocity in solutions
C. Light scattering in air with light scattering in solutions
D. Light scattering by particles in solution
14. What is the principle of the colorimetric reagent strip determination of specific gravity in urine?
A. Ionic strength alters pKa of a polyelectrolyte
B. Sodium and other cations are chelated by a ligand that changes color
C. Anions displace a pH indicator from a mordant, making it water soluble
D. Ionized solutes catalyze oxidation of an azo dye
15. A urine specimen with a specific gravity of 1.008 has been diluted 1:5. The actual specific gravity is:
A. 1.008 B. 1.040 C. 1.055 D. 5.040
16. What are the most common renal stones encountered in the clinical laboratory?
A. Calcium phosphate C. Cystine
B. Calcium oxalate D. Uric acid
17. When using polarized light microscopy, which urinary sediment component exhibits Maltese cross formation?
A. RBCs B. Oval fat bodies C. Yeasts D. WBCs
18. Which of the following urine biochemical results would be obtained in hemolytic anemia?
A. Positive glucose C. Positive nitrate
B. Positive bilirubin D. Positive urobilinogen
19. In renal tubular acidosis, the pH of urine is:
A. Consistently acid C. Neutral
B. Consistently alkaline D. Variable, depending upon diet
20. What is a commonly used drug noted to produce a bright orange-red color in urine?
A. Furadantin B. Levodopa C. Rifampin D. Riboflavin
21. Which of the following would produce a yellow foam in urine when shaken which could be mistaken for bilirubin:
A. Pyridium B. Protein C. Urates D. Glucose
22. A negative glucose oxidase test and a positive test for reducing sugars in urine indicates:
A. True glycosuria C. A false-negative oxidase reaction
B. Presence of nonglucose reducing sugar D. A trace quantity of glucose
23. Acetoacetic acid is detected in urine by reaction with:
A. Sodium nitroprusside C. m-Dinitrobenzene
B. o-Toluidine D. m-Dinitrophenylhydrazine
2

24. Hemoglobin in urine can be differentiated from myoglobin using:


A. 80% ammonium sulfate to precipitate hemoglobin
B. Sodium dithionite to reduce hemoglobin
C. O-Dianisidine instead of benzidine as the color indicator
D. Microscopic exam
25. Which of the following reagents below is used to detect urobilinogen in urine:
A. p-Dinitrobenzene C. p-Dimethylaminobenzaldahyde
B. p-Aminosalicylate D. p-Dichloroaniline
26.
collect a specimen immediately upon arising the next day, the specimen is negative for protein. This indicates:
A. Nocturnal proteinuria C. Microalbuminuria
B. Orthostatic proteinuria D. Diurnal proteinuria
27. The finding of a 2+ reagent strip reaction for blood in the urine of a patient with severe lower back pain can aid in
confirming a diagnosis of:
A. Pyelonephritis B. Appendicitis C. Renal calculi D. Multiple myeloma
28. Bence Jones protein excreted in urine in cases of multiple myeloma has a unique characteristic of:
A. Reacting with reagent strips and not sulfosalicylic acid
B. Precipitating in acetic acid and heat and not with sulfosalicylic acid
C. Precipitating when heated at 60oC and dissolving at 100oC
D. Precipitating when heated at 100oC and dissolving at 60oC
29. Ammonium sulfate was added to clear urine. After filtration the supernatant remains red and has a positive reaction
on the blood portion of the reagent strip. This is caused by the presence of:
A. Myoglobin B. Medication C. Hemoglobin D. Methemoglobin
30. A positive urine bilirubin with a normal test for urobilinogen indicates:
A. Intravascular hemolysis C. Hepatitis
B. Biliary duct obstruction D. Cirrhosis
31. A urinalysis report shows:
Bilurubin: negative Blood: Small Urobilinogen: 4 EU Nitrite: negative
This suggests:
A. Abnormal destruction of RBCs in the body C. Severe UTI
B. Inflammation of the liver D. A normal urine specimen
32. A specimen that produces a cherry-
positive reaction is caused by porphobilinogen, you would expect the extraction to show:
A. Extraction into chloroform and butanol C. Extraction into chloroform but not butanol
B. Extraction into butanol but not chloroform D. No extraction into chloroform or butanol
33. What do the following results suggest:
Color: yellow hazy Bilirubin: negative
Sp. Gr.: 1.019 Blood: negative
pH: 8 Urobilinogen: 0.1 EU
Protein: trace Nitrite: positive
Glucose: negative Leukocytes: positive
Ketones: negative
A. Diebetes mellitus C. Urinary tract infection
B. Unsatisfactory specimen D. Normal female specimen
34. Examination of urine sediment for the presence of casts should be performed with:
A. Increased light under high power C. Reduced light under high power
B. Increased light under low power D. Reduced light under low power
35. Which of the following dyes are used in Sternheimer-Malbin stain?
A. Hematoxylin and eosin C. Methylene blue and eosin
B. Crystal violet and safranin D. Methylene blue and sfranin
36. The predecessor of the standardized urine microscopic examination was the:
A. Sternheimer count C. Kova system
B. Addis count D. T-system
37. The ova of which parasite may be found in the urinary sediment?
A. Trichomonas vaginalis C. Schistosoma haematobium
B. Entamoeba histolytica D. Trichuris trichiura
38. Oval fat bodies are often seen in:
A. Chronic glumerulonephritis C. Acute tubular nephrosis
B. Nephrotic syndrome D. Renal failure
39. Identification of oval fat bodies can be verified using:
A. Bright-field microscopy C. Polarized light
B. Phase contrast D. Interference-contrast microscopy
40. A sediment with moderate hematuria and RBC casts most likely results from:
A. Chronic pyelonephritis C. Acute glomerulonephritis
B. Nephrotic syndrome D. Lower urinary tract obstruction
41. Urine sediment characterized by pyuria with bacteria and WBC casts indicates:
A. Nephrotic syndrome C. Polycystic kidney disease
B. Pyelonephritis D. Cystitis
3

42. Hexagonal uric acid crystals can be distinguished from cystine crystals because:
A. Uric acid is insoluble in hydrochloric acid and cystine is not
B. Cystine gives a positive nitroprusside etst after reduction with sodium cyanide
C. Cystine crystals are colorless
D. All of the above
43. The presence of tyrosine and leucine crystals together in urine sediment usually indicates:
A. Renal failure C. Hemolytic anemia
B. Chronic liver disease D.
44. Which of the following crystals is considered nonpathological?
A. Hemosiderin B. Ammonium biurate C. Bilirubin D. Cholesterol
45. Which crystals appear in urine as a long, thin hexagonal plate, and are linked to ingestion of benzoic acid?
A. Cystine B. Hippuric acid C. Oxalic acid D. Uric acid
46. Oval fat bodies are derived from:
A. Renal tubular epithelium C. Degenerated WBCs
B. Transitional epithelium D. Mucoprotein matrix
47. Which condition is associated with the greatest proteinuria?
A. Acute glomerulonephritis C. Nephrotic syndrome
B. Chronic glomerulonephritis D. Acute pyelonephritis
48. The finding of increased hyaline and granular cast in the urine of an otherwise healthy person may be the result of:
A. Fecal contamination C. Early UTI
B. Recent strenuous exercise D. Analyzing an old specimen
49. The Guthrie test is a:
A. Bacterial inhibition test
B. Fluorometric procedure
C. Chemical procedure measured by spectrophotometer
D. Bacterial agglutination test
50. The abnormal metabolite that is present in the urine in alkaptonuria is:
A. Homogentisic acid C. Phenylpyruvate
B. Alkaptonpyruvate D. Tyrosine
51. A mousy odor in the urine is associated with:
A. Phenylketonuria B. Isovaleric acidemia C. Cystinuria D. Cystinosis
52. The finding of increased amount of the serotonin degradation product 5-HIAA in the urine is indicative of:
A. Platelet disorders B. Intestinal obstruction C. Malabsorption D. Carcinoid tumor invol. argentaffin cells
53. al retardation and increased urinary:
A. Porphyrins B. Amino acids C. Maltose D. Mucopolysaccharides
54. Regarding CSF, all of the following are indications of a traumatic tap, EXCEPT:
A. Clearing of the fluid as it is aspirated C. Xanthochromia
B. A clear supernatant after centrifugation D. Presence of clot in the sample
55. The term used to denote high WBC count in the CSF is:
A. Empyemia C. Pleocytosis
B. Neutrophilia D. Hyperpycorrhachia
56. The limulus lysate test on CSF is a sensitive assay for:
A. Demyelinating diseases of the spinal cord C. Gram-negative bacterial endotoxin
B. Cryptococcal meningitis D. Open neural tube defects
57. Pronounced reduction of CSF glucose can be seen in the following conditions, EXCEPT:
A. Viral meningitis C. Fungal meningitis
B. Bacterial meningitis D. Tuberculous meningitis
58. An elevated IgG level in CSF and an abnormal band on electrophoresis of CSF are findings consistent with the
diagnosis of:
A. Multiple sclerosis C. Meningeal involvement in leukemia
B. Muscular dystrophy D. Secondary stage of syphilis
59. CSF specimen is usually collected in 3 sterile tubes labeled in the order they are drawn. What laboratory test is
done with tube number 3?
A. Chemistry B. Microbiology C. Cell count D. Serology
60. Total volume of CSF in adults:
A. 20 mL B. 140-170 mL C. 10-60 mL D. 220-280 mL
61. The normal CSF protein is:
A. 15 to 45 mg/dL B. 15 to 45 g/dL C. 50 to 100 mg/dL D. 50 to 100 g/dL
62. To determine if fluid draining from the ear of the patient with severe head injury is CSF, the fluid should be:
A. Centrifuged and examined for the presence of ependymal cells
B. Electrophoresed for the presence of transferring isoforms
C. Analyzed for the presence of glutamine
D. Tested for low protein concentration
63. A major CSF chemical that is measured in su
A. Glucose B. Glutamine C. Lactate D. Lactate dehydrogenase
64. Which of the following terms is another name for ascitic fluid?
A. Peritoneal B. Pericardial C. Synovial D. Pleural
65. All of the following statements about serous fluids are true, EXCEPT:
A. An effusion is an abnormal accumulation of a serous fluid
B. Thoracentesis refers the collection of pericardial fluid
C. Ascites refer specifically to peritoneal fluid
D.
4

66. A transudative effusion is usually caused by all of the following, EXCEPT:


A. Congestive heart failure C. Nephrotic syndrome
B. Malignancy D. Cirrhosis
67. An exudative pleural fluid can be caused by:
A. Malignancy C. SLE or rheumatoid arthritis
B. Pulmonary infarction or infection D. All of the above
68. Measurement of blood urea nitrogen and creatinine in peritoneal fluid are requested when there is a concern about:
A. Tubercular peritonitis C. Malignacy
B. Ruptured bladder D. Gastrointestinal perforations
69. These fluid have high protein content:
A. Exudate B. Urates C. Sulfates D. Transudates
70. Useful adjunct test for tuberculous pericarditis in cases with negative stains when tuberculosis is suspected:
A. Lactate dehydrogenase activity C. Adenosine deaminase activity
B. Bilirubin level D. Lactate level
71. All of the statements below about synovial fluid are true, EXCEPT:
A. Arthrocentesis is performed only to evaluate arthritis.
B. Synovial fluid is present only in movable joints
C. Joint fluid resembles plasma in viscosity
D. Joint fluid has approximately the same glucose as plasma
72. Which of the following crystals is the cause of gout?
A. Uric acid or monosodium urate C. Calcium oxalate
B. Calcium pyrophosphate D. Cholesterol
73.
A. Oxalic acid C. Calcium oxalate
B. Calcium pyrophosphate D. Cholesterol
74. Using compensated polarized microscopy, which synovial fluid crystals appear yellow when observed with the long
axes of the crystals parallel to the optical field?
A. Uric acid B. Ca pyrophosphate C. Cyteine D. Cholesterol
75. The synovial fluid glucose level is low in all of the following types of arthritis, EXCEPT:
A. Septic arthritis C. Hemorrhagic arthritis
B. Inflammatory arthritis D. Gout
76. Neutrophils that contain precipitated rheumatoid factor in their cytoplasm are called:
A. LE cells B. Reiter cells C. Ragocytes D. Macrophages
77. Which test correlates with the severity of hemolytic disease of the newborn?
A. Rh antibody titer of the mother C. Amniotic fluid bilirubin
B. L/S ratio D. Urinary estridiol
78. The best available guide in the assessment of gestational age is the amniotic fluid determination of:
A. Bilirubin B. Uric acid C. Creatinine D. Protein
79. Microviscosity of amniotic fluid is measured by:
A. Thin-layer chromatography C. Spectrophotometer
B. Immunologic agglutination D. Fluorescence polarization
80. Bronchoalveloar lavage may be performed in patients with AIDS to detect presence of:
A. Increased helper T cells C. Kaposi sarcoma
B. Bacterial pneumonitis D. Pneumocystic carinii
81. The most abundant cell seen in a bronchoalveolar lavage is the:
A. Neutrophil C. Macrophage
B. Clilited columnar bronchial epithelial cell D. Lymphocyte
82. The milky fluid that that contains acid phosphatase and proteolytic enzymes in the seminal fluid comes from which
part of the male genitalia?
A. Prostate B. Testis C. Seminal vesicles D. Epididymis
83. To detect the presence of semen, which of the following enzyme activities should be measured?
A. Citric acid B. ALP C. ACP D. AST
84. Seminal vesicles produce:
A. Glucose B. Fructose C. Sucrose D. Mannose
85. Semen specimens should be analyzed:
A. Immediately upon receipt C. After liquefaction
B. Prior to liquefaction D. One hour after collection
86. An abnormal amount of prostatic fluid in a semen specimen will:
A. Lower the pH C. Increase the viscosity
B. Raise the pH D. Decrease the viscosity
87. Which of the following stains is used to determine sperm viability?
A. Eosin B. Methylene blue C. Papanicolaou D. Hematoxylin
88. Serum gastrin levels would be greatest in:
A. Atrophic gastritis C. Pernicious anemia
B. Zollinger-Ellison syndrome D. Cancer of the stomach
89. The preferred stimulant of gastric acidity for routine analysis is:
A. Histamine B. Histalog C. Pentagastrin D. Insulin
90. The gastric stimulant used to determine a successful vagotomy procedure is:
A. Insulin B. Pepsin C. Histamine D. Pentagastrin
5

91. A condition in which a patient shows no response to gastric stimulation is:


A. Pernicious anemia B. Zollinger-Ellison syn C. Ulcers D. Dibates
92. It is used as a safer alternative for testing vagal stimulation of gastric acid secretion. The procedure also requires
gastric intubation, however, instead of administering insulin, patients are given a sandwich to chew and spit out.
A. Sweat test B. Sham feeding C. Secretin test D. Breath test
93. The test is based on the measurement of 14CO2 in expired air following the ingestion of various 14C-labeled
triglycerides. Steatorrhea from either pancreatic or other causes results in a decreased absorption of triglycerides
by the digestive system. This in turn results in a decrease in expired CO 2 derived from metabolism of triglyceride
fatty acids.
A. Sweat test B. Sham feeding C. Secretin test D. Breath test
94. It is a valuable test for the differential diagnosis of malabsorption. In this procedure, a 25 g dose of pentose is
administered orally. Blood level is determined two hours later; urine excretion over a five-hour postadministration
period is also determined.
A. D-xylose test B. Sham feeding C. Secretin test D. Breath test
95. Most common cause of malabsorption in developed countries:
A. Celiac disease B. Cystic fibrosis C. Lactase deficiency D. None of these
96. It serves as the laboratory procedure for the definitive diagnosis of steatorrhea:
A. Titrimetric method of Van de Kamer C. Breath test
B. D-Xylose test D. Sweat test
97. Which of the following pairings of stool appearance and cause does not match?
A. Black, tarry: blood C. Pale, frothy: steatorrhea
B. Yellow-green: barium sulfate D. Yellow-gray: bile duct obstrcution
98. In the Apt test, maternal hemoglobin will produce a:
A. Pink solution
B. Yellow-brown precipitate after standing for 2 minutes
C. Yellow-brown supernatant after standing for 2 minutes
D. Red-brown supernatant after standing for 2 minutes
99. When testing for occult blood, the portion of the stool specimen used is:
A. The outer surface B. An emulsion C. The center D. The very end
100. Due to variability in bowel habits, the most representative timed fecal sample is:
A. 2-day collection B. 3-day collection C. 4-day collection D. 5-day collection

END OF EXAM
5

PRE-BOARD KEY (PART 1)


CLINICAL MICROSCOPY

1. B 26. B 51. A 76. C


2. C 27. C 52. D 77. C
3. D 28. C 53. D 78. C
4. B 29. A 54. C 79. D
5. B 30. B 55. C 80. D
6. B 31. A 56. C 81. C
7. A 32. D 57. A 82. A
8. B 33. C 58. A 83. C
9. A 34. D 59. C 84. B
10. C 35. B 60. B 85. C
11. B 36. B 61. A 86. A
12. C 37. C 62. B 87. A
13. B 38. B 63. B 88. B
14. A 39. C 64. A 89. C
15. B 40. C 65. B 90. A
16. B 41. B 66. B 91. A
17. B 42. D 67. D 92. B
18. D 43. B 68. B 93. D
19. B 44. B 69. A 94. A
20. C 45. B 70. C 95. A
21. A 46. A 71. C 96. A
22. B 47. C 72. A 97. B
23. A 48. B 73. B 98. C
24. A 49. A 74. A 99. C
25. C 50. A 75. C 100. B
PRE-BOARD EXAMINATION IN CLINICAL MICROSCOPY (PART 2)

1. In what sequence does urine formation occur?


A. Proximal convoluted tubule, loop of Henle, distal convoluted tubule, collecting tubule
B. Glomerulus, Bowman’s space, proximal convoluted tubule, loop of Henle, distal convoluted tubule, collecting
duct
C. Bowman’s space, glomerulus, proximal convoluted tubule, loop of Henle, distal convoluted tubule, collecting
duct
D. Glomerulus, proximal convoluted tubule, distal convoluted tubule, loop of Henle, Bowman’s space
2. While performing urinalysis, a technologist notices that the urine has a fruity odor. This patient’s urine most likely
contains:
A. Acetone B. Bilirubin C. Coliform bacilli D. Porphyrin
3. Renal clearance tests are used to evaluate which of the following parameters?
A. Concentrating ability C. Glomerular filtration rate
B. Tubular reabsorption D. Glomerular permeability
4. Which renal function is assessed using specific gravity and osmolality measurements?
A. Tubular excretion ability C. Concentrating ability
B. Glomerular filtration ability D. Tubular secretion ability
5. The color of normal urine is mainly due to the pigment:
A. Bilirubin B. Urobilin C. Urochrome D. Uroerythrin
6. To quantify formed elements such as white blood cells, red blood cells, and casts in a 12-hour urine specimen,
which of the following procedures is used?
A. Guthrie count B. Rothera count C. Addis count D, Folin-Lowry count
7. Which of the following pigments will deposit on urate and uric acid crystals to form a precipitate described as “brick
dust”?
A. Bilirubin B. Urobilin C. Uroerythrin D. Urochrome
8. A white precipitate in normal alkaline urine is most likely caused by:
A. Uric acid crystals C. Amorphous phosphate
B. Radiographic contrast media D. Amorphous urates
9. With which crystal are urinary uric acid crystals often confused?
A. Calcium pyrophosphate C. Calcium oxalate
B. Cystine D. Leucine
10. The method of choice for performing a specific gravity measurement of urine following administration of x-ray
contrast dyes is:
A. Reagent strip B. Refractometer C. Urinometer D. Densitometer
11. Which of the following methods for determining the urine’s specific gravity does NOT detect the presence of urinary
protein or glucose?
A. The urinometer method C. The refractometer method
B. The falling drop method D. The reagents trop method
12. The “double indicator system” employed by commercial reagent strips to determine urine pH uses which two
indicator dyes?
A. Methyl orange and bromphenol blue C. Methyl red and bromthymol blue
B. Phenol red and thymol blue D. Phenolphthalein and litmus
13. Most often, the predominant protein excreted in excess in urine is:
A. Lactoferrin B. Albumin C. -globulin D. Globulin
14. Bence Jones protein is:
A. An abnormal protein associated with UTI
B. An abnormal protein associated with hemolysis
C. An abnormal globulin associated with multiple myeloma
D. A normal serum protein
15. Which of the following aids in the differentiation of hemoglobinuria and hematuria?
A. Urine pH C. Microscopic examination
B. Urine color D. Leukocyte esterase test
16. Myoglobinuria is MOST likely to be noted in urine specimens from patients with which of the following disorders?
A. Hemolytic anemias C. Myocardial infarctions
B. Lower UTI D. Paroxysmal nocturnal hemoglobinuria
17. Ammonium sulfate was added to red urine. The urine had a positive reaction for blood, but no RBCs were seen on
microscopic examination. After centrifugation, the supernatant fluid is red. The abnormal color is caused by:
A. Pyridium B. Hemoglobin C. Porphyrins D. Myoglobin
18. Which statement about hemoglobin and myoglobin is true?
A. They are both heme-containing proteins involved in oxygen transport
B. Their presence is suspected when both the urine and the serum are colored red
C. Their presence in serum is associated with high creatine kinase values
D. They precipitate out of solution when the urine is 80 % saturated with ammonium sulfate
19. A false negative urine reaction when using a dipstick pad for hemoglobin can occur after ingestion of:
A. Vitamin A B. Ascorbic acid C. Vitamin B6 D. Cholesterol
20. Which of the following substances if present in the urine will result in a negative Clinitest?
A. Fructose B. Lactose C. Sucrose D. Galactose
21. The glucose specificity of the “double sequential enzyme reaction” employed on reagent strip tests is due to the use
of:
A. Gluconic acid C. Glucose oxidase-peroxidase
B. Hydrogen peroxide D. Neocuproine
22. A 17-year-old girl decided to go on a starvation diet. After one week of starving herself, what substance would most
likely be found in her urine?
A. Protein B. Ketones C. Glucose D. Blood
2

23. Which of the following ketones are NOT detected by the reagent strip or tablet test?
A. Acetone C. B-hydroxybutyrate
B. Acetoacetate D. Acetone and acetoacetate
24. The ketone reagent strip and tablet tests are based on the reactivity of ketones with:
A. Ferric chloride B. Nitroprusside C. Ferric nitrate D. Nitroglycerin
25. Which of the following are characteristic urine findings from a patient with hemolytic jaundice?
A. A positive test for bilirubin and an increased amount of urobilinogen
B. A positive test for bilirubin and a decreased amount of urobilinogen
C. A negative test for bilirubin and an increased amount of urobilinogen
D. A negative test for bilirubin and a decreased amount of urobilinogen
26. Which of the following results are characteristic urine findings from a patient with an obstruction of the bile duct?
A. A positive test for bilirubin and an increased amount of urobilinogen
B. A positive test for bilirubin and a decreased amount of urobilinogen
C. A negative test for bilirubin and an increased amount of urobilinogen
D. A negative test for bilirubin and a decreased amount of urobilinogen
27. The classic Ehrlich’s reaction is based on the reaction of urobilinogen with:
A. Diazotized dichloroaniline C. p-dichlorobenzene diazonium salt
B. p-dimethylaminobenzaldehyde D. p-aminobenzoic acid
28. Red colored-compound produced with the Watson-Schwartz test that is insoluble in chloroform and butanol is:
A. Uroporphyrin C. Urobilinogen
B. Porphobilinogen D. Protoporphyrin
29. Nitrite in urine specimen suggests the presence of:
A. White blood cells C. Bacteria
B. Ammonia D. Urea
30. Which of the following blood cells will NOT be detected by the leukocyte esterase pad because it lacks esterases?
A. Eosinophils B. Lymphocytes C. Monocytes D. Neutrophils
31. Which of the following reagent strip tests can be affected by ascorbic acid, resulting in falsely low or false negative
results?
1. Blood 2. Bilirubin 3. Glucose 4. Nitrite
A. 1, 2 and 3 are correct C. 4 is correct
B. 1 and 3 are correct D. All are correct
32. Which of the following dyes are used in Sternheimer-Malbin stain?
A. Hematoxylin and eosin C. Methylene blue and eosin
B. Crystal violet and eosin D. Crystal violet and safranin
33. The microscopic identification of hemosiderin is enhanced when the urine sediment is stained with:
A. Gram stain B. Hansel stain C. Sudan stain D. Prussian blue stain
34. When a laboratorian performs the microscopic examination of urine sediment, which of the following are
enumerated using the low-power magnification?
A. White blood cells C. Red blood cells
B. Casts D. Renal tubular cells
35. The ova of which parasite may be found in the urinary sediment?
A. Trichomonas vaginalis C. Entamoeba histolytica
B. Schistosoma haematobium D. Trichuris trichiura
36. A technologist is having trouble differentiating between red blood cells, oil droplets, and yeast cells on a urine
microscopy. Acetic acid should be added to the sediment to:
A. Lyse the yeast cells C. Dissolve the oil droplets
B. Lyse the red blood cells D. Crenate the red blood cells
37. Glitter cells are a microscopic finding of:
A. Red blood cells in hypertonic urine C. White blood cells in hypertonic urine
B. Red blood cells in hypotonic urine D. White blood cells in hypotonic urine
38. Urine sediment could have which of the following formed elements and still be considered “normal”?
A. 5 to 10 red blood cells C. A few bacteria
B. 0 to 2 hyaline casts D. A few yeast cells
39. Urinary casts are formed in the:
A. Distal tubules and loops of Henle C. Proximal and distal tubules
B. Distal and collecting tubules D. Proximal tubules and loops of Henle
40. Urinary casts are formed with a core matrix of:
A. Albumin C. Bence Jones protein
B. Tamm-Horsfall mucoprotein D. Transferrin
41. Which of the following crystals, when found in the urine sediment, most likely indicates an abnormal metabolic
condition?
A. Calcium oxalate C. Triple phosphate
B. Bilirubin D. Uric acid
42. Oval fat bodies are derived from:
A. Renal tubular epithelium C. Degenerated WBCs
B. Transitional epithelium D. Mucoprotein matrix
43. In a patient with nephrotic syndrome, the microscopic examination of the urine sediment often reveals:
A. Granular casts C. Red blood cell casts
B. Waxy casts D. Leukocyte casts
44. The most frequently observed cast in urine is:
A. Red cell B. Hyaline C. Waxy D. Fatty
3

45. When using polarized light microscopy, which urinary sediment component exhibits Maltese cross formation?
A. RBC B. Oval fat bodies C. Yeast D. Parasites
46. Which of the following frequently occurs following a bacterial infection of the skin or throat?
A. Acute glomerulonephritis C. Membranous glomerulonephritis
B. Chronic glomerulonephritis D. Rapidly progressive glomerulonephritis
47. Phenylketonuria is an inherited metabolic disease in which there is deficiency of an enzyme:
A. Phenylalanine oxidase C. Phosphoalanine hydroxylase
B. Phenylalanine hydroxylase D. Phosphoalanine phosphatase
48. Which of the following disorders is characterized by the urinary excretion of large amounts of arginine, cystine,
lysine, and ornithine?
A. Cystinosis B. Cystinuria C. Lysinuria D. Tyrosinuria
49. Which of the following hereditary diseases results in the accumulation and excretion of large amounts of
homogentisic acid?
A. Melanuria B. Alkaptonuria C. Phenylketonuria D. Tyrosinuria
50. Which of the following substances oxidizes with exposure to air, causing the urine to turn brown or black?
A. Porphyrin B. Melanin C. Tyrosine D. Urobilinogen
51. A disease that causes large amounts of valine, leucine and isoleucine to be excreted in urine is:
A. Hurler’s syndrome C. Maple syrup urine disease
B. Cystinuria D. Alkaptonuria
52. Increased amounts of 5-hydroxy indoleacetic acid in the urine is indicative of:
A. Platelet disorders C. Malabsorption syndromes
B. Intestinal obstruction D. Argentaffin cell tumors
53. During a lumbar puncture procedure, the first collection tube of CSF removed should be used for:
A. Microbiology studies C. Hematology tests
B. Cytology studies D. Chemistry tests
54. All of the following proteins are normally present in the CSF, EXCEPT:
A. Transferrin B. Albumin C. Prealbumin D. Fibrinogen
55. Pleocytosis is a term used to describe:
A. A pink, orange, or yellow CSF specimen
B. An increased protein content in the CSF owing in cellular lysis
C. Inflammation and sloughing of the cells from the choroids plexus
D. An increased number of cells in the CSF
56. Regarding CSF, all of the following are indications of a traumatic tap, EXCEPT:
A. Clearing of the fluid as it is aspirated C. Xanthochromia
B. A clear supernatant after centrifugationD. Presence of clot in the sample
57. All of the following proteins are normally present in the CSF except:
A. Albumin B. Fibrinogen C. Prealbumin D. Transferrin
58. An unknown fluid can positively identified as being CSF by:
A. Determining the lactate concentration
B. Determining the albumin concentration
C. Determining the presence of oligoclonal banding on electrophoresis
D. Determining the presence of carbohydrate-deficient transferrin on electrophoresis
59. How many leukocytes are normally present in the CSF obtained from an adult?
A. 0-30 cells/uL B. 0-20 cells/uL C. 0-10 cells/uL D. 0-5 cells/uL
60. Which of the following cells may be present in small numbers in normal CSF?
A. Erythrocytes B. Macrophages C. Plasma cells D. Lymphocytes
61. Which of the following cell types predominate in CSF during a classic case of bacterial meningitis?
A. Lymphocytes B. Macrophages C. Monocytes D. Neutrophils
62. Which of the following cell types predominate in CSF during a classic case of viral meningitis?
A. Lymphocytes B. Macrophages C. Monocytes D. Neutrophils
63. A web-like clot or pellicle is formed in the CSF from patients infected with:
A. Neisseria meningitidis C. Mycobacterium tuberculosis
B. Haemophilus influenzae D. Cryptococcus neoformans
64. India ink preparations and microbial antigen tests can aid in the diagnosis of:
A. Bacterial meningitis C. Viral meningitis
B. Tuberculous meningitis D. Fungal meningitis
65. Limulus Lysate test will detect the presence of:
A. Gram-positive bacteria C. Acid-fast organisms
B. Gram-negative bacteria D. All microorganisms
66. Normal CSF lactate levels (below 25 mg/dL) are commonly found in patients with:
A. Bacterial meningitis C. Tuberculous meningitis
B. Fungal meningitis D. Viral meningitis
67. In suspected case of Reye’s syndrome, it is useful to measure:
A. Glucose B. Ammonia C. Lactic acid D. Glutamine
68. Which of the following parameters directly relates to and provides a check of the spermatozoa motility evaluation?
A. The agglutination evaluation C. The morphology assessment
B. The concentration determination D. The viability assessment
69. The concentration of which of the following substances can be used to positively identify a fluid as seminal fluid?
A. Zinc C. Fructose
B. Citric acid D. Acid phosphatase
70. The most common cause of male infertility is:
A. Mumps B. Varicocele C. Malignancy D. Klinefelter’s syndrome
4

71. Which of the following is a characteristic of normal synovial fluid?


A. It forms small clots upon standing C. It is slightly turbid
B. It is dark yellow D. It is viscous
72. A synovial fluid has a high cell count and requires dilution to be counted. Which of the following diluents should be
used?
A. Dilute acetic acid C. Dilute methanol
B. Normal saline D. Phosphate buffer solution
73. Normal joint fluid is viscous because of a high concentration of:
A. -globulin B. Hyaluronate C. Mucin D. Neutrophils
74. Differentiation of synovial fluid crystals, based on their birefringence, is achieved using:
A. Transmission electron microscopy C. Direct polarizing microscopy
B. Compensated polarizing microscopy D. Phase contrast microscopy
75. Which of the following crystals is characteristically seen in patients with gout?
A. Cholesterol crystals
B. Monosodium urate monohydrate crystals
C. Hydroxyapatite crystals
D. Calcium pyrophosphate dihydrate crystals
76. Which crystal causes “pseudogout”?
A. Oxalic acid C. Calcium oxalate
B. Calcium pyrophosphate D. Cholesterol
77. Neutrophils that contain a precipitated rheumatoid factor in their cytoplasm:
A. LE cells B. Reiter cells C. Ragocytes D. Macrophages
78. What is the preferred stimulant of gastric acidity for routine analysis?
A. Histalog B. Histamine C. Insulin D. Pentagastrin
79. A gastric stimulant to determine a successful vagotomy procedure is:
A. Insulin B. Pepsin C. Histamine D. Pentagastrin
80. Serum gastrin levels would be greatest in:
A. Atrophic gastritis C. Pernicious anemia
B. Zollinger-Ellison syndrome D. Cancer of the stomach
81. A condition in which the patient shows no response to gastric stimulation is:
A. Pernicious anemia C. Ulcers
B. Zollinger-Ellison syndrome D. Diabetes
82. Select the term used to describe a decreased volume of amniotic fluid present in the amniotic sac:
A. Ahydramnios C. Hydramnios
B. Oligohydramnios D. Polyhydramnios
83. Amniotic fluid is immediately protected from light to preserve which of the following substances?
A. Phospholipids B. Fetal cells C. Meconium D. Bilirubin
84. Which of the following causes erythroblastosis fetalis?
A. Immaturity of the fetal liver C. Inadequate fetal surfactants
B. Decreased amount of amniotic fluid D. Maternal immunization by fetal antigens
85. The best available guide in the assessment of gestational age is the amniotic determination of:
A. Bilirubin C. Creatinine
B. Uric acid D. Protein
86. Which of the following amniotic fluid measurements is increased in neural tube disorder?
A. Lecithin/sphingomyelin ratio C. Acetylcholinesterase activity
B. Bilirubin D. Phospatidylinositol
87. Which of the following terms is another name for peritoneal fluid?
A. Synovial B. Ascitic C. Pelvic D. Abdominal
88. Thoracentesis refers specifically to the removal of fluid from the:
A. Abdominal cavity C. Peritoneal cavity
B. Pleural cavity D. Pericardial cavity
89. A pleural or peritoneal fluid amylase level 2 times higher than the serum amylase level may be found in effusions
resulting from:
A. Hepatic cirrhhosis C. Rheumatoid arthritis
B. Pancreatitis D. Lymphatic obstruction
90. Bronchoalveolar lavage is useful in the detection of:
A. Tuberculosis C. Malignant melanoma
B. Pneumocystis carinii D. Klebsiella pneumoniae
91. The most abundant cell seen in a BAL is:
A. Neutrophil C. Macrophage
B. Bronchial epithelial cell D. Lymphocyte
92. Which of the following tests is required in order to diagnose steatorrhea?
A. Fecal fat C. Fecal occult blood
B. Fecal carbohydrates D. fecal osmolality
93. Which of the following is responsible for the characteristic color of normal feces?
A. Urobilin B. Hemoglobin C. Bilirubin D. Urobilinogen
94. Mass screening for occult blood in the feces is primarily performed to detect:
A. Colorectal cancer C. Ulcer
B. Hemorrhoids D. Malabsorption
95. Which of the following dietary substances can cause a false negative fecal occult blood slide test?
A. Ascorbic acid B. Fish C. Red meat D. Fruits and vegetables
5

96. In the Apt test, maternal hemoglobin will produce a:


A. Pink solution
B. Yellow-brown precipitate after standing for 2 minutes
C. Yellow-brown supernatant after standing for 2 minutes
D. Red-brown supernatant after standing for 2 minutes
97. Due to variability in bowel habits, the most representative timed fecal sample is:
A. 2-day collection C. 4-day collection
B. 3-day collection D. 5-day collection
98. Which of the following is the indicator of choice in fecal occult blood slide tests?
A. Guaiac B. Benzidine C. Orthotoluidine D. Tetramethylbenzidine
99. Yellowish or gray caseous bodies with the size of a pinhead, found in sputum:
A. Cheesy masses C. Broncholiths
B. Bronchial casts D. Dittrich plugs
100. Eosinophils and Curschmann’s spirals may be found in which of the following conditions:
A. Bronchiectasis C. Pneumonia
B. Chronic bronchitis D. Bronchial asthma

END OF THE EXAM


5

PRE-BOARD KEY (PART 2)


CLINICAL MICROSCOPY

1. B 26. B 51. C 76. B


2. A 27. B 52. D 77. C
3. C 28. B 53. D 78. D
4. C 29. C 54. D 79. A
5. C 30. B 55. D 80. B
6. C 31. D 56. C 81. A
7. C 32. D 57. B 82. B
8. C 33. D 58. D 83. D
9. B 34. B 59. D 84. D
10. A 35. B 60. D 85. C
11. D 36. B 61. D 86. C
12. C 37. D 62. A 87. B
13. B 38. B 63. C 88. B
14. C 39. B 64. D 89. B
15. C 40. B 65. B 90. B
16. C 41. B 66. D 91. C
17. D 42. A 67. D 92. A
18. A 43. B 68. D 93. A
19. B 44. B 69. D 94. A
20. C 45. B 70. B 95. A
21. C 46. A 71. D 96. C
22. B 47. B 72. B 97. B
23. C 48. B 73. B 98. A
24. B 49. B 74. B 99. D
25. C 50. B 75. B 100. D
SEM100 - MTAP100 - MLSCI 100 | MEDICAL LABORATORY SCIENCE INTERNSHIP

ANALYSIS OF URINE AND BODY FLUIDS


Roderick Dasco Balce, RMT December 6, 2021

OUTLINE C. TYPES OF URINE SPECIMEN/COLLECTION


I. Introduction to Urinalysis V. CSF TECHNIQUES
II. Physical Examination of VI. Synovial Fluid
Urine VII. Serous Fluid First Morning
III. Chemical Examination of VIII. Semen ● Routine screening
Urine IX. Feces → Most concentrated specimen of the day
IV. Microscopic Examination INDEX: APPENDIX ● Pregnancy test
of Urine → Specimen of choice for pregnancy testing to prevent
false-negative results
I. INTRODUCTION TO URINALYSIS ● Establish a diagnosis for orthostatic or postural proteinuria
● Ultrafiltrate of plasma → Benign type of proteinuria as a result of long periods of
→ Relative concentrations are not similar to plasma standing
● Urine formation processes: reabsorption and secretion → Disappears when patient assumes a lying or recumbent
→ Two processes why plasma and urine are not identical position
in composition. → Diagnosis requires two urine specimens:
A. COMPOSITION OF URINE ▪ Random specimen: (+)
▪ First morning specimen: (-)
● 95% water, 5% solutes
→ Organic components - urea, creatinine, uric acid, Random
ammonia, undetermined nitrogen, others ● Routine screening
▪ Most abundant (50%) - urea ● Note: the laboratory may not accept a urine specimen that
→ Inorganic components - chloride, Na, K, Ca, is too dilute
phosphates, sulfates → May cause false-negative results
▪ Most abundant - chloride (in form of NaCl salts)
Timed specimens
B. URINE VOLUME ● 24-hour
Daily Output → Start and end with an empty bladder
→ All successive voidings should be collected within 24
● Daily urine output → amount of urine needed in 24 hours
hours
● Daily output:
→ At the end of the collection period, the patient voids and
→ Random: 600-2000 mL per day
is added to the total volume
▪ Encompasses all states of hydration
→ Used for quantitative tests, hormone studies, and
→ Average: 1200-1500 mL per day
clearance tests (measurement of GFR)
→ D/N ratio: 2-3:1
● 12-hour
▪ Nocturia → increased urine volume at night
→ Used to be specimen for Addis count
Polyuria ▪ Quantitation of formed elements (WBC, RBC, casts)
● Abnormal increase in urine output in a 12-hr specimen preserved with formalin
● Seen in diabetes mellitus, diabetes insipidus − Formalin is an excellent sediment preservative
→ SG of DI is low (hyposthenuria) ● 2-hour afternoon specimen
▪ Due to failure of water reabsorption → Indicated for urobilinogen determination
▪ Low ADH levels or inactive ADH → Recommended time of collection is 2-4 PM
▪ Recall central vs. nephrotic DI ▪ Studies show that urobilinogen levels reach their
→ SG of DM is high (hypersthenuria) peak during this time of the day
▪ Due to osmotic diuresis → May be important in the differential diagnosis of
▪ Note: despite high SG in DM, the urine may be jaundice
colorless → Container must be amber, or transparent (must be
● Volume: > 2.5L/day protected from light)
▪ Urobilinogen is photosensitive
Oliguria
● Abnormal decrease in urine output Specimens for diabetic screening/monitoring
● Associated with dehydration, renal insufficiency, heart ● Fasting/second morning specimen
disease, calculi, kidney tumors → First morning specimen may contain metabolites from
→ Causes: food ingested prior beginning of the fasting period
▪ Decreased renal blood flow → Second specimen is free of metabolites
− If blood volume or cardiac output is low, so will be ● 2-hr postprandial
the renal blood flow, leading to oliguria → Detects glucosuria
▪ Partial obstruction ● GTT urine
→ Used as accompaniment to serial blood collections in
Anuria/Anuresis
routine OGTT
● Total suppression of urine production → No longer included in the revised guidelines
● Associated with severe acute nephritis, mercury ● Important parameters for diabetic screening include
poisoning, obstructive uropathy, kidney failure glucose, ketones, and protein
→ Total obstruction and cessation of urine production
Drug testing specimen
Nocturia
● Requires strictest protocols for collection and processing
● Excretion of > 500 mL urine at night with a SG of < 1.018 → Involves legal implications
● Reduction of bladder capacity resulting from pregnancy or ● Volume: 30-45 mL
calculi, or be related to prostate enlargement
● May be physiologic or pathologic
INTERNSHIP GGGL, AJDM, JAGT, KMBG, CTFD; vidrec: CME BSMLS 2022 1 of 16
● Take into consideration for any possible adulteration: Preserves glucose and sediments well;
→ Acceptable temperature: 32.5-37.7०C Thymol interferes with acid precipitation tests
→ Collected within 4 minutes and should be witnessed for protein
→ Bluing agents should be added to toilet water reservoirs Excellent sediment preservative
if not witnessed Ideal for samples to be subjected to
● Follow chain of custody (COC) Addis count
→ Documentation of every step in the procedure of drug Formalin
Interferes with reagent strip (Glucose,
testing
blood and LE)
Midstream clean-catch Do not add before chemical exam
● Routine screening, bacterial culture Good preservative for drug analysis
● Least traumatic Interferes with reagent strip (Glucose,
● Detection of UTI and patient is amenable Sodium fluoride
blood and LE)
Catheterization Do not add before chemical exam
● Acceptable for bacterial culture Preserves protein and formed elements
well
Suprapubic aspiration
May be found in grey-top and is used
● Routine screening, bacterial culture Boric acid for culture sensitivity
● Ideal for cytology studies
● Traumatic procedure due to direct collection from bladder Interferes with drug and hormone
analysis
Prostatitis specimens Saccomanno Preserves cellular elements for cytology
● Three-glass collection fixative studies
→ All portions are collected
→ WBC and bacterial counts in tube 3 must be 10x higher E. CHANGES IN UNPRESERVED URINE ( > 2h)
than in tube 1 ● Many of the changes are related to bacterial growth
→ Tube 1: Bladder and urethral elements → Odor, pH, nitrite, clarity, glucose, and ketones are
→ Tube 2: Control for kidney and bladder infection affected due to the exponential growth of bacterial
▪ Midstream clean catch colonies
▪ Tube 2 should not contain significant elements INCREASE
▪ Pyelonephritis and cystitis cannot be ruled out if tube Usually darkens, can change into
2 & 3 have similar elements Color
another color due to oxidation
→ Tube 3: Urethral, bladder, and prostatic elements Odor Becomes ammoniacal
● Four-glass collection Formation of ammonia from urea; loss
→ Also called Stamey-Mears pH
of CO2
→ Consists of
May increase if nitrate-reducing bacteria
▪ Initial voided urine Nitrite
are present
▪ Midstream urine
▪ Expressed prostatic secretion Bacteria Growth multiplies
− Collected while massage is carried out Most crystals Precipitates when standing at low
▪ Post-prostatic massage urine specimen temperatures

D. SPECIMEN COLLECTION, HANDLING, DECREASE


AND PRESERVATION Precipitation of amorphous crystals and
Clarity
● Specimen container for routine urinalysis multiplication of bacteria
→ Capacity of 50 mL Undergoes glycolysis for bacterial
Glucose
→ Enough to contain volume for urinalysis and enough metabolism
room for mixing (important step prior to performing Metabolized by bacteria
Ketones
examination) Undergo volatilization
● Minimum labeling requirements Bilirubin & Photosensitive substances
→ Patient’s full name, date and time of collection urobilinogen
● All specimens must be analyzed within 2 hours of Cells, casts, Dissolves in alkaline urine
collection T vaginalis
→ Age of specimen is an important factor
II. PHYSICAL EXAMINATION OF URINE
● If analysis will be delayed, refrigerate or use chemical
preservatives A. COLOR
Preservatives Comments ● Normal - colorless, straw or light yellow to dark yellow,
Bacteriostatic for 24 hours; maintains yellow-orange, or amber
acid pH up to 8; → Yellow shade is due to the endogenous pigment,
Does not interfere with tests urochrome
● Factors to consider:
Possible precipitation of amorphous → State of hydration
urates and phosphates that may cause → Any color can be considered normal if etiology is from
Refrigeration
turbidity; obscure more clinically food or medication
significant elements ▪ Majority of the variations in color are a result of
Remedy (if precipitated): medications in cases of nonpathologic causes
Phosphate - add acetic acid Variations
Urates - warm specimen Color Nonpathologic Pathologic causes
Causes odor change, mask odor causes
Phenol
Not interfere with routine dipstick Colorles Recent fluid Related to polyuria
Floats on surface and clings to pipettes s/ pale consumption, DM (osmotic diuresis)
Toluene yellow alcohol or diuretic DI
Not interfere with routine dipstick
intake

INTERNSHIP MTAP 5: ANALYSIS OF URINE AND BODY FLUIDS BSMLS 2022 2 of 16


Amber/ Acriflavine, Bilirubinuria (obstructive Methods
orange phenazopyridine, jaundice)
1. Urinometry
nitrofurantoin,
→ Based on density measurement; uses a weighted float
phenindione
that will displace a volume of liquid equal to its weight
Blue/ Amitriptyline, Pseudomonas infection ▪ Designed to sink to a level of 1.00 in distilled water
green methocarbamol, (natural pigment → Calibrated with distilled water at 20°C; Requires 15mL
clorets, production; pyocyanin, of urine
methylene blue, pyoverdin; biofilm producer) → Disadvantage:
phenol (oxidized), Indicanuria (Blue Diaper ▪ Require a large volume of the specimen
indigo carmine syndrome; indican is ▪ Less accurate than other method, no longer
normally a colorless recommended
substance, but turns blue → Corrections:
when oxidized) ▪ Temperature
Pink/ Beets, rifampin, Hemoglobinuria/ − Add or subtract 0.001 for every 3°C difference
red menstrual Myoglobinuria: Clear from the calibration temperature
contamination Hematuria: Cloudy/turbid − Subtract if lower than 20°C (only applicable if
refrigerated); add if higher than 20°C
Porphyria: Port wine red ▪ Protein (if the levels are high)
(porphyria cutanea tarda − Subtract 0.003 for every g/dL
and erythropoietic porphyria ▪ Glucose (if the levels are high)
or increased excretion of − Subtract 0.004 for every g/dL
uroporphyrins) ▪ Dilution
Brown/ Argyrols, Alkaptonuria: homogentisic − Multiply the last 2 digits by the dilution factor
Black methyldopa, acid, seen in cases of 2. Refractometry
levodopa, increased pH → Refractive index measurement
metronidazole, Melanuria: becomes black → Most commonly used principle by automated machines
phenol derivatives when exposed to air → Compensated between 15-38°C
▪ Renders temperature correction unnecessary
Methemoglobin: lysed
→ Subject to interference by radiographic contrast media
RBCs lead to accumulation
▪ Will increase specific gravity beyond range (1.040)
of methemoglobin
→ Calibrating media:
B. TRANSPARENCY ▪ Distilled water: 1.000 + 0.001
● Normal ▪ 5% NaCl: 1.022 + 0.001
→ Clear - no visible particulates, transparent ▪ 9% Sucrose: 1.034 + 0.001
● Variations ▪ Recalibrate instrument if out of range
→ Hazy - few particulates, print easily seen through urine → Corrections:
→ Cloudy - many particulates, print blurred through urine ▪ Perform glucose and protein corrections
→ Turbid - print cannot be seen through urine ▪ No need to correct temperature
→ Milky - may precipitate or be clotted 3. Harmonic Oscillation Densitometry
● Variations in clarity may be considered normal as long as → Density measurement based on the change in the
the cause is nonpathologic frequency of sound waves oscillating through urine
→ Soundwaves of specific frequency are generated at
C. ODOR one end of a tube and as waves oscillate through urine,
● Not routinely reported unless odor is peculiar and may be the frequency is altered by the density of SG of the
indicative of a disorder specimen
● Normal → Previously used by Yellow IRIS
→ Faint aromatic due to volatile acids ▪ No longer utilized
→ Becomes ammoniacal as the specimen stands
→ If freshly collected and smells ammoniacal, may be III. CHEMICAL EXAMINATION OF URINE
indicative of UTI A. SPECIFIC GRAVITY
● Variations ● Random urine reference range: 1.002-1.035
→ Odorless: Acute tubular necrosis
→ Mousy: Inborn error of metabolism (phenylketonuria) Clinical Significance
→ Rancid: Tyrosinemia ● Hypersthenuria: persistently high SG (above 1.010)
→ Sweaty feet: Isovaleric acidemia ● Hyposthenuria: persistently low SG (below 1.010)
→ Maple syrup: MSUD ● Isosthenuria: 1.010 (fixed at 1.010)
→ Rotting fish: Trimethylaminuria or aminoaciduria → 1.010 is the SG of the glomerular filtrate at the level of
→ Sulfur: Cystinuria (Fanconi syndrome) the bowman's capsule
→ Fruity/sweet: Diabetic ketoacidosis → Kidneys are no longer able to modify the glomerular
→ Cabbage: Methionine malabsorption filtrate: Renal failure, DI, ATN
→ Fecaloid: Recto-vesicular fistula, fecal contamination ● Note: Reagent strip detects ionized solutes only
→ Bleach: Contamination with bleach or semen Test Specific Gravity
→ Mercaptan: Asparagus, garlic, eggs
Principle pKa change of polyelectrolytes
D. SPECIFIC GRAVITY Reagents Polyelectrolytes
● Normal values: Bromthymol blue (pH indicator)
→ Random: 1.002-1.035 Color and Blue-green-yellow in increasing SG
▪ Wide range as it encompasses all states of hydration Reading time 45 seconds
→ 24-h (Average): 1.015-1.025 Sensitivity/ 1.000-1.030
Detection limit (1.000 not physiologically possible)
Sources of F(+): high concentrations of protein
interference F(-): highly alkaline urine

INTERNSHIP MTAP 5: ANALYSIS OF URINE AND BODY FLUIDS BSMLS 2022 3 of 16


B. pH 4. Post-renal
● Random: 4.5-8.0 → Addition of proteins as urine passes through the lower
● Fasting/first AM: 5-6 (acidic or slightly acidic) urinary tract (ureters, bladder, urethra)
→ Due to slight hypoventilation during sleep → Lower UTI, injury, trauma. urethritis
Clinical Significance SSA Test/Cold Protein Precipitation
● Acid urine: ● Traditional test based on cold protein precipitation
a. Hypoventilation ● Protein is precipitated by an acid
b. Diabetic ketoacidosis ● Procedure:
c. Diarrhea/dehydration → 3mL of 3% SSA added to 3mL centrifuged urine
d. Acid-producing bacteria → Centrifuging the urine sample eliminates a false source
e. High protein diet of turbidity such as albumin and globulins
f. Medications Degree of Turbidity Grade mg/dL
● Alkaline urine: No increase in turbidity − <6
a. Hyperventilation Noticeable turbidity ± 6-30
b. RTA Distinct turbidity, no granulation 1+ 30-100
▪ Acidemia
Turbidity with granulation 2+ 100-200
▪ Due to failure of hydrogen ion secretion, bicarbonate
ions cannot be absorbed Turbidity with granulation and 3+ 200-400
c. Vomiting and after meals flocculation
▪ Transient alkaline tide Clumps of protein 4+ > 400
▪ Withdrawal of hydrogen ions from the blood to make Test Protein
HCl for digestion Principle Protein error of indicators
d. Urease-producing bacteria (indicators change color due to presence
▪ Urea to ammonia by bacteria of proteins)
e. Vegetarian diet Reagents Tetrabromphenol blue or
f. Old specimen (> 2 hours old) tetrachlorophenol
Test pH tetrabromosulfonphthalein
Principle Double indicator system Buffer (pH 3.0) → prevents pH change
Reagents Methyl red Color and Shades of green to blue
Bromthymol blue Reading time 60 seconds
Color and Red-yellow-blue in increasing pH Sensitivity/ Multistix: 15-30 mg/dL
Reading time 60 seconds Detection limit Chemstrip: 6 mg/dL
Sensitivity/ Multistix: 5-8.5 Sources of F(+): increased pH, pigmented
Detection limit Chemstrip: 5-9 interference specimens, chlorhexidine,
Sources of F(+): Old specimens phenazopyridine, QACs (detergents),
interference F(-): Runover from adjacent pads antiseptics, loss of buffer (due to
overexposure), increased SG
C. PROTEIN
F(-): presence of proteins other than
● <30 mg/dL or <150 mg/day (may be slightly higher due to albumin (globulins), high salt
orthostatic protein) concentration, microalbuminuria
● Normal urine does not contain significant amounts of high
molecular weight proteins D. GLUCOSE
Clinical Significance: Types of Proteinuria ● < 15 mg/dL normally present in urine
→ Reagent strip will not detect this amount of glucose
1. Pre-renal/Overflow Proteinuria ● Renal threshold for glucose: 160-180 mg/dL
→ Increase in LMW protein in the blood
▪ IV hemolysis (hemoglobin) Clinical Significance: Types of Glucosuria
▪ Muscle injury (myoglobin) ● If glucose concentration exceeds the renal threshold;
▪ Infection and inflammation (acute phase reactants) glucose will be secreted and will no longer be reabsorbed
▪ Multiple myeloma (Bence-Jones Protein) 1. Hyperglycemia-associated
− BJP has thermal sensitivity → Overflow, due to problematic glucose levels in the
− Light chain component of IgG blood
− Precipitates at 40-60°C, disappears at 100°C, → DM, endocrine disorders, pancreatic disorders, GDM,
reappears upon cooling Cushing’s syndrome
− Detected by immunoelectrophoresis 2. Renal-associated
→ Myoglobin will not stay in the plasma because there is → Failure of glucose reabsorption
no binding protein for myoglobin → Renal tubular dysfunction, tubular necrosis, Fanconi
▪ Myoglobin ⇒ Urine: red; Plasma: clear syndrome, osteomalacia, pregnancy
▪ Hemoglobin ⇒ Urine: red; Plasma: red Tests for Glucose (Copper Reduction)
2. Renal (glomerular)
→ Diabetic nephropathy, amyloidosis, glomerulonephritis, ● Principle:
autoimmune disorders, toxic agents, hypertension, → Glucose reduce cupric ions to cuprous ions
strenuous exercise, pre-eclampsia, dehydration, ● Benedict’s test
orthostatic proteinuria → Reagent: copper sulfate
→ Earliest indicator of diabetic nephropathy is → Positive result: Brick-red precipitate 4+
microalbuminuria (Micral test) ● Clinitest
3. Renal (tubular) → Reagents:
→ Fanconi syndrome, toxic agents, severe viral infections 1. Copper sulfate: source of cupric ions
▪ Fanconi syndrome → characterized by failure of 2. Sodium hydroxide, sodium citrate: heat production
reabsorption, causing inappropriate substances to 3. Sodium carbonate: removes room air interference
appear in urine → Positive result: Orange-red color

INTERNSHIP MTAP 5: ANALYSIS OF URINE AND BODY FLUIDS BSMLS 2022 4 of 16


● Sources of errors:
F. BLOOD
→ False-positive:
▪ Presence of reducing agents other than glucose Clinical Significance
− Ascorbic acid, drug metabolites ● Hematuria (smokey, cloudy red urine)
→ False-negative: → Microscopic hematuria: Renal calculi
▪ Pass-through phenomenon → Macroscopic hematuria: Glomerular disorders,
− When the orange color reverts back to green pyelonephritis, tumors, trauma, toxic chemicals,
▪ Occurs when glucose levels are too high anticoagulant therapy
− Remedy: reduce number of drops of glucose → Positive result: speckled pattern on reagent strips
(from 5 drops to 2 drops) ▪ Due to the presence of intact RBCs
Test Glucose ● Hemoglobinuria (clear, red urine)
Principle Glucose oxidase reaction/Double → Hemolytic anemias, transfusion reactions, severe
sequential enzyme reaction burns, malaria
Reagents GOD, POD → Positive result: homogenous or uniform pattern
KI (multistix) or ● Myoglobinuria
tetramethylbenzidine(chemstrip) → Conditions associated with rhabdomyolysis
→ Muscular trauma, crush syndrome, convulsions,
Color and Multistix: green-brown
muscle-wasting diseases, alcoholism, extensive
Reading time Chemstrip: yellow-green
exertion
30 seconds
→ Positive result: homogenous or uniform pattern
Sensitivity/ Multistix: 75-115 mg/dL
Detection limit Chemstrip: 40 mg/dL Myoglobin vs Hemoglobin
Sources of F(+): strong oxidizing agents (detergents) ● Plasma appearance:
interference F(-): reducing agents (ascorbic acid, → Myoglobin: Normal colored (straw/yellow)
ketones, drug metabolites), low ▪ Recall that there is no scavenging mechanism for
temperature, old specimen myoglobin
● Reducing agents produce: → Hemoglobin: Red/pink
→ False positive result in Clinitest ▪ Due to hemoglobin binding with haptoglobin in the
▪ Based on reduction plasma
→ False negative in reagent strip of glucose ● Ammonium sulfate method (Blondheim test)
▪ Based on oxidation → Procedure:
▪ Add 2.8g (NH4)2SO4 to 5 mL urine
E. KETONES ▪ Centrifuge
● Accumulation of ketones is due to abnormal fat catabolism → Red precipitate: hemoglobinuria
Clinical Significance ▪ Hemoglobin is precipitated by (NH4)2SO4
● Ketone bodies: ▪ After centrifugation, the supernatant will not produce
→ B-hydroxybutyric acid: 78% a positive result in the reagent strip
▪ Not detected by routine reagent strip testing → Red supernatant: myoglobinuria
→ Acetoacetic acid: 20% ▪ The reagent strip test will still be positive if the test is
→ Acetone: 2% repeated after centrifugation
● Reagent strip only detects acetoacetic acid and acetone Test Blood
● Causes of ketonuria: Principle Pseudoperoxidase activity of heme
→ Diabetes mellitus (DKA), starvation, fasting, weight Reagents Lysing agent
reduction, strenuous exercise, malabsorption, Peroxide (OA)
pancreatic disorders, inborn errors of amino acid Tetramethylbenzidine
metabolism Color and Blue-green
Tests for Ketones: Reading time Speckled or uniform appearance
● Acetest 60 seconds
→ Counterpart of Clinitest Sensitivity/ 5-20 RBCs/mL
→ Reagents: Detection limit
▪ Sodium nitroprusside: main reagent Sources of F(+): Strong oxidizing agents, bacterial
▪ Glycine: allows detection of acetone interference peroxidases, menstrual contamination
→ Positive result: purple color F(-): high SG, crenated cells, nitrite,
● Gerhardt’s, Lindeman’s → diacetic acid (same name as formalin, captopril, ascorbic acid
acetoacetic acid)
G. BILIRUBIN
● Legal’s, Rothera → diacetic acid and acetone
● Osterber’s, Hart’s → B-hydroxybutyric acid Clinical Significance
Test Ketones ● Bilirubinuria
Principle Sodium nitroprusside reaction → Hepatitis, cirrhosis, other liver disorders, and biliary
Reagents Sodium nitroprusside ± glycine obstruction
W/ glycine: Chemstrip → Hepatic and obstructive conditions result in bilirubinuria
W/out glycine: Multistix Differential diagnosis of jaundice
Color and Purple Urine Bilirubin Urine UBG
Reading time 40 seconds Hemolytic Negative 3+
Sensitivity/ Multistix: 5-10 mg/dL acetoacetic acid (B1)
Detection limit Chemstrip: 9 mg/dL acetoacetic acid Hepatic +/- 2+
70 mg/dL acetone (B1/B2)
Sources of F(+): phthalein dyes, red urine, levodopa, Obstructive 3+ Negative
interference medications (sulfhydryl groups) (B2) (Normal strip test)
F(-): old specimen, improperly preserved
specimen

INTERNSHIP MTAP 5: ANALYSIS OF URINE AND BODY FLUIDS BSMLS 2022 5 of 16


Tests for bilirubin → Ehrlich-reactive compounds
▪ Only butanol layer will be penetrated
1. Foam shake test ▪ Adding PDAB will cause the urine layer and the
→ Need to rule out phenazopyridine (produce yellow butanol layer to turn cherry-red
foam)
2. Oxidation test (Gmelin’s/Fouchet’s test) Solubility
Substrate Red color in
→ Tests for bile pigments Chloroform Butanol
▪ Based on acidic oxidation of bile pigments Urobilinogen Chloroform layer
Soluble Soluble
→ (+) rainbow arrow of colors Butanol layer
▪ Green (biliverdin) Porpho- Urine-acetate
Insoluble Insoluble
▪ Blue (bilicyanin) bilinogen mixture
▪ Yellow (choletelin) Ehrlich- Urine-acetate
3. Diazotization test (Icotest) reactive subs. Insoluble Soluble layer
→ Counterpart of Acetest: ketones; Clinitest: glucose Butanol layer
→ Confirmatory test for bilirubin See appendix for diagram
▪ 0.05 mg/dL sensitivity for bilirubin Additional reading From AUBF trans (2021)
→ Reagents: p-nitrobenzene-diazonium-p-toluene ● Watson Schwartz Test (better than Hoesch)
sulfonate, Na2CO3, boric acid, SSA → Equal parts (≈2 mL each) of urine and Ehrlich’s reagent
→ (+) result: blue-purple color are mixed in a tube
Test Bilirubin → Add saturated sodium acetate (≈4 mL), and the solution is
Principle Diazo reaction mixed again
Reagents Multistix: Dichloroaniline diazonium salt → A positive test results in the development of a
Chemstrip: dichlorobenzene diazonium characteristic red or magenta color
tetrafluoroborate ▪ Means that urobilinogen, porphobilinogen, and other
Color and Tan, Pink, Violet Ehrlich’s reactive substances may be present
Reading time 30 seconds ▪ If the test yields a no red or magenta color, stop testing
Sensitivity/ Multistix: 0.4-0.8 mg/dL ● Extraction method to detect what specific substance was
Detection limit Chemstrip: 0.5 mg/dL present
→ Extraction is performed by adding chloroform (≈2 to 5 mL)
Sources of F(+): highly pigmented urine, indican,
to the mixture, followed by vigorous shaking
interference phenazopyridine, metabolites of Lodine
→ If the red color resides only in the aqueous phase (the top
F(-): Exposure to light, ascorbic acid,
layer), porphobilinogen or another Ehrlich’s reactive
high concentration of nitrite
substance is present
H. UROBILINOGEN ▪ Proceed to butanol extraction to differentiate the
● < 1.0 mg/dL substances present
● < 1.0 Ehrlich unit → If the red color resides only in the chloroform phase (the
bottom layer), increased amounts of urobilinogen are
Clinical Significance
present.
● Increased urobilinogen ▪ No need to proceed to butanol extraction
→ Hemolytic disorders ● Butanol Extraction
→ Hepatitis → To perform the butanol extraction, the aqueous phase (top
→ Cirrhosis layer) is transferred to another tube and an equal volume
→ Carcinoma of butanol is added
● Never reported as negative in reagent strip test → The tube is shaken vigorously, and the phases are allowed
→ Reagent strip lacks sensitivity to separate
Tests for Urobilinogen → If the red color remains in the aqueous phase (now the
1. Ehrlich’s Tube Test bottom layer), porphobilinogen is present
→ Nonspecific test → If the red color is in the butanol phase (the top layer),
▪ Detects urobilinogens, porphobilinogen, Ehrlich urobilinogen or other Ehrlich’s reactive substances are
reactive compounds (ERC) present
→ Reagent: paradimethylaminobenzaldehyde (PDAB)
→ Positive result: cherry red color Test Urobilinogen
2. Schwartz-Watson Differentiation Test Principle Ehrlich reaction
→ BAsed on the varying solubilities of urobilinogen, Reagents Multistix: PDAB
porphobilinogen, ERC in chloroform and butanol Chemstrip: Methoxybenzene diazonium
→ Perform chloroform and butanol extraction salt
▪ Urobilinogen: soluble in chloroform and butanol Color and Red after 60 seconds
▪ PBG: insoluble in chloroform and butanol Reading time
▪ ERCs: soluble only in butanol Sensitivity/ Multistix: 0.2 mg/dL
→ Utilizes two tubes: 1 containing chloroform, and the Detection limit Chemstrip: 0.4 mg/dL
other containing butanol Sources of F(+): Porphobilinogen, Ehrlich-reactive
▪ Chloroform: urine on top, chloroform at bottom interference compounds, indican, sulfonamides
▪ Butanol: butanol on top, urine at bottom F(-): old specimen, formalin, nitrite
→ Urobilinogen
I. NITRITE
▪ Chloroform and butanol will contain urobilinogen
upon shaking as it is soluble with both substances Clinical Significance
▪ Upon adding PDAB, chloroform and butanol layers ● UTI caused by nitrate-reducing bacteria:
should turn cherry-red → E.g. uropathogens, gram-negative bacilli;
→ Porphobilinogen Enterobacteriaceae; Escherichia coli
▪ Agitation will not allow PBG to penetrate chloroform ● Non-nitrate-reducing bacteria (negative nitrite test)
and butanol, causing it to remain in the urine layers → Gram-positive cocci, Staphylococcus, Enterococcus
▪ Adding PDAB will cause both urine layers to turn ● No. of bacteria detected (sensitivity):
cherry-red → > 100,000 CFU/mL
INTERNSHIP MTAP 5: ANALYSIS OF URINE AND BODY FLUIDS BSMLS 2022 6 of 16
Test Nitrite IV. MICROSCOPIC EXAMINATION OF URINE
Principle Greiss’ reaction A. SEDIMENTATION EXAMINATION TECHNIQUES
Reagents Multistix: p-arsanilic acid THBQ or
Chemstrip: sulfanilamide-hydroxy THBQ Standard Manual Microscopic Examination Technique
Color and Pink ● Volume of urine examined:
Reading time 60 seconds → 10-15 mL (average 12 mL)
Sensitivity/ Multistix: 0.06-0.10 mg/dL ● Centrifugation placed in a conical tube:
Detection limit Chemstrip: 0.05 mg/dL → 400g (RCF) or 1000-1200 rpm for 5 minutes
● Volume of sediment examined:
Sources of F(+): Old specimen
→ 20 mL under a 22x22 mm coverslip
interference F(-): non reductase-containing bacteria,
● Sediment examination:
insufficient contact time*, lack of urinary
→ >10 LPF and >10 HPF averaged
nitrate, bacterial overgrowth, antibiotics,
ascorbic acid, high SG Reporting Format and Reference Values
*First morning specimen is recommended to prevent false (-) ● RBCs, WBCs, RTE: average number per HPF
Urine must be held in the bladder for at least 4 hours ● Casts: average number per LPF
→ Cellular casts should be reported as per HPF instead
J. LEUKOCYTE ESTERASE
→ LPF is for counting; HPF is for identification
Clinical Significance Sed. None Rare Few Mod. Many
● Advantages over nitrite Epithelial 0 0-5 5-20 20-100 >100
→ Detection of bacterial (nitrate-reducing and non-nitrate cells LPF is for squamous, HPF is for TECs
reducing) and non-bacterial UTI (Candida albicans, 0 0-2 2-5 5-20 >20
Trichomonas vaginalis) Crystals
per LPF or HPF
→ Detect inflammation of the urinary tract (acute 0 0-10 10-50 50-200 >200
interstitial nephritis) Bacteria
per HPF
● WBC types detected: granulocytes, monocytes Mucus 0 0-1 1-3 3-10 >10
→ Granules contain the enzyme esterase threads per LPF
Test Leukocyte Esterase Rare, few, Rare, few,
Principle Granulocyte esterase reaction mod, many mod, many
Average Average
Reagents Multistix: amino acid ester (LPF) (HPF) (LPF) (HPF)
Chemstrip: indoxylcarbonic acid ester SEC TEC Casts RBCs
Both: Diazonium salt Crystals Crystals WBCs
Color and Purple Mucus thread Bacteria RTE
Reading time 120 seconds (longest reading time)
Sediment Stains
Sensitivity/ Multistix: 5-10 WBC/HPF
Detection limit Chemstrip: 10-25 WBC/HPF ● Not routinely used, but may be used to enhance
identification of constituents
Sources of F(+): Strong OAs, formalin, highly
interference pigmented urine, nitrofurantoin STAIN Elements/structures stained
F(-): high protein, glucose, oxalic acid, Sternheimer-Malbin WBCs, epithelial cells, and casts
ascorbic acid, antibiotics (Crystal violet +
safranin)
Ascorbic acid
Toluidine blue WBC nuclei
● Some manufacturers include an 11th pad to detect Hansel stain Eosinophilic granules
possible interference (MB and alc. eosin)
● Affects:
Lipid stains Triglycerides and neutral fats in
1. Glucose
(Oil red O/Sudan) cells and casts
2. Blood
3. Bilirubin Hemosiderin granules in cells and
Prussian blue stain
4. Nitrite casts
5. Leukocyte esterase Cellular elements for cytologic
Papanicolaou stain
studies
K. REAGENT STRIP TESTING
Microscopy
● Care of reagent strips: Store at < 30°C with desiccant in
an opaque, tightly closed container ● Manipulate the iris or the rheostat to optimize visualization
● Technique Type Comments
→ Let refrigerated specimens warm to room temperature Routine urinalysis, must use
and mix specimen well before testing Bright-field
subdued light
→ Dip the strip into specimen completely but briefly ~1sec Elements with low RI (hyaline
→ To ensure against run-over, blot edge of the strip on Phase-contrast
cast, mucus threads, T. vaginalis)
absorbent paper and hold the strip horizontally while Cholesterol-containing cells and
comparing it with the color chart Polarizing casts (maltese cross)
→ Compare reaction colors with the manufacturer’s chart Birefringent crystals
under a good light source at the specified time Unstained specimens (eg. T.
● Quality Control Dark-field
pallidum)
→ Test reagent strips with known positive and negative Fluorescence Fluorescent microorganisms
controls
3D microscopy image and
1. Every 24 hours
layer-by-layer imaging
2. When a new bottle is opened
Interference contrast Types:
3. When results are questionable
Hoffman: modulation IC
→ Distilled water should not be used as the negative
Nomarski: differential IC
control, use the controls provided by the manufacturer
→ Positive control may be ± 1 color block

INTERNSHIP MTAP 5: ANALYSIS OF URINE AND BODY FLUIDS BSMLS 2022 7 of 16


B. SEDIMENT CONSTITUENTS ● Factors that encourage formation (“CAST”)
1. Concentration increased (sodium and calcium salts)
Body Cells and Microorganisms 2. Acid pH
Sediment Description 3. Stasis of urine flow
Normal: 0-2/HPF 4. Tamm-Horsefall protein (AKA uromodulin)
Hematuria: glomerulonephritis, renal calculi, 1. Hyaline
malignancy → Prototype of all casts
Ghost cells: seen in dilute, hypotonic urine → Major component: uromodulin/Tamm-Horsefall protein
RBCs → RI: low (same as urine), difficult to demonstrate under
Crenated cells: concentrated/hypertonic
Dysmorphic RBCs: indicative of glomerular brightfield microscope
bleeding; conclusively diagnose glomerular → Normal number: 0-2/LPF
bleeding with RBC casts → Significance of increased number:
Normal: <5/HPF ▪ Nonpathologic: stress, strenuous exercise
Pyuria: urinary tract infection or inflammation ▪ Pathologic: renal disease, congestive heart failure,
Upper UTI: pyelonephritis; demonstrate casts pyelonephritis
Lower UTI: cystitis 2. Cellular
→ When cellular elements are present, they adhere to the
WBCs Neutrophilia: most common
casts’ matrix (casts are known to be sticky), forming
Glitter cells: sparkling; Brownian movement
cellular casts
Eosinophils: allergic reaction or acute
→ Types and significance:
interstitial nephritis (>1% diff count)
▪ RBC – glomerulonephritis, strenuous exercise
Mononuclears: graft rejection
▪ WBC – upper UTI (pyelonephritis), inflammation
Most significant EC type (reabsorptive (acute interstitial nephritis)
properties) ▪ RTE – renal tubular damage
RTECs >2/HPF = tubular injury or tubular necrosis 3. Granular
Large, Oval fat bodies: nephrotic syndrome → In times of urine stasis, cellular elements undergo
eccentric Bubble cells: acute tubular necrosis disintegration within the casts, causing granules to
nucleus Bilirubin-laden: hepatic and obstructive appear throughout the casts
jaundice → Not all granular casts are a result of disintegration
Hemosiderin-laden: hemolytic disorders (nonpathologic)
Singly or in syncytia after urologic procedures → Normal no: 0-1/LPF
TECs eg. catheterization
Central → Significance of increased number:
nucleus Abnormal morphology indicates: ▪ Increased in strenuous exercise
Malignancy or viral infections ▪ Chronic glomerulonephritis and pyelonephritis
Abundant, irregular cytoplasm and a 4. Waxy
prominent central nucleus about the size of → In times of extreme stasis, granular casts may give rise
an RBC to waxy casts that are often broad
SECs Largest cell in the urine sediments → Appearance: with jagged ends and notches; easily
Only significant if it is associated with fragmented
Gardnerella vaginalis (bacterial vaginosis) → RI: high (phase contrast microscope is unnecessary)
Clue cells: SECs with G. vaginalis → Significance:
Gram-neg rods of family Enterobacteriaceae, ▪ Extreme stasis
and Cocci esp Staphylococcus and ▪ Chronic glomerulonephritis
Bacteria Enterococcus spp. ▪ Chronic pyelonephritis
Bacteriuria: indicates UTI when seen in ▪ Renal failure
conjunction with pyuria and a positive culture ▪ End stage renal disease
Small, refractile oval structures 5. Broad waxy cast or “Renal failure casts”
May be mistaken as RBCs (to differentiate: → Not a type of cast but a descriptive term for casts with
presence of budding in yeasts) unusual width
Yeast → All types of casts can form broad casts
Most common agent: Candida albicans
cells → Stasis leads to distention of tubular walls, and
Associated with diabetes mellitus,
immunocompromised state and vaginal formation of broad casts in the collecting duct
moniliasis 6. Cylindroid
T. vaginalis: most commonly encountered; → With tapered end due to formation at the junction of
may resemble a WBC or epithelial cell; ascending Loop of Henle and distal convoluted tubule
causes urethritis; demonstrate characteristic → Same clinical significance as cast, should be reported
motility along with casts
Parasites S. haematobium: ova have a terminal spine; 7. Fatty cast
shed eggs in the urine → Contains lipids; highly refractile and may produce
E. vermicularis: contaminant from the maltese cross pattern if cholesterol is present
perianal, commonly seen in female patients → Seen in nephrotic syndrome and other lipiduria
due to proximity of perianal to vagina Crystals
Casts ● Factors that encourage formation
● When casts are demonstrated, the nephrons are involved 1. Increased solute concentration
● RBCs + RBC casts indicates that bleeding has originated 2. Decreased temperature
from the nephrons, same applies to WBCs 3. Optimal pH
● Sites of formation: ● Identification techniques:
→ Lumen of DCT 1. pH determination → acidic or alkaline
→ Collecting duct 2. Polarizing microscopy → cholesterol-containing and
▪ For chronic renal diseases; can form broad casts birefringent crystals)
3. Solubility characteristic → use solvents to dissolve
crystals
INTERNSHIP MTAP 5: ANALYSIS OF URINE AND BODY FLUIDS BSMLS 2022 8 of 16
● Crystals in Acidic Urine 4. Tyrosine
1. Uric Acid ▪ Fine refractile needles that form clumps or rosettes
▪ Yellow-brown; pleomorphic; birefringent ★ Soluble in alkali and heat
− May be colorless and hexagonal which may be − Ammonium hydroxide
mistaken for cysteine ▪ Severe liver disease or tyrosinemia or infantile
★ Soluble in alkali tyrosinuria
▪ Seen in normal urine, but is clinically significant 5. Bilirubin
when increased in number ▪ Clumped yellow needles or granules
− Associated with gout, leukemia, Lesch-Nyhan ▪ Soluble in “CHASE”
syndrome (orange-sand in diapers) 1. Chloroform
2. Amorphous urates 2. Hydrochloric acid
▪ “Brick dust” macroscopically; granular 3. Acetic acid
microscopically 4. Sodium hydroxide
★ Soluble in alkali and heat 5. Ether
▪ Associated with pink sediment after centrifugation ▪ Severe liver disease or biliary obstruction or hepatic
due to uroerythrin disease
▪ Commonly seen following refrigeration ● Iatrogenic Crystals in Acidic Urine
3. Acid urates 1. Radiographic Dye
▪ Small brown spheres; resemble ammonium biurate ▪ Colorless, notched rhombic plates; highly
crystals without the spicules birefringent
★ Soluble in alkali and heat ★ Soluble in 10% NaOH
▪ Rarely encountered and have little clinical ▪ Associated with SG > 1.040
significance ▪ Refractometry is heavily affected by radiographic
4. Sodium urates dyes
▪ Colorless birefringent needles 2. Sulfonamide
▪ Not commonly encountered in urine ▪ Colorless to yellow-brown needles, sheaves of
− More common in synovial fluid of patients with wheat, rosettes, or petals
gouty arthritis ★ Soluble in acetone
★ Soluble in alkali and heat ▪ Indicates inadequate hydration; may cause tubular
5. Calcium oxalate damage is crystals form in the nephron
▪ Dihydrate: colorless envelope 3. Ampicillin
▪ Monohydrate: oval or dumbbell ▪ Colorless needles
★ Soluble in dilute hydrochloric acid ▪ Refrigeration forms bundles
▪ Small amounts may be seen in normal urine ▪ Indicates inadequate hydration; may cause tubular
▪ Pathologic: damage is crystals form in the nephron
− Dihydrate: renal calculi ● Crystals in Alkaline Urine
− Monohydrate: hallmark crystal in ethylene glycol 1. Amorphous phosphates
poisoning (antifreeze) ▪ Milky white macroscopically
6. Calcium sulfate ★ Soluble in dilute acetic acid
▪ Colorless needles or prisms identical to calcium ▪ Commonly seen following refrigeration
phosphate 2. Calcium phosphate
★ Soluble in acetic acid ▪ Colorless, flat plates, needles or prisms often in
▪ No clinical significance rosette formation
7. Hippuric acid ★ Soluble in dilute acetic acid
▪ Yellow-brown or colorless needles, rhombic plates or ▪ Associated with renal calculi, may be seen with
prisms calcium oxalate
★ Soluble in hot water and alkali 3. Triple phosphate
▪ Ingestion of food preservatives containing benzoic ▪ A.k.a. ammonium magnesium phosphate
acid ▪ Prism shape resembling coffin lid; birefringent
● Pathologic Crystals in Acidic Urine ★ Soluble in dilute acetic acid
1. Cysteine ▪ UTI caused by urease-producing bacteria; renal
▪ Colorless, hexagonal plates calculi (struvite)
▪ May be confused with hexagonal uric acid − Urease-producing bacteria break down urea to
★ Soluble in alkali and dilute HCl ammonia
− Main differentiator between cysteine and uric acid 4. Ammonium biurate
▪ Cystinuria (associated with Fanconi syndrome) ▪ Yellow-brown spicule-covered spheres
2. Cholesterol ▪ UTI caused by urease-producing bacteria
▪ May be confused with radiographic dye ★ Soluble in acetic acid and heat
− Colorless 5. Calcium carbonate
− Notched rhombic plates ▪ Colorless dumbbell or spherical; may form clumps
− Highly birefringent ★ Soluble in acetic acid with evolution of gas
− Producing interference colors under polarizing ▪ May be mistaken as amorphous phosphates
microscope; ▪ No clinical significance
▪ Differentiate cholesterol from radiographic dye by
Summary of solubility characteristics of crystals
solubility tests
★ Cholesterol is soluble in chloroform ● Acetic acid: calcium sulfate
− Radiographic dye is soluble in 10% NaOH ● Acetic acid and heat: ammonium biurate
▪ Associated with nephrotic syndrome ● Acetic acid with evolution of gas: calcium carbonate
3. Leucine ● Acetone: sulfonamide
▪ Yellow-brown oily-looking spheres with concentric ● Alkali: uric acid
and radial striations ● Alkali and heat: amorphous urates, acid urates, sodium
★ Soluble in hot alkali or hot alcohol urates, tyrosine
▪ Severe liver disease or MSUD ● Alkali and hot water: hippuric acid

INTERNSHIP MTAP 5: ANALYSIS OF URINE AND BODY FLUIDS BSMLS 2022 9 of 16


● Alkali and dilute HCl: cysteine OTHER TESTS
● Dilute acetic acid: amorphous phosphate, calcium Normal value Clinical Significance
phosphate, triple phosphate
CSF/Serum ↑ damage to blood-brain
● Hot alkali or hot alcohol: leucine <9
albumin index barrier
● Dilute HCl: calcium oxalate
● CHASE: bilirubin CSF IgG ↑ multiple sclerosis
< 0.7
● Chloroform: cholesterol index
● 10% NaOH: radiographic dye ↓ bacterial meningitis
Glucose 50-80 mg/dL
65% of plasma glucose
V. CEREBROSPINAL FLUID ↑ bacterial meningitis,
Lactate < 25 mg/dL
A. SPECIMEN CONSIDERATIONS intracranial hemorrhage
● Technique used to collect CSF is lumbar/spinal tap ↑ liver disorders
between the 3rd and 4th or 4th and 5th lumbar vertebrae E.g. Reye syndrome, coma
● Volume collected: up to 20mL distributed into 3-4 tubes Glutamine 8-18 mg/dL of unknown origin,
→ Opening pressure must be normal to allow collection disturbance of
● All specimens should be examined under 1 hour consciousness

Tubes Tests Storage D. MICROSCOPIC EXAMINATION


1 Chemistry and Frozen ● Pleocytosis - increased number of cells in CSF
Immunology → May need to dilute specimen to facilitate counting
2 Microbiology Room temperature WBC Count
(to exclude contaminants) ● Normal values: < 5/uL (adult), < 30/uL (neonates)
3 Hematology Refrigerated ● Diluent: 3% acetic acid
(to avoid cells introduced ● Clarity and recommended dilutions
during puncture) → Slightly hazy: 1:10
4 Additional tests Depends on test to → Hazy: 1:20
be performed → Slightly cloudy: 1:100
→ Slightly bloody: 1:200
B. GROSS EXAMINATION: APPEARANCE
→ Cloudy, bloody, turbid 1:10,000
● Normal: colorless and crystal clear; non-viscous ● Calculation
● Variations: → Count 5 large squares per side of the Neubauer
→ Hazy, cloudy, turbid, milky: WBCs and chamber with a total of 10 large squares
microorganisms, high proteins (damage of BBB, MS) → Volume of each square = 0.1 mL
→ Oily: radiographic contrast media
→ Pellicle formation (web-like): tubercular meningitis
→ Bloody: traumatic tap or subarachnoid hemorrhage
→ Xanthochromic: indicated presence of RBC
Total cell count
degradation products or other pigments
● Same procedure as WBC count
Traumatic Tap Hemorrhage ● Diluent: NSS
Distribution of blood Uneven Even
Differential Cell count
Clear &
Supernatant Xanthochromic* ● Cytocentrifugation requires addition of 30% albumin, to:
Colorless
Clot formation Present Absent 1. Minimize cell distortion
D-dimer Negative Positive 2. Increase the yield
● A daily control slide for bacteria should be prepared using
Erythrophagocytosis Absent Present
distilled water and 30% albumin
● *Note: Degree of xanthochromia depends on amount of
time elapsed and amount of blood introduced Type of Cell Clinical Significance
Normal ratio
C. CHEMICAL EXAMINATION Lymphocytes and
In adults: 70:30
Monocytes
Total Protein In children: 30:70
● Normal Values: 15-45 mg/dL Bacterial meningitis, cerebral
● CSF Protein Electrophoresis Neutrophils hemorrhage; neutrophilic
→ Purposes: pleocytosis
1. Enable visualization of atypical bands Intracranial hemorrhage
2. Identify an unknown fluid as CSF Siderophage: macrophages with
Erythrophages
→ Protein fractions hemosiderin; if hemorrhage is
▪ Prealbumin, albumin, a-globulins, transferrin, tau more than 48 hrs
protein (Carbohydrate-deficient transferrin fraction), Blast forms Leukemias and lymphomas
gamma globulins (IgG, IgA) Multiple sclerosis, lymphocyte
→ Oligoclonal bands in the gamma region Plasma cells
reactions
▪ Multiple sclerosis Ependymal and Invasive diagnostic procedures
▪ Neurosyphilis choroidal cells
▪ Encephalitis Metastatic carcinomas, primary
▪ Neoplasms Malignant cells
CNS carcinoma
▪ Guillain-Barre syndrome Intracranial shunt malfunctions,
→ Note: Oligoclonal bands disappear after remission, Eosinophils eosinophilic meningitis
except in multiple sclerosis > 10%: considered significant
Bone marrow contamination during
nRBCs
spinal tap

INTERNSHIP MTAP 5: ANALYSIS OF URINE AND BODY FLUIDS BSMLS 2022 10 of 16


E. DIFFERENTIAL DIAGNOSIS OF MENINGITIS Significant when high
Lactate < 30 mg/dL
Septic arthritis
Bacterial Tubercular Fungal Viral
Significant when high
Moderate to Moderate to Moderate Total
Protein Marked ↑ < 3 g/dL Inflammatory and hemorrhagic
marked ↑ marked ↑ ↑ protein
Glucose Marked ↓ ↓ N to ↓ N arthritis
Lactate >35 mg/dL >25 mg/dL >25 mg/dL <25 mg/dL Significant when high
Uric acid 2-8 mg/dL
≥ 1,000/uL V V <100/uL Gouty arthritis
WBC
(N) (L & M) (L & M) (L)
D. MICROSCOPIC EXAMINATION
● Other tests:
1. Bacterial ● WBC count
▪ Gram stain & culture → Dilutions done only on turbid or bloody fluids using
▪ Limulus lysate test: For endotoxins of GN bacteria hypotonic saline, saponin
▪ Bacterial antigen test → Do not use acetic acid as it could cause clotting of the
2. Tubercular synovial fluid
▪ AFB and culture ● Differential count
▪ Enzyme immunoassay → Requires incubation of the fluid with hyaluronidase
3. Fungal → Recall that the synovial fluid contains hyaluronic acid
▪ Negative staining (india ink/nigrosin): C. neoformans Cellular Inclusions
▪ Cryptococcal antigen test: based on latex 1. Neutrophil
agglutination immunoassay → Normally < 25% of the differential
4. Viral → Septic or crystal-induced arthritis
▪ Immunoassay 2. Lymphocyte
▪ PCR → Normally 15% of the differential
VI. SYNOVIAL FLUID → Non-septic, viral inflammation
3. Macrophage
A. SPECIMEN CONSIDERATIONS → Normally seen
● Arthrocentesis → Non-septic/viral inflammation
→ Volume usually collected is about 25 mL 4. Synoviocyte
▪ When there is effusion associated with joint disorder → No clinical significance
→ Normal volume in knee: <3.5 mL → Cell that produces hyaluronic acid
→ Usually contains hyaluronic acid for normal viscosity → May be multinucleated resembling a mesothelial cell
● Tubes/Additives 5. LE Cell
→ Gram stain and culture (sterile heparin) → Neutrophil containing characteristic ingested round
→ Cell counts (Sodium heparin or liquid EDTA) body
▪ Powdered EDTA may leave artifacts → Seen in SLE
→ Chemical/serologic tests (non-anticoagulated) 6. Reiter cell (Neutrophage)
→ Glucose analysis (sodium fluoride) → Vacuolated macrophage with ingested neutrophils
→ Seen in Reiter syndrome
B. GROSS EXAMINATION
7. Ragocyte (RA Cell)
Normal Abnormal Clinical Significance → Neutrophil with dark granules with immune complexes
Green Septic arthritis → Seen in rheumatoid arthritis
Inflammatory and 8. Cartilage cells
Deep → Large, multinucleated cells
Colorless non-inflammatory
yellow → Seen in osteoarthritis
Color to pale arthritis
yellow Pathologic 9. Rice bodies
Red-brown hemarthrosis or → Resemble polished rice, microscopically show collagen
hemorrhagic arthritis & fibrin
WBCs, rice bodies, → Seen in tubercular, septic, and rheumatoid arthritis
Turbid 10. Fat droplets
cartilage fragments
Milky and → Refractile globules stained with Sudan
Clarity Clear Crystals → Seen in traumatic injury, chronic inflammation
opalescent
Ground Alkaptonuria 11. Hemosiderin
pepper (ochronosis) → Inclusion within clusters of synovial cells
→ Seen in pigmented villonodular synovitis
Viscosity
4-6 cm Crystal Examination
Falling No string Immunologic arthritis
string
drop ● Compensated polarized microscopy
Clot → Compensator imparts a red background
Viscosity
formation No clot ▪ Usually, polarized microscopes have a dark black
Ropes Immunologic arthritis
w/ acetic formation background
test
acid* → Crystals will appear blue or yellow against red
*Acetic acid is added to facilitate the polymerization of background depending on the birefringence nature
hyaluronic acid; will be seen as clot formation ● Negative birefringence
→ Yellow under compensated polarized light when crystal
C. CHEMICAL EXAMINATION
is parallel to slow vibration
Test Normal Clinical Significance → Blue when perpendicular to slow vibration of
10 mg/dL lower than plasma compensated polarized light
glucose ● Positive birefringence
70-110 → Blue under compensated polarized light when crystal is
Glucose Significant when low
mg/dL parallel to slow vibration
Inflammatory or septic arthritis → Yellow when perpendicular to slow vibration of
compensated polarized light
INTERNSHIP MTAP 5: ANALYSIS OF URINE AND BODY FLUIDS BSMLS 2022 11 of 16
Description MILKY
Birefring
Crystal (shape, Significance Parameters of
ence Chylous Pseudochylous
significance) Interest
Monosod Triglycerides > 110 mg/dL < 50 mg/dL
Negative Needles Gout
ium urate Sudan III staining Strongly (+) Weak or -
Calcium Presence of
Rhombic, Cholesterol
pyrophos Positive Pseudogout Absent cholesterol
square rods crystals
phate crystals
Choleste Notched, Chronic Lymphatic Chronic
Negative Results from
rol rhombic plates Inflammation obstruction inflammation
Positive Flat,
Corticost Injection of C. TRANSUDATES VS EXUDATES
and variable-shaped
eroid corticosteroids
Negative plates TRANSUDATE EXUDATE
Calcium Result from Non-inflammatory Inflammatory
Negative Envelopes Renal dialysis
oxalate Systemic process Localized process
Calcium LDH < 200 IU/L ≥ 200 IU/L
phosphat or > ⅔ of plasma
None Small particles Osteoarthritis
e (a.k.a. LDH
apatite) F:S LDH ratio < 0.6 ≥ 0.6
E. DIFFERENTIAL DIAGNOSIS OF JOINT DISORDERS F:S protein ratio < 0.5 ≥ 0.5
● Group I: Non inflammatory Total protein < 3 g/dL ≥3
→ E.g. osteoarthritis Rivalta’s test Negative (-) Positive (+)
● Group II: Inflammatory
→ Immunologic: SLE, RA, Lyme arthritis Test for protein
→ Crystal induced: Gout, pseudogout based on acid
● Group III: Septic precipitation
→ E.g. bacterial infection (acetic acid)
● Group IV: Hemorrhagic Cholesterol < 45-60 mg/dL ≥ 45-60 mg/dL
→ E.g. trauma, hemophilia, tumor F:S cholesterol < 0.3 ≥ 0.3
Grp Viscosity Glucose WBCs/ul PMN ratio
(Ropes test) F:S bilirubin ratio < 0.6 ≥ 0.6
I Good dog N < 1,000 < 30 Serum-ascites > 1.1 ≤ 1.1
II Poor* ↓ 2,000-75,000* > 50* albumin gradient
Low** up to 100,000** < 70**
III Variable ↓ 50,000-100,000 > 75 Work-up Tests
IV Low N Same as blood < 50 Test Level Significance
*Group II (inflammatory) Immunologic Rheumatoid inflammation and
**Group II (inflammatory) Crystal-induced Glucose ↓
purulent infection
VII. SEROUS FLUIDS Lactate ↑ Bacterial infection
Triglyceride ↑ Chylous effusions
A. SPECIMEN CONSIDERATIONS Pneumonia (<7.2); esophageal
● Ultrafiltrate of plasma pH ↓
rupture (<6.0)
● Collection techniques Pancreatitis, esophageal rupture,
→ Thoracentesis: pleural fluid Amylase ↑
GI perforation
→ Pericardiocentesis: pericardial fluid ALP ↑ GI perforation
→ Paracentesis: peritoneal fluid ADA ↑ Tubercular effusions
● Collection tubes
BUN and
→ Cell counts and differential: EDTA ↑ Punctured bladder
creatinine
→ Microbiology and cytology: Sterile heparin
→ Chemistry tests: Non-anticoagulated D. MICROSCOPIC EXAMINATION
→ pH: Heparinized; maintain anaerobic condition; ● Differential count: Routinely performed to examine WBCs
transport in ice and demonstrate malignant cells
B. GROSS EXAMINATION: APPEARANCE Test Cell types Significance
● Normal: clear and colorless to pale yellow Mesothelial cells ↓ in tuberculosis
BLOODY Plasma cells ↑ in tuberculosis
Hemothorax Hemorrhagic Pleural Eosinophils (>10%) Hemothorax,
TT effusion fluid pneumothorax, allergic
Distribution of Uneven/streaked/ reaction, parasitic
Even infection
blood diminishing
> 50% of blood < 50% of blood WBC > 500 cells/uL Bacterial peritonitis
Fluid Hematocrit Hct Hct (exudative), cirrhosis
compare w/ blood Principle: (transudative)*
Hct If blood has been introduced, then Absolute Bacterial peritonitis
Peritoneal
its Hct will be close to that of blood Hct granulocyte count
fluid
> 250/uL
RBC > 100,000/uL Malignancy
Psammoma bodies Benign tumors, ovarian
and thyroid CA

INTERNSHIP MTAP 5: ANALYSIS OF URINE AND BODY FLUIDS BSMLS 2022 12 of 16


RBC > 100,000/uL Intraabdominal 3. Liquefaction
Peritoneal → Normal 30-60 minutes
bleeding (blunt trauma
lavage → Abnormal:
injury)
● *Note: Must perform absolute granulocyte count to ▪ > 2 hours: Deficiency in prostatic enzymes
differentiate between peritonitis and cirrhosis 4. pH
→ >250/uL: bacterial peritonitis → Normal: 7.2-8.0
→ <250/uL: cirrhosis → Abnormal:
● Psammoma bodies → represent concentric striations of ▪ < 7.2: Increased prostatic fluid
collagen-like materials ▪ > 8.0: Infection within the reproductive tract
5. Viscosity
VIII. SEMEN → Normal: pours in droplets
Contribution to → Abnormal:
Structure Function ▪ Clumped, stringy or gel-like: Deficiency in prostatic
semen volume
Site of enzymes
Testes ➢ Note: Numeric terms can be used to grade viscosity
spermatogenesis
5% sperm → 0 indicating a normal, watery specimen forming
Site of sperm
Epididymis discrete drops
maturation
60-70% seminal → 4 indicating a specimen with gel-like consistency
plasma C. ROUTINE MICROSCOPIC EXAMINATION
Seminal Provide alkaline fluid
containing
vesicles and nutrients for sperm Sperm Motility
fructose and
flavin (yellow) ● Performed on well mixed, undiluted semen immediately
20-30% acidic upon liquefaction
Secrete enzymes for fluid containing ● Speed and direction are both evaluated using
Prostate approximately 20 HPFs:
coagulation and citric acid, zinc,
gland
liquefaction and acid 4 Rapid, straight line motility
phosphatase (strong linear or forward progression)
Bulbourethral Neutralize prostatic 5% alkaline 3 Slower speed, some lateral movement
glands fluid and vaginal acidity mucus (moderate linear or forward progression)
A. SPECIMEN CONSIDERATIONS 2 Slow progression, noticeable lateral movement
(slow nonlinear or meandering progression)
Specimen collection 1 No forward progression
● Masturbation - recommended method 0 No movement (immotile)
● Condom method - silastic or non-lubricant containing
rubber or polyurethane condoms ● Normal motility: > 50% within 1 hour (quality > 2.0)
● Vaginal aspiration - for post-coital tests ● Causes of abnormal motility:
● Coitus interruptus - withdrawal method; not → Midpiece and tail abnormalities, male antisperm
recommended due to the possibility of losing the first antibodies (causes sperms to agglutinate)
portion of semen, which contain a great number of sperm Sperm Concentration and Sperm Count
Important considerations ● Dilution: 1:20 using sodium bicarbonate in formalin, saline,
● Patient preparation or distilled water
→ Sexual asbtiennce of 2-5, up to 7 days ● Sperm concentration (sperm/mL)
→ Must empty bladder before collection → Calculation: (shortcut)
● Specimen container ▪ 105 when using 2 large (WBC) squares per side
→ Prewarmed sterile glass or plastic containers ▪ 106 when using 5 RBC squares per side
● Transport
→ Kept at 37°C → Neubauer formula:
→ Delivered to the laboratory within 1 hour of collection
● Fertility testing
→ 2-3 samples tested at 2-week intervals
→ 2 abnormal samples considered significant ● Sperm count (sperm/ejaculate)
● Fructose test
→ Transported in ice and frozen within 2 hours of
collection if analysis will be delayed to prevent ● Normal values:
fructolysis → Concentration: >20,000,000/mL
→ Count: >40,000,000/ejaculate
B. GROSS EXAMINATION ● Clinical significance of abnormal counts:
1. Appearance → Azoospermia, oligospermia
→ Normal: Gray-white; translucent
Sperm Morphology
→ Abnormal:
▪ Yellow: Pyospermia, urine contamination, prolonged ● Evaluation is done on a thin smear stained using Wright’s,
abstinence (due to accumulation of flavin) Giemsa, or Papanicolau (best stain)
▪ Red/rust color: Bleeding ● 200 sperm are evaluated under OIO for abnormalities in
▪ Turbid: Infection the head, midpiece, and tail
▪ Clear: Azoospermia ● Strict criteria:
2. Volume → Oval-shaped head (approx. 3x5 um)
→ Normal: 2-5 mL → Tail (approx. 45 um long)
→ Abnormal: → No big cytoplasmic droplet
▪ < 2mL: Improper functioning of one of the → Acrosome >50% of head volume; covering ⅔ of
seme-producing organs nucleus
▪ > 5mL: Prolonged abstinence (decreased motility) ● % Normal forms:
→ >14% (strict); >30% (routine)

INTERNSHIP MTAP 5: ANALYSIS OF URINE AND BODY FLUIDS BSMLS 2022 13 of 16


IX. FECES D. CHEMICAL EXAMINATION
● Specimen considerations: VAN DE KAMER
→ Routine fecalysis: random specimen Gold Standard; requires 3-day stool collection
→ Quantitative fecal fat analysis: 3-day collection Method/principle Hydrolysis; fecal fat extraction and
A. GROSS EXAMINATION titration of fatty acids using NaOH
Reagent NaOH
Appearance Clinical Significance
Result 1-6 g/day - Normal
Brown Normal (stercobilin, urobilin)
>6 g/day - Pathologic
Pathologic: Upper GI bleeding
Significance >6 g/day indicative of steatorrhea
Black/Tarry Nonpathologic: Iron, bismuth,
charcoal TRYPSIN
Pathologic: Lower GI bleeding Method/principle Gelatin hydrolysis
Red Nonpathologic: Beets, food Reagent Gelatin on x-ray film
coloring, rifampin Result Clearing: evidence of hydrolysis of
Pathologic: Bile duct obstruction trypsin
Pale yellow, white, (acholic stool: absence of bile No clarity: pathologic
or gray pigment) Significance Pancreatic insufficiency (no clearing)
Nonpathologic: barium sulfate
Biliverdin, oral antibiotics, green CARBOHYDRATES
Green Method/principle Copper reduction
vegetables
Pathologic: Bile duct obstruction, Reagent Clinitest
Bulky, frothy pancreatic insufficiency Result Orange-red
(steatorrhea) Significance Carbohydrate intolerance
Ribbon-like, slender Intestinal constriction APT TEST
Small, hard Constipation Method/principle Differentiation between fetal and
Watery Diarrhea maternal blood
Fetal hemoglobin is resistant to
B. TYPES OF DIARRHEA
denaturation
Secretory Osmotic Reagent NaOH
Increased secretion of Water and electrolyte Result Pink/Bloody
water and electrolytes retention in the large Yellow-brown
Cause due to bacterial, viral intestine due to Significance Pink - fetal hemoglobin (fetal blood)
or protozoan maldigestion or Yellow-brown - adult hemoglobin
infections malabsorption (maternal blood)
Output > 200 g < 200 g
Guaiac-based FOBT
pH > 5.6 < 5.3
Method/principle Pseudoperoxidase reaction
Fecal > 90 mmol/L < 60 mmol/L
sodium Reagent Guaiac
Osmotic < 50 mOsm/kg > 50 mOsm/kg Result (+) Blue color
gap Significance GI bleeding (colon cancer)
Stool culture, fecal Fecal fats, muscle Interferences F(+): red meat, vegetable
Common leukocytes fiber, trypsin*, peroxidases (e.g. horseradish,
tests Clinitest**, cauliflower, broccoli), aspirin,
D-xylose*** non-steroidal anti-inflammatory drugs
● *Trypsin: test for pancreatic insufficiency (NSAIDS)
● **Clinitest: test for carbohydrate intolerance F(-): ascorbic acid
● *** D-xylose absorption test : differentiates malabsorption
and maldigestion
→ Administer D-xylose and following its excretion in urine
▪ 5-hr urine collection
C. MICROSCOPIC EXAMINATION
Test Method/Principle Significance
Stained wet mount > 3/HPF indicates
Fecal (methylene blue) invasive condition
neutrophils Dried smear
(Wright’s)
Examination of >10 undigested
Undigested
smear stained with muscle fibers
muscle
10% alcoholic eosin indicate pancreatic
fibers
to visualize striations insufficiency
Examination of direct 60 large orange-red
smear stained with droplets indicate
Sudan III (for neutral malabsorption
fats)
Qualitative
Examination of 100 orange-red
fecal fats
smear heated with droplets (6-75um)
36% acetic acid and indicate
Sudan III (for total fat malabsorption
content)

INTERNSHIP MTAP 5: ANALYSIS OF URINE AND BODY FLUIDS BSMLS 2022 14 of 16


INDEX: APPENDIX
Parameter Principle Reagents Color Time Sensitivity Interferences
Specific pKa change of Polyelectrolytes Blue-green-yello 45s 1.000-1.030 F(+): high
Gravity polyelectrolytes Bromthymol blue w concentrations of protein
Random: (pH indicator)
1.002-1.035 F(-): highly alkaline urine
pH Double indicator Methyl red Red-yellow-blue 60s Multistix: 5-8.5 F(+): Old specimens
Random: system Bromthymol blue
4.5-8.0 Chemstrip: F(-): Runover from
Fasting/first 5-9 adjacent pads
AM: 5-6
(acidic or
slightly
acidic)
Protein error of Tetrabromphenol Green to blue 60s Multistix: F(+): increased pH,
indicators blue or 15-30 mg/dL pigmented
tetrachlorophenol specimens,
tetrabromosulfonpht Chemstrip: 6 chlorhexidine,
halein mg/dL phenazopyridine,
QACs (detergents),
Buffer (pH 3.0) antiseptics, loss of
buffer (due to
Protein
overexposure),
<30 mg/dL or
increased SG
<150 mg/day
F(-): presence of
proteins other than
albumin (globulins),
high salt
concentration,
microalbuminuria

Glucose GOD, POD Multistix: 30s Multistix: F(+): strong oxidizing


Glucose oxidase green-brown 75-115 mg/dL agents (detergents)
< 15 mg/dL reaction/ KI (multistix) or
Renal Double tetramethylbenzidine Chemstrip: Chemstrip: 40 F(-): reducing agents
threshold: sequential (chemstrip) yellow-green mg/dL (ascorbic acid, ketones,
160-180 enzyme drug metabolites), low
mg/dL reaction temperature, old
specimen
Sodium Sodium Purple 40s Multistix: F(+): phthalein dyes, red
nitroprusside nitroprusside ± 5-10 mg/dL urine, levodopa,
reaction glycine acetoacetic medications
W/ glycine: acid (sulfhydryl groups)
Chemstrip
Ketones W/out glycine: Chemstrip: F(-): old specimen,
Multistix 9 mg/dL improperly preserved
acetoacetic specimen
acid
70 mg/dL
acetone
Pseudoperoxida Lysing agent Blue-green 60s 5-20 F(+): Strong oxidizing
se activity of Peroxide (OA) RBCs/mL agents, bacterial
heme Tetramethylbenzidin Speckled or peroxidases,
e uniform menstrual
appearance contamination
Blood
F(-): high SG, crenated
cells, nitrite, formalin,
captopril, ascorbic
acid
Diazo reaction Multistix: Tan, Pink, Violet 30s Multistix: F(+): highly pigmented
Dichloroaniline 0.4-0.8 urine, indican,
diazonium salt mg/dL phenazopyridine,
metabolites of Lodine
Bilirubin Chemstrip: Chemstrip:
dichlorobenzene 0.5 mg/dL F(-): Exposure to light,
diazonium ascorbic acid, high
tetrafluoroborate concentration of
nitrite

INTERNSHIP MTAP 5: ANALYSIS OF URINE AND BODY FLUIDS BSMLS 2022 15 of 16


Ehrlich reaction Multistix: PDAB Red 60s Multistix: 0.2 F(+): Porphobilinogen,
mg/dL Ehrlich-reactive
Urobilinogen
Chemstrip: compounds, indican,
< 1.0 mg/dL
Methoxybenzene Chemstrip: sulfonamides
< 1.0 Ehrlich
diazonium salt 0.4 mg/dL
unit
F(-): old specimen,
formalin, nitrite
Greiss’ reaction Multistix: p-arsanilic Pink 60s Multistix: F(+): Old specimen
acid THBQ or 0.06-0.10
mg/dL F(-): non
Chemstrip: reductase-containing
sulfanilamide-hydrox Chemstrip: bacteria, insufficient
Nitrite
y THBQ 0.05 mg/dL contact time*, lack of
urinary nitrate,
bacterial overgrowth,
antibiotics, ascorbic
acid, high SG
Granulocyte Multistix: amino acid Purple 120s Multistix: 5-10 F(+): Strong OAs,
esterase ester WBC/HPF formalin, highly
reaction pigmented urine,
Chemstrip: Chemstrip: nitrofurantoin
Leukocyte
indoxylcarbonic acid 10-25
Esterase
ester WBC/HPF F(-): high protein,
glucose, oxalic acid,
Both: Diazonium salt ascorbic acid,
antibiotics

INTERNSHIP MTAP 5: ANALYSIS OF URINE AND BODY FLUIDS BSMLS 2022 16 of 16

You might also like